User login
Is chest radiography routinely needed after thoracentesis?
No. After thoracentesis, chest radiography or another lung imaging study should be done only if pneumothorax is suspected, if thoracentesis requires more than 1 attempt, if the patient is on mechanical ventilation or has pre-existing lung disease, or if a large volume (> 1,500 mL) of fluid is removed. Radiography is also usually not necessary after diagnostic thoracentesis in a patient breathing spontaneously. In most cases, pneumothorax found incidentally after thoracentesis does not require decompression and can be managed supportively.
WHAT ARE THE RISKS OF THORACENTESIS?
Thoracentesis is a minimally invasive procedure usually performed at the bedside that involves insertion of a needle into the pleural cavity for drainage of fluid.1 Diagnostic thoracentesis should be done in most cases of a new pleural effusion unless the effusion is small and with a clear diagnosis, or in cases of typical heart failure.
Therapeutic thoracentesis, often called large-volume thoracentesis, aims to improve symptoms such as dyspnea attributed to the pleural effusion by removing at least 1 L of pleural fluid. The presence of active respiratory symptoms and suspicion of infected pleural effusion should lead to thoracentesis as soon as possible.
Complications of thoracentesis may be benign, such as pain and anxiety associated with the procedure and external bleeding at the site of needle insertion. Pneumothorax is the most common serious procedural complication and the principal reason to order postprocedural chest radiography.1 Less common complications include hemothorax, re-expansion pulmonary edema, infection, subdiaphragmatic organ puncture, and procedure-related death. Bleeding complications and hemothorax are rare even in patients with underlying coagulopathy.2
Point-of-care pleural ultrasonography is now considered the standard of care to guide optimal needle location for the procedure and to exclude other conditions that can mimic pleural effusion on chest radiography, such as lung consolidation and atelectasis.3 High proficiency in the use of preprocedural point-of-care ultrasonography reduces the rate of procedural complications, though it does not eliminate the risk entirely.3,4
Factors associated with higher rates of complications include lack of operator proficiency, poor understanding of the anatomy, poor patient positioning, poor patient cooperation with the procedure, lack of availability of bedside ultrasonography, and drainage of more than 1,500 mL of fluid. Addressing these factors has been shown to decrease the risk of pneumothorax and infection.1–5
HOW OFTEN DOES PNEUMOTHORAX OCCUR AFTER THORACENTESIS?
Several early studies have examined the incidence of pneumothorax after thoracentesis. Lack of ultrasonography use likely explains a higher incidence of complications in early studies: rates of pneumothorax after thoracentesis without ultrasonographic guidance ranged from 5.2% to 26%.6,7
Gervais et al8 analyzed thoracentesis with ultrasonographic guidance in 434 patients, 92 of whom were intubated, and reported that pneumothorax occurred in 10 patients, of whom 6 were intubated. Two of the intubated patients required chest tubes. Other studies have confirmed the low incidence of pneumothorax in patients undergoing thoracentesis, with rates such as 0.61%,1 5%,9 and 4%.10
The major predictor of postprocedural pneumothorax was the presence of symptoms such as chest pain and dyspnea. No intervention was necessary for most cases of pneumothorax in asymptomatic patients. The more widespread use of procedural ultrasonography may explain some discrepancies between the early5,6 and more recent studies.1,8–10
Several studies have demonstrated that postprocedural radiography is unnecessary unless a complication is suspected based on the patient’s symptoms or the need to demonstrate lung re-expansion.1,4,9,10 Clinical suspicion and the patient’s symptoms are the major predictors of procedure-related pneumothorax requiring treatment with a chest tube. Otherwise, incidentally discovered pneumothorax can usually be observed and managed supportively.
WHAT MECHANISMS UNDERLIE POSTPROCEDURAL PNEUMOTHORAX?
Major causes of pneumothorax in patients undergoing thoracentesis are direct puncture during needle or catheter insertion, the introduction of air through the needle or catheter into the pleural cavity, and the inability of the ipsilateral lung to fully expand after drainage of a large volume of fluid, known as pneumothorax ex vacuo.5
Pneumothorax ex vacuo may be seen in patients with medical conditions such as endobronchial obstruction, pleural scarring from long-standing pleural effusion, and lung malignancy, all of which can impair the lung’s ability to expand after removal of a large volume of pleural fluid. It is believed that transient parenchymal pleural fistulae form if the lung cannot expand, causing air leakage into the pleural cavity.5,8,9 Pleural manometry to monitor changes in pleural pressure and elastance can decrease the rates of pneumothorax ex vacuo in patients with the above risk factors.5
WHEN IS RADIOGRAPHY INDICATED AFTER THORACENTESIS?
Current literature suggests that imaging to evaluate for postprocedural complications should be done if there is suspicion of a complication, if thoracentesis required multiple attempts, if the procedure caused aspiration of air, if the patient has advanced lung disease, if the patient is scheduled to undergo thoracic radiation, if the patient is on mechanical ventilation, and after therapeutic thoracentesis if a large volume of fluid is removed.1–10 Routine chest radiography after thoracentesis is not supported in the literature in the absence of these risk factors.
Some practitioners order chest imaging after therapeutic thoracentesis to assess for residual pleural fluid and for visualization of other abnormalities previously hidden by pleural effusion, rather than simply to exclude postprocedural pneumothorax. Alternatively, postprocedural bedside pleural ultrasonography with recording of images can be done to assess for complications and residual pleural fluid volume without exposing the patient to radiation.11
Needle decompression and chest tube insertion should be considered in patients with tension pneumothorax, large pneumothorax (distance from the chest wall to the visceral pleural line of at least 2 cm), mechanical ventilation, progressing pneumothorax, and symptoms.
KEY POINTS
- Pneumothorax is a rare complication of thoracentesis when performed by a skilled operator using ultrasonographic guidance.
- Mechanisms behind the occurrence of pneumothorax are direct lung puncture, introduction of air into the pleural cavity, and pneumothorax ex vacuo.
- In asymptomatic patients, pneumothorax after thoracentesis rarely requires intervention beyond supportive care and close observation.
- Factors such as multiple thoracentesis attempts, symptoms, clinical suspicion, air aspiration during thoracentesis, presence of previous lung disease, and removal of a large volume of fluid may require postprocedural lung imaging (eg, bedside ultrasonography, radiography).
- Ault MJ, Rosen BT, Scher J, Feinglass J, Barsuk JH. Thoracentesis outcomes: a 12-year experience. Thorax 2015; 70(2):127–132. doi:10.1136/thoraxjnl-2014-206114
- Hibbert RM, Atwell TD, Lekah A, et al. Safety of ultrasound-guided thoracentesis in patients with abnormal preprocedural coagulation parameters. Chest 2013; 144(2):456–463. doi:10.1378/chest.12-2374
- Barnes TW, Morgenthaler TI, Olson EJ, Hesley GK, Decker PA, Ryu JH. Sonographically guided thoracentesis and rate of pneumothorax. J Clin Ultrasound 2005; 33(9):442–446. doi:10.1002/jcu.20163
- Gordon CE, Feller-Kopman D, Balk EM, Smetana GW. Pneumothorax following thoracentesis: a systematic review and meta-analysis. Arch Intern Med 2010; 170(4):332–339. doi:10.1001/archinternmed.2009.548
- Heidecker J, Huggins JT, Sahn SA, Doelken P. Pathophysiology of pneumothorax following ultrasound-guided thoracentesis. Chest 2006; 130(4):1173–1184. doi:10.1016/S0012-3692(15)51155-0
- Brandstetter RD, Karetzky M, Rastogi R, Lolis JD. Pneumothorax after thoracentesis in chronic obstructive pulmonary disease. Heart Lung 1994; 23(1):67–70. pmid:8150647
- Doyle JJ, Hnatiuk OW, Torrington KG, Slade AR, Howard RS. Necessity of routine chest roentgenography after thoracentesis. Ann Intern Med 1996; 124(9):816–820. pmid:8610950
- Gervais DA, Petersein A, Lee MJ, Hahn PF, Saini S, Mueller PR. US-guided thoracentesis: requirement for postprocedure chest radiography in patients who receive mechanical ventilation versus patients who breathe spontaneously. Radiology 1997; 204(2):503–506. doi:10.1148/radiology.204.2.9240544
- Capizzi SA, Prakash UB. Chest roentgenography after outpatient thoracentesis. Mayo Clin Proc 1998; 73(10):948–950. doi:10.4065/73.10.948
- Alemán C, Alegre J, Armadans L, et al. The value of chest roentgenography in the diagnosis of pneumothorax after thoracentesis. Am J Med 1999; 107(4):340–343. pmid:10527035
- Lichtenstein D. Lung ultrasound in the critically ill. Curr Opin Crit Care 2014; 20(3):315–322. doi:10.1097/MCC.0000000000000096
No. After thoracentesis, chest radiography or another lung imaging study should be done only if pneumothorax is suspected, if thoracentesis requires more than 1 attempt, if the patient is on mechanical ventilation or has pre-existing lung disease, or if a large volume (> 1,500 mL) of fluid is removed. Radiography is also usually not necessary after diagnostic thoracentesis in a patient breathing spontaneously. In most cases, pneumothorax found incidentally after thoracentesis does not require decompression and can be managed supportively.
WHAT ARE THE RISKS OF THORACENTESIS?
Thoracentesis is a minimally invasive procedure usually performed at the bedside that involves insertion of a needle into the pleural cavity for drainage of fluid.1 Diagnostic thoracentesis should be done in most cases of a new pleural effusion unless the effusion is small and with a clear diagnosis, or in cases of typical heart failure.
Therapeutic thoracentesis, often called large-volume thoracentesis, aims to improve symptoms such as dyspnea attributed to the pleural effusion by removing at least 1 L of pleural fluid. The presence of active respiratory symptoms and suspicion of infected pleural effusion should lead to thoracentesis as soon as possible.
Complications of thoracentesis may be benign, such as pain and anxiety associated with the procedure and external bleeding at the site of needle insertion. Pneumothorax is the most common serious procedural complication and the principal reason to order postprocedural chest radiography.1 Less common complications include hemothorax, re-expansion pulmonary edema, infection, subdiaphragmatic organ puncture, and procedure-related death. Bleeding complications and hemothorax are rare even in patients with underlying coagulopathy.2
Point-of-care pleural ultrasonography is now considered the standard of care to guide optimal needle location for the procedure and to exclude other conditions that can mimic pleural effusion on chest radiography, such as lung consolidation and atelectasis.3 High proficiency in the use of preprocedural point-of-care ultrasonography reduces the rate of procedural complications, though it does not eliminate the risk entirely.3,4
Factors associated with higher rates of complications include lack of operator proficiency, poor understanding of the anatomy, poor patient positioning, poor patient cooperation with the procedure, lack of availability of bedside ultrasonography, and drainage of more than 1,500 mL of fluid. Addressing these factors has been shown to decrease the risk of pneumothorax and infection.1–5
HOW OFTEN DOES PNEUMOTHORAX OCCUR AFTER THORACENTESIS?
Several early studies have examined the incidence of pneumothorax after thoracentesis. Lack of ultrasonography use likely explains a higher incidence of complications in early studies: rates of pneumothorax after thoracentesis without ultrasonographic guidance ranged from 5.2% to 26%.6,7
Gervais et al8 analyzed thoracentesis with ultrasonographic guidance in 434 patients, 92 of whom were intubated, and reported that pneumothorax occurred in 10 patients, of whom 6 were intubated. Two of the intubated patients required chest tubes. Other studies have confirmed the low incidence of pneumothorax in patients undergoing thoracentesis, with rates such as 0.61%,1 5%,9 and 4%.10
The major predictor of postprocedural pneumothorax was the presence of symptoms such as chest pain and dyspnea. No intervention was necessary for most cases of pneumothorax in asymptomatic patients. The more widespread use of procedural ultrasonography may explain some discrepancies between the early5,6 and more recent studies.1,8–10
Several studies have demonstrated that postprocedural radiography is unnecessary unless a complication is suspected based on the patient’s symptoms or the need to demonstrate lung re-expansion.1,4,9,10 Clinical suspicion and the patient’s symptoms are the major predictors of procedure-related pneumothorax requiring treatment with a chest tube. Otherwise, incidentally discovered pneumothorax can usually be observed and managed supportively.
WHAT MECHANISMS UNDERLIE POSTPROCEDURAL PNEUMOTHORAX?
Major causes of pneumothorax in patients undergoing thoracentesis are direct puncture during needle or catheter insertion, the introduction of air through the needle or catheter into the pleural cavity, and the inability of the ipsilateral lung to fully expand after drainage of a large volume of fluid, known as pneumothorax ex vacuo.5
Pneumothorax ex vacuo may be seen in patients with medical conditions such as endobronchial obstruction, pleural scarring from long-standing pleural effusion, and lung malignancy, all of which can impair the lung’s ability to expand after removal of a large volume of pleural fluid. It is believed that transient parenchymal pleural fistulae form if the lung cannot expand, causing air leakage into the pleural cavity.5,8,9 Pleural manometry to monitor changes in pleural pressure and elastance can decrease the rates of pneumothorax ex vacuo in patients with the above risk factors.5
WHEN IS RADIOGRAPHY INDICATED AFTER THORACENTESIS?
Current literature suggests that imaging to evaluate for postprocedural complications should be done if there is suspicion of a complication, if thoracentesis required multiple attempts, if the procedure caused aspiration of air, if the patient has advanced lung disease, if the patient is scheduled to undergo thoracic radiation, if the patient is on mechanical ventilation, and after therapeutic thoracentesis if a large volume of fluid is removed.1–10 Routine chest radiography after thoracentesis is not supported in the literature in the absence of these risk factors.
Some practitioners order chest imaging after therapeutic thoracentesis to assess for residual pleural fluid and for visualization of other abnormalities previously hidden by pleural effusion, rather than simply to exclude postprocedural pneumothorax. Alternatively, postprocedural bedside pleural ultrasonography with recording of images can be done to assess for complications and residual pleural fluid volume without exposing the patient to radiation.11
Needle decompression and chest tube insertion should be considered in patients with tension pneumothorax, large pneumothorax (distance from the chest wall to the visceral pleural line of at least 2 cm), mechanical ventilation, progressing pneumothorax, and symptoms.
KEY POINTS
- Pneumothorax is a rare complication of thoracentesis when performed by a skilled operator using ultrasonographic guidance.
- Mechanisms behind the occurrence of pneumothorax are direct lung puncture, introduction of air into the pleural cavity, and pneumothorax ex vacuo.
- In asymptomatic patients, pneumothorax after thoracentesis rarely requires intervention beyond supportive care and close observation.
- Factors such as multiple thoracentesis attempts, symptoms, clinical suspicion, air aspiration during thoracentesis, presence of previous lung disease, and removal of a large volume of fluid may require postprocedural lung imaging (eg, bedside ultrasonography, radiography).
No. After thoracentesis, chest radiography or another lung imaging study should be done only if pneumothorax is suspected, if thoracentesis requires more than 1 attempt, if the patient is on mechanical ventilation or has pre-existing lung disease, or if a large volume (> 1,500 mL) of fluid is removed. Radiography is also usually not necessary after diagnostic thoracentesis in a patient breathing spontaneously. In most cases, pneumothorax found incidentally after thoracentesis does not require decompression and can be managed supportively.
WHAT ARE THE RISKS OF THORACENTESIS?
Thoracentesis is a minimally invasive procedure usually performed at the bedside that involves insertion of a needle into the pleural cavity for drainage of fluid.1 Diagnostic thoracentesis should be done in most cases of a new pleural effusion unless the effusion is small and with a clear diagnosis, or in cases of typical heart failure.
Therapeutic thoracentesis, often called large-volume thoracentesis, aims to improve symptoms such as dyspnea attributed to the pleural effusion by removing at least 1 L of pleural fluid. The presence of active respiratory symptoms and suspicion of infected pleural effusion should lead to thoracentesis as soon as possible.
Complications of thoracentesis may be benign, such as pain and anxiety associated with the procedure and external bleeding at the site of needle insertion. Pneumothorax is the most common serious procedural complication and the principal reason to order postprocedural chest radiography.1 Less common complications include hemothorax, re-expansion pulmonary edema, infection, subdiaphragmatic organ puncture, and procedure-related death. Bleeding complications and hemothorax are rare even in patients with underlying coagulopathy.2
Point-of-care pleural ultrasonography is now considered the standard of care to guide optimal needle location for the procedure and to exclude other conditions that can mimic pleural effusion on chest radiography, such as lung consolidation and atelectasis.3 High proficiency in the use of preprocedural point-of-care ultrasonography reduces the rate of procedural complications, though it does not eliminate the risk entirely.3,4
Factors associated with higher rates of complications include lack of operator proficiency, poor understanding of the anatomy, poor patient positioning, poor patient cooperation with the procedure, lack of availability of bedside ultrasonography, and drainage of more than 1,500 mL of fluid. Addressing these factors has been shown to decrease the risk of pneumothorax and infection.1–5
HOW OFTEN DOES PNEUMOTHORAX OCCUR AFTER THORACENTESIS?
Several early studies have examined the incidence of pneumothorax after thoracentesis. Lack of ultrasonography use likely explains a higher incidence of complications in early studies: rates of pneumothorax after thoracentesis without ultrasonographic guidance ranged from 5.2% to 26%.6,7
Gervais et al8 analyzed thoracentesis with ultrasonographic guidance in 434 patients, 92 of whom were intubated, and reported that pneumothorax occurred in 10 patients, of whom 6 were intubated. Two of the intubated patients required chest tubes. Other studies have confirmed the low incidence of pneumothorax in patients undergoing thoracentesis, with rates such as 0.61%,1 5%,9 and 4%.10
The major predictor of postprocedural pneumothorax was the presence of symptoms such as chest pain and dyspnea. No intervention was necessary for most cases of pneumothorax in asymptomatic patients. The more widespread use of procedural ultrasonography may explain some discrepancies between the early5,6 and more recent studies.1,8–10
Several studies have demonstrated that postprocedural radiography is unnecessary unless a complication is suspected based on the patient’s symptoms or the need to demonstrate lung re-expansion.1,4,9,10 Clinical suspicion and the patient’s symptoms are the major predictors of procedure-related pneumothorax requiring treatment with a chest tube. Otherwise, incidentally discovered pneumothorax can usually be observed and managed supportively.
WHAT MECHANISMS UNDERLIE POSTPROCEDURAL PNEUMOTHORAX?
Major causes of pneumothorax in patients undergoing thoracentesis are direct puncture during needle or catheter insertion, the introduction of air through the needle or catheter into the pleural cavity, and the inability of the ipsilateral lung to fully expand after drainage of a large volume of fluid, known as pneumothorax ex vacuo.5
Pneumothorax ex vacuo may be seen in patients with medical conditions such as endobronchial obstruction, pleural scarring from long-standing pleural effusion, and lung malignancy, all of which can impair the lung’s ability to expand after removal of a large volume of pleural fluid. It is believed that transient parenchymal pleural fistulae form if the lung cannot expand, causing air leakage into the pleural cavity.5,8,9 Pleural manometry to monitor changes in pleural pressure and elastance can decrease the rates of pneumothorax ex vacuo in patients with the above risk factors.5
WHEN IS RADIOGRAPHY INDICATED AFTER THORACENTESIS?
Current literature suggests that imaging to evaluate for postprocedural complications should be done if there is suspicion of a complication, if thoracentesis required multiple attempts, if the procedure caused aspiration of air, if the patient has advanced lung disease, if the patient is scheduled to undergo thoracic radiation, if the patient is on mechanical ventilation, and after therapeutic thoracentesis if a large volume of fluid is removed.1–10 Routine chest radiography after thoracentesis is not supported in the literature in the absence of these risk factors.
Some practitioners order chest imaging after therapeutic thoracentesis to assess for residual pleural fluid and for visualization of other abnormalities previously hidden by pleural effusion, rather than simply to exclude postprocedural pneumothorax. Alternatively, postprocedural bedside pleural ultrasonography with recording of images can be done to assess for complications and residual pleural fluid volume without exposing the patient to radiation.11
Needle decompression and chest tube insertion should be considered in patients with tension pneumothorax, large pneumothorax (distance from the chest wall to the visceral pleural line of at least 2 cm), mechanical ventilation, progressing pneumothorax, and symptoms.
KEY POINTS
- Pneumothorax is a rare complication of thoracentesis when performed by a skilled operator using ultrasonographic guidance.
- Mechanisms behind the occurrence of pneumothorax are direct lung puncture, introduction of air into the pleural cavity, and pneumothorax ex vacuo.
- In asymptomatic patients, pneumothorax after thoracentesis rarely requires intervention beyond supportive care and close observation.
- Factors such as multiple thoracentesis attempts, symptoms, clinical suspicion, air aspiration during thoracentesis, presence of previous lung disease, and removal of a large volume of fluid may require postprocedural lung imaging (eg, bedside ultrasonography, radiography).
- Ault MJ, Rosen BT, Scher J, Feinglass J, Barsuk JH. Thoracentesis outcomes: a 12-year experience. Thorax 2015; 70(2):127–132. doi:10.1136/thoraxjnl-2014-206114
- Hibbert RM, Atwell TD, Lekah A, et al. Safety of ultrasound-guided thoracentesis in patients with abnormal preprocedural coagulation parameters. Chest 2013; 144(2):456–463. doi:10.1378/chest.12-2374
- Barnes TW, Morgenthaler TI, Olson EJ, Hesley GK, Decker PA, Ryu JH. Sonographically guided thoracentesis and rate of pneumothorax. J Clin Ultrasound 2005; 33(9):442–446. doi:10.1002/jcu.20163
- Gordon CE, Feller-Kopman D, Balk EM, Smetana GW. Pneumothorax following thoracentesis: a systematic review and meta-analysis. Arch Intern Med 2010; 170(4):332–339. doi:10.1001/archinternmed.2009.548
- Heidecker J, Huggins JT, Sahn SA, Doelken P. Pathophysiology of pneumothorax following ultrasound-guided thoracentesis. Chest 2006; 130(4):1173–1184. doi:10.1016/S0012-3692(15)51155-0
- Brandstetter RD, Karetzky M, Rastogi R, Lolis JD. Pneumothorax after thoracentesis in chronic obstructive pulmonary disease. Heart Lung 1994; 23(1):67–70. pmid:8150647
- Doyle JJ, Hnatiuk OW, Torrington KG, Slade AR, Howard RS. Necessity of routine chest roentgenography after thoracentesis. Ann Intern Med 1996; 124(9):816–820. pmid:8610950
- Gervais DA, Petersein A, Lee MJ, Hahn PF, Saini S, Mueller PR. US-guided thoracentesis: requirement for postprocedure chest radiography in patients who receive mechanical ventilation versus patients who breathe spontaneously. Radiology 1997; 204(2):503–506. doi:10.1148/radiology.204.2.9240544
- Capizzi SA, Prakash UB. Chest roentgenography after outpatient thoracentesis. Mayo Clin Proc 1998; 73(10):948–950. doi:10.4065/73.10.948
- Alemán C, Alegre J, Armadans L, et al. The value of chest roentgenography in the diagnosis of pneumothorax after thoracentesis. Am J Med 1999; 107(4):340–343. pmid:10527035
- Lichtenstein D. Lung ultrasound in the critically ill. Curr Opin Crit Care 2014; 20(3):315–322. doi:10.1097/MCC.0000000000000096
- Ault MJ, Rosen BT, Scher J, Feinglass J, Barsuk JH. Thoracentesis outcomes: a 12-year experience. Thorax 2015; 70(2):127–132. doi:10.1136/thoraxjnl-2014-206114
- Hibbert RM, Atwell TD, Lekah A, et al. Safety of ultrasound-guided thoracentesis in patients with abnormal preprocedural coagulation parameters. Chest 2013; 144(2):456–463. doi:10.1378/chest.12-2374
- Barnes TW, Morgenthaler TI, Olson EJ, Hesley GK, Decker PA, Ryu JH. Sonographically guided thoracentesis and rate of pneumothorax. J Clin Ultrasound 2005; 33(9):442–446. doi:10.1002/jcu.20163
- Gordon CE, Feller-Kopman D, Balk EM, Smetana GW. Pneumothorax following thoracentesis: a systematic review and meta-analysis. Arch Intern Med 2010; 170(4):332–339. doi:10.1001/archinternmed.2009.548
- Heidecker J, Huggins JT, Sahn SA, Doelken P. Pathophysiology of pneumothorax following ultrasound-guided thoracentesis. Chest 2006; 130(4):1173–1184. doi:10.1016/S0012-3692(15)51155-0
- Brandstetter RD, Karetzky M, Rastogi R, Lolis JD. Pneumothorax after thoracentesis in chronic obstructive pulmonary disease. Heart Lung 1994; 23(1):67–70. pmid:8150647
- Doyle JJ, Hnatiuk OW, Torrington KG, Slade AR, Howard RS. Necessity of routine chest roentgenography after thoracentesis. Ann Intern Med 1996; 124(9):816–820. pmid:8610950
- Gervais DA, Petersein A, Lee MJ, Hahn PF, Saini S, Mueller PR. US-guided thoracentesis: requirement for postprocedure chest radiography in patients who receive mechanical ventilation versus patients who breathe spontaneously. Radiology 1997; 204(2):503–506. doi:10.1148/radiology.204.2.9240544
- Capizzi SA, Prakash UB. Chest roentgenography after outpatient thoracentesis. Mayo Clin Proc 1998; 73(10):948–950. doi:10.4065/73.10.948
- Alemán C, Alegre J, Armadans L, et al. The value of chest roentgenography in the diagnosis of pneumothorax after thoracentesis. Am J Med 1999; 107(4):340–343. pmid:10527035
- Lichtenstein D. Lung ultrasound in the critically ill. Curr Opin Crit Care 2014; 20(3):315–322. doi:10.1097/MCC.0000000000000096
Significant increase in low-attenuation coronary plaques found in lupus
SAN FRANCISCO – according to an investigation from Johns Hopkins University, Baltimore.
All of the 102 lupus patients in the coronary artery CT angiography study also had positive plaque remodeling, meaning that at least one low-attenuation plaque was growing into the lumen wall, not the lumen itself, which makes them difficult to detect on standard imaging. Low-attenuation plaques were defined in the study as a plaque larger than 1 mm2 with a radiodensity below 30 Hounsfield units.
Low-attenuation plaques are inherently unstable; they’re fatty, necrotic, and have a high risk of rupturing; their presence in the lumen wall is especially worrisome. In the general population, they sometimes regress, scarring down over time and no longer posing a threat. That didn’t happen in the 30 lupus patients who had follow-up CT angiographies, some 9 years after their first.
The team conducted the study to help understand why cardiovascular disease is so common in lupus, and the leading cause of death. Hopkins investigators have shown previously that statins have no effect on the risk or plaque occurrence and progression, and the cardiovascular risk doesn’t always seem to correlate with disease control. For those and other reasons, the current thinking at Hopkins is that cardiovascular disease in lupus is somehow different than in the general population, said George Stojan, MD, an assistant professor of rheumatology at the school and codirector of the Hopkins Lupus Center.
The goal is “to figure out exactly what to look for when we assess the risk; I don’t think we understand that at this point. We assume patients with lupus behave exactly like patients who don’t have lupus, but they obviously don’t. They do not respond to statins. They have a higher risk no matter what you do for them, even when their disease activity is low, and how much plaque they have over time doesn’t really correlate with disease activity,” he said at an international conference on systemic lupus erythematosus.
“Once we understand” the mechanism, “then we can try to [alter] it. Maybe we can look at new drugs, like the PCSK9 inhibitors which have shown a lot of promise in the general population.” At this point, however, “we don’t really know how to intervene,” Dr. Stojan said.
In the meantime, positive remodeling and low-attenuation, noncalcified plaques (LANCPs) might be something to look for when assessing systemic lupus erythematosus cardiovascular risk. “A simple coronary calcium score, something that all doctors do,” is not enough in lupus, nor is simply checking for lumen obstruction. Also, it’s important not to be misled by an overall reduction in noncalcified plaques. “Low-attenuation, noncalcified plaques don’t [regress] over time in lupus, and they are the ones that lead to cardiovascular events,” he said.
The CT angiography findings were compared with findings in 100 healthy controls who had two CT angiograms in a University of California, Los Angeles, cohort. Overall, there was a mean of 458 LANCPs among lupus patients, versus 42 among controls, a more than 900% difference (P less than .001).
Women with lupus aged under 44 years had a mean of 63 LANCPs; none were detected in healthy women under 44 years. Among women aged 45-59 years, there was a mean of 451 LANCPs in the lupus group versus 53 in the control arm. The findings were highly statistically significant, and almost statistically significant for women 60 years or older, 695 lesions among lupus patients versus 22 (P = .0576).
There were only nine men with lupus in the study, but the findings were similar versus male controls.
While mean LANCP volume regressed over time in the control group (mean, –6.90 mm3; P = .0002), a mean regression of –13.56 mm3 in the lupus group was not statistically significant (P = .4570).
Both controls and lupus patients had a positive remodeling index. It progressed in the lupus group over time, and regressed in controls, but the findings were not statistically significant.
“Statins did nothing for the lupus patients. They didn’t affect progress of coronary plaques at all. We still treat patients because theoretically we don’t have anything better, but we know that they don’t really work in this population,” Dr. Stojan said
The work is funded by the National Institutes of Health. Dr. Stojan didn’t report any relevant disclosures.
SOURCE: Stojan G et al. Lupus Sci Med. 2019;6[suppl 1]:A200, Abstract 274.
SAN FRANCISCO – according to an investigation from Johns Hopkins University, Baltimore.
All of the 102 lupus patients in the coronary artery CT angiography study also had positive plaque remodeling, meaning that at least one low-attenuation plaque was growing into the lumen wall, not the lumen itself, which makes them difficult to detect on standard imaging. Low-attenuation plaques were defined in the study as a plaque larger than 1 mm2 with a radiodensity below 30 Hounsfield units.
Low-attenuation plaques are inherently unstable; they’re fatty, necrotic, and have a high risk of rupturing; their presence in the lumen wall is especially worrisome. In the general population, they sometimes regress, scarring down over time and no longer posing a threat. That didn’t happen in the 30 lupus patients who had follow-up CT angiographies, some 9 years after their first.
The team conducted the study to help understand why cardiovascular disease is so common in lupus, and the leading cause of death. Hopkins investigators have shown previously that statins have no effect on the risk or plaque occurrence and progression, and the cardiovascular risk doesn’t always seem to correlate with disease control. For those and other reasons, the current thinking at Hopkins is that cardiovascular disease in lupus is somehow different than in the general population, said George Stojan, MD, an assistant professor of rheumatology at the school and codirector of the Hopkins Lupus Center.
The goal is “to figure out exactly what to look for when we assess the risk; I don’t think we understand that at this point. We assume patients with lupus behave exactly like patients who don’t have lupus, but they obviously don’t. They do not respond to statins. They have a higher risk no matter what you do for them, even when their disease activity is low, and how much plaque they have over time doesn’t really correlate with disease activity,” he said at an international conference on systemic lupus erythematosus.
“Once we understand” the mechanism, “then we can try to [alter] it. Maybe we can look at new drugs, like the PCSK9 inhibitors which have shown a lot of promise in the general population.” At this point, however, “we don’t really know how to intervene,” Dr. Stojan said.
In the meantime, positive remodeling and low-attenuation, noncalcified plaques (LANCPs) might be something to look for when assessing systemic lupus erythematosus cardiovascular risk. “A simple coronary calcium score, something that all doctors do,” is not enough in lupus, nor is simply checking for lumen obstruction. Also, it’s important not to be misled by an overall reduction in noncalcified plaques. “Low-attenuation, noncalcified plaques don’t [regress] over time in lupus, and they are the ones that lead to cardiovascular events,” he said.
The CT angiography findings were compared with findings in 100 healthy controls who had two CT angiograms in a University of California, Los Angeles, cohort. Overall, there was a mean of 458 LANCPs among lupus patients, versus 42 among controls, a more than 900% difference (P less than .001).
Women with lupus aged under 44 years had a mean of 63 LANCPs; none were detected in healthy women under 44 years. Among women aged 45-59 years, there was a mean of 451 LANCPs in the lupus group versus 53 in the control arm. The findings were highly statistically significant, and almost statistically significant for women 60 years or older, 695 lesions among lupus patients versus 22 (P = .0576).
There were only nine men with lupus in the study, but the findings were similar versus male controls.
While mean LANCP volume regressed over time in the control group (mean, –6.90 mm3; P = .0002), a mean regression of –13.56 mm3 in the lupus group was not statistically significant (P = .4570).
Both controls and lupus patients had a positive remodeling index. It progressed in the lupus group over time, and regressed in controls, but the findings were not statistically significant.
“Statins did nothing for the lupus patients. They didn’t affect progress of coronary plaques at all. We still treat patients because theoretically we don’t have anything better, but we know that they don’t really work in this population,” Dr. Stojan said
The work is funded by the National Institutes of Health. Dr. Stojan didn’t report any relevant disclosures.
SOURCE: Stojan G et al. Lupus Sci Med. 2019;6[suppl 1]:A200, Abstract 274.
SAN FRANCISCO – according to an investigation from Johns Hopkins University, Baltimore.
All of the 102 lupus patients in the coronary artery CT angiography study also had positive plaque remodeling, meaning that at least one low-attenuation plaque was growing into the lumen wall, not the lumen itself, which makes them difficult to detect on standard imaging. Low-attenuation plaques were defined in the study as a plaque larger than 1 mm2 with a radiodensity below 30 Hounsfield units.
Low-attenuation plaques are inherently unstable; they’re fatty, necrotic, and have a high risk of rupturing; their presence in the lumen wall is especially worrisome. In the general population, they sometimes regress, scarring down over time and no longer posing a threat. That didn’t happen in the 30 lupus patients who had follow-up CT angiographies, some 9 years after their first.
The team conducted the study to help understand why cardiovascular disease is so common in lupus, and the leading cause of death. Hopkins investigators have shown previously that statins have no effect on the risk or plaque occurrence and progression, and the cardiovascular risk doesn’t always seem to correlate with disease control. For those and other reasons, the current thinking at Hopkins is that cardiovascular disease in lupus is somehow different than in the general population, said George Stojan, MD, an assistant professor of rheumatology at the school and codirector of the Hopkins Lupus Center.
The goal is “to figure out exactly what to look for when we assess the risk; I don’t think we understand that at this point. We assume patients with lupus behave exactly like patients who don’t have lupus, but they obviously don’t. They do not respond to statins. They have a higher risk no matter what you do for them, even when their disease activity is low, and how much plaque they have over time doesn’t really correlate with disease activity,” he said at an international conference on systemic lupus erythematosus.
“Once we understand” the mechanism, “then we can try to [alter] it. Maybe we can look at new drugs, like the PCSK9 inhibitors which have shown a lot of promise in the general population.” At this point, however, “we don’t really know how to intervene,” Dr. Stojan said.
In the meantime, positive remodeling and low-attenuation, noncalcified plaques (LANCPs) might be something to look for when assessing systemic lupus erythematosus cardiovascular risk. “A simple coronary calcium score, something that all doctors do,” is not enough in lupus, nor is simply checking for lumen obstruction. Also, it’s important not to be misled by an overall reduction in noncalcified plaques. “Low-attenuation, noncalcified plaques don’t [regress] over time in lupus, and they are the ones that lead to cardiovascular events,” he said.
The CT angiography findings were compared with findings in 100 healthy controls who had two CT angiograms in a University of California, Los Angeles, cohort. Overall, there was a mean of 458 LANCPs among lupus patients, versus 42 among controls, a more than 900% difference (P less than .001).
Women with lupus aged under 44 years had a mean of 63 LANCPs; none were detected in healthy women under 44 years. Among women aged 45-59 years, there was a mean of 451 LANCPs in the lupus group versus 53 in the control arm. The findings were highly statistically significant, and almost statistically significant for women 60 years or older, 695 lesions among lupus patients versus 22 (P = .0576).
There were only nine men with lupus in the study, but the findings were similar versus male controls.
While mean LANCP volume regressed over time in the control group (mean, –6.90 mm3; P = .0002), a mean regression of –13.56 mm3 in the lupus group was not statistically significant (P = .4570).
Both controls and lupus patients had a positive remodeling index. It progressed in the lupus group over time, and regressed in controls, but the findings were not statistically significant.
“Statins did nothing for the lupus patients. They didn’t affect progress of coronary plaques at all. We still treat patients because theoretically we don’t have anything better, but we know that they don’t really work in this population,” Dr. Stojan said
The work is funded by the National Institutes of Health. Dr. Stojan didn’t report any relevant disclosures.
SOURCE: Stojan G et al. Lupus Sci Med. 2019;6[suppl 1]:A200, Abstract 274.
REPORTING FROM LUPUS 2019
Key clinical point: Positive remodeling and low-attenuation, noncalcified plaques might be something to look for when assessing systemic lupus erythematosus cardiovascular risk.
Major finding: There was a mean of 458 low-attenuation, noncalcified plaques among lupus patients versus 42 among controls, a more than 900% difference (P less than .001)
Study details: Coronary CT angiography in 102 lupus patients and 100 healthy controls
Disclosures: The National Institutes of Health funded the work. The lead investigator didn’t report any relevant disclosures.
Source: Stojan G et al. Lupus Sci Med. 2019;6[suppl 1]:A200, Abstract 274.
Gastric outlet obstruction: A red flag, potentially manageable
A 72-year-old woman presents to the emergency department with progressive nausea and vomiting. One week earlier, she developed early satiety and nausea with vomiting after eating solid food. Three days later her symptoms progressed, and she became unable to take anything by mouth. The patient also experienced a 40-lb weight loss in the previous 3 months. She denies symptoms of abdominal pain, hematemesis, or melena. Her medical history includes cholecystectomy and type 2 diabetes mellitus, diagnosed 1 year ago. She has no family history of gastrointestinal malignancy. She says she smoked 1 pack a day in her 20s. She does not consume alcohol.
On physical examination, she is normotensive with a heart rate of 105 beats per minute. The oral mucosa is dry, and the abdomen is mildly distended and tender to palpation in the epigastrium. Laboratory evaluation reveals hypokalemia and metabolic alkalosis.
Computed tomography (CT) reveals a mass 3 cm by 4 cm in the pancreatic head. The mass has invaded the medial wall of the duodenum, with obstruction of the pancreatic and common bile ducts and extension into and occlusion of the superior mesenteric vein, with soft-tissue expansion around the superior mesenteric artery. CT also reveals retained stomach contents and an air-fluid level consistent with gastric outlet obstruction.
INTRINSIC OR EXTRINSIC BLOCKAGE
Gastric outlet obstruction, also called pyloric obstruction, is caused by intrinsic or extrinsic mechanical blockage of gastric emptying, generally in the distal stomach, pyloric channel, or duodenum, with associated symptoms of nausea, vomiting, abdominal pain, and early satiety. It is encountered in both the clinic and the hospital.
Here, we review the causes, diagnosis, and management of this disorder.
BENIGN AND MALIGNANT CAUSES
In a retrospective study of 76 patients hospitalized with gastric outlet obstruction between 2006 and 2015 at our institution,2 29 cases (38%) were due to malignancy and 47 (62%) were due to benign causes. Pancreatic adenocarcinoma accounted for 13 cases (17%), while gastric adenocarcinoma accounted for 5 cases (7%); less common malignant causes were cholangiocarcinoma, cancer of the ampulla of Vater, duodenal adenocarcinoma, hepatocellular carcinoma, and metastatic disease. Of the benign causes, the most common were peptic ulcer disease (13 cases, 17%) and postoperative strictures or adhesions (11 cases, 14%).
These numbers reflect general trends around the world.
Less gastric cancer, more pancreatic cancer
The last several decades have seen a trend toward more cases due to cancer and fewer due to benign causes.3–14
In earlier studies in both developed and developing countries, gastric adenocarcinoma was the most common malignant cause of gastric outlet obstruction. Since then, it has become less common in Western countries, although it remains more common in Asia and Africa.7–14 This trend likely reflects environmental factors, including decreased prevalence of Helicobacter pylori infection, a major risk factor for gastric cancer, in Western countries.15–17
At the same time, pancreatic cancer is on the rise,16 and up to 20% of patients with pancreatic cancer develop gastric outlet obstruction.18 In a prospective observational study of 108 patients with malignant gastric outlet obstruction undergoing endoscopic stenting, pancreatic cancer was by far the most common malignancy, occurring in 54% of patients, followed by gastric cancer in 13%.19
Less peptic ulcer disease, but still common
Peptic ulcer disease used to account for up to 90% of cases of gastric outlet obstruction, and it is still the most common benign cause.
In 1990, gastric outlet obstruction was estimated to occur in 5% to 10% of all hospital admissions for ulcer-related complications, accounting for 2,000 operations annually.20,21 Gastric outlet obstruction now occurs in fewer than 5% of patients with duodenal ulcer disease and fewer than 2% of patients with gastric ulcer disease.22
Peptic ulcer disease remains an important cause of obstruction in countries with poor access to acid-suppressing drugs.23
Gastric outlet obstruction occurs in both acute and chronic peptic ulcer disease. In acute peptic ulcer disease, tissue inflammation and edema result in mechanical obstruction. Chronic peptic ulcer disease results in tissue scarring and fibrosis with strictures.20
Environmental factors, including improved diet, hygiene, physical activity, and the decreased prevalence of H pylori infection, also contribute to the decreased prevalence of peptic ulcer disease and its complications, including gastric outlet obstruction.3 The continued occurrence of peptic ulcer disease is associated with widespread use of low-dose aspirin and nonsteroidal anti-inflammatory drugs (NSAIDs), the most common causes of peptic ulcer disease in Western countries.24,25
Other nonmalignant causes of gastric outlet obstruction are diverse and less common. They include caustic ingestion, postsurgical strictures, benign tumors of the gastrointestinal tract, Crohn disease, and pancreatic disorders including acute pancreatitis, pancreatic pseudocyst, chronic pancreatitis, and annular pancreas. Intramural duodenal hematoma may cause obstruction after blunt abdominal trauma, endoscopic biopsy, or gastrostomy tube migration, especially in the setting of a bleeding disorder or anticoagulation.26
Tuberculosis should be suspected in countries in which it is common.7 In a prospective study of 64 patients with benign gastric outlet obstruction in India,27 16 (25%) had corrosive injury, 16 (25%) had tuberculosis, and 15 (23%) had peptic ulcer disease. Compared with patients with corrosive injury and peptic ulcer disease, patients with gastroduodenal tuberculosis had the best outcomes with appropriate treatment.
Other reported causes include Bouveret syndrome (an impacted gallstone in the proximal duodenum), phytobezoar, diaphragmatic hernia, gastric volvulus, and Ladd bands (peritoneal bands associated with intestinal malrotation).7,28,29
PRESENTING SYMPTOMS
Symptoms of gastric outlet obstruction include nausea, nonbilious vomiting, epigastric pain, early satiety, abdominal distention, and weight loss.
In our patients, the most common presenting symptoms were nausea and vomiting (80%), followed by abdominal pain (72%); weight loss (15%), abdominal distention (15%), and early satiety (9%) were less common.2
Patients with gastric outlet obstruction secondary to malignancy generally present with a shorter duration of symptoms than those with peptic ulcer disease and are more likely to be older.8,13 Other conditions with an acute onset of symptoms include gastric polyp prolapse, percutaneous endoscopic gastrostomy tube migration, gastric volvulus, and gallstone impaction.
Patients with gastric outlet obstruction associated with peptic ulcer disease generally have a long-standing history of symptoms, including dyspepsia and weight loss over several years.4
SIGNS ON EXAMINATION
On examination, look for signs of chronic gastric obstruction and its consequences, such as malnutrition, cachexia, volume depletion, and dental erosions.
A succussion splash may suggest gastric outlet obstruction. This is elicited by rocking the patient back and forth by the hips or abdomen while listening over the stomach for a splash, which may be heard without a stethoscope. The test is considered positive if present 3 or more hours after drinking fluids and suggests retention of gastric materials.30,31
In thin individuals, chronic gastric outlet obstruction makes the stomach dilate and hypertrophy, which may be evident by a palpably thickened stomach with visible gastric peristalsis.4
Other notable findings on physical examination may include a palpable abdominal mass, epigastric pain, or an abnormality suggestive of metastatic gastric cancer, such as an enlarged left supraclavicular lymph node (Virchow node) or periumbilical lymph node (Sister Mary Joseph nodule). The Virchow node is at the junction of the thoracic duct and the left subclavian vein where the lymphatic circulation from the body drains into the systemic circulation, and it may be the first sign of gastric cancer.32 Sister Mary Joseph nodule (named after a surgical assistant to Dr. William James Mayo) refers to a palpable mass at the umbilicus, generally resulting from metastasis of an abdominal malignancy.33
SIGNS ON FURTHER STUDIES
Laboratory evaluation may show signs of poor oral intake and electrolyte abnormalities secondary to chronic nausea, vomiting, and dehydration, including hypochloremic metabolic alkalosis and hypokalemia.
The underlying cause of gastric outlet obstruction has major implications for treatment and prognosis and cannot be differentiated by clinical presentation alone.1,9 Diagnosis is based on clinical features and radiologic or endoscopic evaluation consistent with gastric outlet obstruction.
Plain radiography may reveal an enlarged gastric bubble, and contrast studies may be useful to determine whether the obstruction is partial or complete, depending on whether the contrast passes into the small bowel.
CT or magnetic resonance imaging may show gastric distention with retained stomach contents, suggesting a gastric, pyloric, duodenal, or pancreatic mass (Figure 1).
Upper endoscopy is often needed to establish the diagnosis and cause. Emptying the stomach with a nasogastric tube is recommended before endoscopy to minimize the risk of aspiration during the procedure, and endotracheal intubation should be considered for airway protection.34 Findings of gastric outlet obstruction on upper endoscopy include retained food and liquid. Endoscopic biopsy is important to differentiate between benign and malignant causes. For patients with malignancy, endoscopic ultrasonography is useful for diagnosis via tissue sampling with fine-needle aspiration and locoregional staging.35
A strategy. Most patients whose clinical presentation suggests gastric outlet obstruction require cross-sectional radiologic imaging, upper endoscopy, or both.36 CT is the preferred imaging study to evaluate for intestinal obstruction.36,37 Patients with suspected complete obstruction or perforation should undergo CT before upper endoscopy. Oral contrast may interfere with endoscopy and should be avoided if endoscopy is planned. Additionally, giving oral contrast may worsen patient discomfort and increase the risk of nausea, vomiting, and aspiration.36,37
Following radiographic evaluation, upper endoscopy can be performed after gastric decompression to identify the location and extent of the obstruction and to potentially provide a definitive diagnosis with biopsy.36
DIFFERENTIATE FROM GASTROPARESIS
Gastroparesis is a chronic neuromuscular disorder characterized by delayed gastric emptying without mechanical obstruction.38 The most common causes are diabetes, surgery, and idiopathy. Other causes include viral infection, connective tissue diseases, ischemia, infiltrative disorders, radiation, neurologic disorders, and paraneoplastic syndromes.39,40
Gastric outlet obstruction and gastroparesis share clinical symptoms including nausea, vomiting, abdominal pain, early satiety, and weight loss and are important to differentiate.36,38 Although abdominal pain may be present in both gastric outlet obstruction and gastroparesis, in gastroparesis it tends not to be the dominant symptom.40
Gastric scintigraphy is most commonly used to objectively quantify delayed gastric emptying.39 Upper endoscopy is imperative to exclude mechanical obstruction.39
MANAGEMENT
Initially, patients with signs and symptoms of gastric outlet obstruction should be given:
- Nothing by mouth (NPO)
- Intravenous fluids to correct volume depletion and electrolyte abnormalities
- A nasogastric tube for gastric decompression and symptom relief if symptoms persist despite being NPO
- A parenteral proton pump inhibitor, regardless of the cause of obstruction, to decrease gastric secretions41
- Medications for pain and nausea, if needed.
Definitive treatment of gastric outlet obstruction depends on the underlying cause, whether benign or malignant.
Management of benign gastric outlet obstruction
Symptoms of gastric outlet obstruction resolve spontaneously in about half of cases caused by acute peptic ulcer disease, as acute inflammation resolves.9,22
Endoscopic dilation is an important option in patients with benign gastric outlet obstruction, including peptic ulcer disease. Peptic ulcer disease-induced gastric outlet obstruction can be safely treated with endoscopic balloon dilation. This treatment almost always relieves symptoms immediately; however, the long-term response has varied from 16% to 100%, and patients may require more than 1 dilation procedure.25,42,43 The need for 2 or more dilation procedures may predict need for surgery.44 Gastric outlet obstruction after caustic ingestion or endoscopic submucosal dissection may also respond to endoscopic balloon dilation.36
Eradication of H pylori may be effective and lead to complete resolution of symptoms in patients with gastric outlet obstruction due to this infection.45–47
NSAIDs should be discontinued in patients with peptic ulcer disease and gastric outlet obstruction. These drugs damage the gastrointestinal mucosa by inhibiting cyclo-oxygenase (COX) enzymes and decreasing synthesis of prostaglandins, which are important for mucosal defense.48 Patients may be unaware of NSAIDs contained in over-the-counter medications and may have difficulty discontinuing NSAIDs taken for pain.49
These drugs are an important cause of refractory peptic ulcer disease and can be detected by platelet COX activity testing, although this test is not widely available. In a study of patients with peptic ulcer disease without definite NSAID use or H pylori infection, up to one-third had evidence of surreptitious NSAID use as detected by platelet COX activity testing.50 In another study,51 platelet COX activity testing discovered over 20% more aspirin users than clinical history alone.
Surgery for patients with benign gastric outlet obstruction is used only when medical management and endoscopic dilation fail. Ideally, surgery should relieve the obstruction and target the underlying cause, such as peptic ulcer disease. Laparoscopic surgery is generally preferred to open surgery because patients can resume oral intake sooner, have a shorter hospital stay, and have less intraoperative blood loss.52 The simplest surgical procedure to relieve obstruction is laparoscopic gastrojejunostomy.
Patients with gastric outlet obstruction and peptic ulcer disease warrant laparoscopic vagotomy and antrectomy or distal gastrectomy. This removes the obstruction and the stimulus for gastric secretion.53 An alternative is vagotomy with a drainage procedure (pyloroplasty or gastrojejunostomy), which has a similar postoperative course and reduction in gastric acid secretion compared with antrectomy or distal gastrectomy.53,54
Daily proton pump inhibitors can be used for patients with benign gastric outlet obstruction not associated with peptic ulcer disease or risk factors; for such cases, vagotomy is not required.
Management of malignant gastric outlet obstruction
Patients with malignant gastric outlet obstruction may have intractable nausea and abdominal pain secondary to retention of gastric contents. The major goal of therapy is to improve symptoms and restore tolerance of an oral diet. The short-term prognosis of malignant gastric outlet obstruction is poor, with a median survival of 3 to 4 months, as these patients often have unresectable disease.55
Surgical bypass used to be the standard of care for palliation of malignant gastric obstruction, but that was before endoscopic stenting was developed.
Endoscopic stenting allows patients to resume oral intake and get out of the hospital sooner with fewer complications than with open surgical bypass. It may be a more appropriate option for palliation of symptoms in patients with malignant obstruction who have a poor prognosis and prefer a less invasive intervention.55,56
Endoscopic duodenal stenting of malignant gastric outlet obstruction has a success rate of greater than 90%, and most patients can tolerate a mechanical soft diet afterward.34 The procedure is usually performed with a 9-cm or 12-cm self-expanding duodenal stent, 22 mm in diameter, placed over a guide wire under endoscopic and fluoroscopic guidance (Figure 2). The stent is placed by removing the outer catheter, with distal-to-proximal stent deployment.
Patients who also have biliary obstruction may require biliary stent placement, which is generally performed before duodenal stenting. For patients with an endoscopic stent who develop biliary obstruction, endoscopic retrograde cholangiopancreatography can be attempted with placement of a biliary stent; however, these patients may require biliary drain placement by percutaneous transhepatic cholangiography or by endoscopic ultrasonographically guided transduodenal or transgastric biliary drainage.
From 20% to 30% of patients require repeated endoscopic stent placement, although most patients die within several months after stenting.34 Surgical options for patients who do not respond to endoscopic stenting include open or laparoscopic gastrojejunostomy.55
Laparoscopic gastrojejunostomy may provide better long-term outcomes than duodenal stenting for patients with malignant gastric outlet obstruction and a life expectancy longer than a few months.
A 2017 retrospective study of 155 patients with gastric outlet obstruction secondary to unresectable gastric cancer suggested that those who underwent laparoscopic gastrojejunostomy had better oral intake, better tolerance of chemotherapy, and longer overall survival than those who underwent duodenal stenting. Postsurgical complications were more common in the laparoscopic gastrojejunostomy group (16%) than in the duodenal stenting group (0%).57
In most of the studies comparing endoscopic stenting with surgery, the surgery was open gastrojejunostomy; there are limited data directly comparing stenting with laparoscopic gastrojejunostomy.55 Endoscopic stenting is estimated to be significantly less costly than surgery, with a median cost of $12,000 less than gastrojejunostomy.58 As an alternative to enteral stenting and surgical gastrojejunostomy, ultrasonography-guided endoscopic gastrojejunostomy or gastroenterostomy with placement of a lumen-apposing metal stent is emerging as a third treatment option and is under active investigation.59
Patients with malignancy that is potentially curable by resection should undergo surgical evaluation before consideration of endoscopic stenting. For patients who are not candidates for surgery or endoscopic stenting, a percutaneous gastrostomy tube can be considered for gastric decompression and symptom relief.
CASE CONCLUDED
The patient underwent esophagogastroduodenoscopy with endoscopic ultrasonography for evaluation of her pancreatic mass. Before the procedure, she was intubated to minimize the risk of aspiration due to persistent nausea and retained gastric contents. A large submucosal mass was found in the duodenal bulb. Endoscopic ultrasonography showed a mass within the pancreatic head with pancreatic duct obstruction. Fine-needle aspiration biopsy was performed, and pathology study revealed pancreatic adenocarcinoma. The patient underwent stenting with a 22-mm by 12-cm WallFlex stent (Boston Scientific), which led to resolution of nausea and advancement to a mechanical soft diet on hospital discharge.
She was scheduled for follow-up in the outpatient clinic for treatment of pancreatic cancer.
- Johnson CD. Gastric outlet obstruction malignant until proved otherwise. Am J Gastroenterol 1995; 90(10):1740. pmid:7572886
- Koop AH, Palmer WC, Mareth K, Burton MC, Bowman A, Stancampiano F. Tu1335 - Pancreatic cancer most common cause of malignant gastric outlet obstruction at a tertiary referral center: a 10 year retrospective study [abstract]. Gastroenterology 2018; 154(6, suppl 1):S-1343.
- Hall R, Royston C, Bardhan KD. The scars of time: the disappearance of peptic ulcer-related pyloric stenosis through the 20th century. J R Coll Physicians Edinb 2014; 44(3):201–208. doi:10.4997/JRCPE.2014.303
- Kreel L, Ellis H. Pyloric stenosis in adults: a clinical and radiological study of 100 consecutive patients. Gut 1965; 6(3):253–261. pmid:18668780
- Shone DN, Nikoomanesh P, Smith-Meek MM, Bender JS. Malignancy is the most common cause of gastric outlet obstruction in the era of H2 blockers. Am J Gastroenterol 1995; 90(10):1769–1770. pmid:7572891
- Ellis H. The diagnosis of benign and malignant pyloric obstruction. Clin Oncol 1976; 2(1):11–15. pmid:1277618
- Samad A, Khanzada TW, Shoukat I. Gastric outlet obstruction: change in etiology. Pak J Surg 2007; 23(1):29–32.
- Chowdhury A, Dhali GK, Banerjee PK. Etiology of gastric outlet obstruction. Am J Gastroenterol 1996; 91(8):1679. pmid:8759707
- Johnson CD, Ellis H. Gastric outlet obstruction now predicts malignancy. Br J Surg 1990; 77(9):1023–1024. pmid:2207566
- Misra SP, Dwivedi M, Misra V. Malignancy is the most common cause of gastric outlet obstruction even in a developing country. Endoscopy 1998; 30(5):484–486. doi:10.1055/s-2007-1001313
- Essoun SD, Dakubo JCB. Update of aetiological patterns of adult gastric outlet obstruction in Accra, Ghana. Int J Clin Med 2014; 5(17):1059–1064. doi:10.4236/ijcm.2014.517136
- Jaka H, Mchembe MD, Rambau PF, Chalya PL. Gastric outlet obstruction at Bugando Medical Centre in Northwestern Tanzania: a prospective review of 184 cases. BMC Surg 2013; 13:41. doi:10.1186/1471-2482-13-41
- Sukumar V, Ravindran C, Prasad RV. Demographic and etiological patterns of gastric outlet obstruction in Kerala, South India. N Am J Med Sci 2015; 7(9):403–406. doi:10.4103/1947-2714.166220
- Yoursef M, Mirza MR, Khan S. Gastric outlet obstruction. Pak J Surg 2005; 10(4):48–50.
- Ferlay J, Soerjomataram I, Dikshit R, et al. Cancer incidence and mortality worldwide: sources, methods and major patterns in GLOBOCAN 2012. Int J Cancer 2015; 136(5):E359–E386. doi:10.1002/ijc.29210
- Parkin DM, Stjernsward J, Muir CS. Estimates of the worldwide frequency of twelve major cancers. Bull World Health Organ 1984; 62(2):163–182. pmid:6610488
- Karimi P, Islami F, Anandasabapathy S, Freedman ND, Kamangar F. Gastric cancer: descriptive epidemiology, risk factors, screening, and prevention. Cancer Epidemiol Biomarkers Prev 2014; 23(5):700–713. doi:10.1158/1055-9965.EPI-13-1057
- Jeurnink SM, Steyerberg EW, van Hooft JE, et al; Dutch SUSTENT Study Group. Surgical gastrojejunostomy or endoscopic stent placement for the palliation of malignant gastric outlet obstruction (SUSTENT) study): a multicenter randomized trial. Gastrointest Endosc 2010; 71(3):490–499. doi:10.1016/j.gie.2009.09.042
- Tringali A, Didden P, Repici A, et al. Endoscopic treatment of malignant gastric and duodenal strictures: a prospective, multicenter study. Gastrointest Endosc 2014; 79(1):66–75. doi:10.1016/j.gie.2013.06.032
- Malfertheiner P, Chan FK, McColl KE. Peptic ulcer disease. Lancet 2009; 374(9699):1449–1461. doi:10.1016/S0140-6736(09)60938-7
- Gibson JB, Behrman SW, Fabian TC, Britt LG. Gastric outlet obstruction resulting from peptic ulcer disease requiring surgical intervention is infrequently associated with Helicobacter pylori infection. J Am Coll Surg 2000; 191(1):32–37. pmid:10898181
- Kochhar R, Kochhar S. Endoscopic balloon dilation for benign gastric outlet obstruction in adults. World J Gastrointest Endosc 2010; 2(1):29–35. doi:10.4253/wjge.v2.i1.29
- Kotisso R. Gastric outlet obstruction in Northwestern Ethiopia. East Cent Afr J Surg 2000; 5(2):25-29.
- Hamzaoui L, Bouassida M, Ben Mansour I, et al. Balloon dilatation in patients with gastric outlet obstruction related to peptic ulcer disease. Arab J Gastroenterol 2015; 16(3–4):121–124. doi:10.1016/j.ajg.2015.07.004
- Najm WI. Peptic ulcer disease. Prim Care 2011; 38(3):383–394. doi:10.1016/j.pop.2011.05.001
- Veloso N, Amaro P, Ferreira M, Romaozinho JM, Sofia C. Acute pancreatitis associated with a nontraumatic, intramural duodenal hematoma. Endoscopy 2013; 45(suppl 2):E51–E52. doi:10.1055/s-0032-1325969
- Maharshi S, Puri AS, Sachdeva S, Kumar A, Dalal A, Gupta M. Aetiological spectrum of benign gastric outlet obstruction in India: new trends. Trop Doct 2016; 46(4):186–191. doi:10.1177/0049475515626032
- Sala MA, Ligabo AN, de Arruda MC, Indiani JM, Nacif MS. Intestinal malrotation associated with duodenal obstruction secondary to Ladd’s bands. Radiol Bras 2016; 49(4):271–272. doi:10.1590/0100-3984.2015.0106
- Alibegovic E, Kurtcehajic A, Hujdurovic A, Mujagic S, Alibegovic J, Kurtcehajic D. Bouveret syndrome or gallstone ileus. Am J Med 2018; 131(4):e175. doi:10.1016/j.amjmed.2017.10.044
- Lau JY, Chung SC, Sung JJ, et al. Through-the-scope balloon dilation for pyloric stenosis: long-term results. Gastrointest Endosc 1996; 43(2 Pt 1):98–101. pmid:8635729
- Ray K, Snowden C, Khatri K, McFall M. Gastric outlet obstruction from a caecal volvulus, herniated through epiploic foramen: a case report. BMJ Case Rep 2009; pii:bcr05.2009.1880. doi:10.1136/bcr.05.2009.1880
- Baumgart DC, Fischer A. Virchow’s node. Lancet 2007; 370(9598):1568. doi:10.1016/S0140-6736(07)61661-4
- Dar IH, Kamili MA, Dar SH, Kuchaai FA. Sister Mary Joseph nodule—a case report with review of literature. J Res Med Sci 2009; 14(6):385–387. pmid:21772912
- Tang SJ. Endoscopic stent placement for gastric outlet obstruction. Video Journal and Encyclopedia of GI Endoscopy 2013; 1(1):133–136.
- Valero M, Robles-Medranda C. Endoscopic ultrasound in oncology: an update of clinical applications in the gastrointestinal tract. World J Gastrointest Endosc 2017; 9(6):243–254.
- ASGE Standards of Practice Committee; Fukami N, Anderson MA, Khan K, et al. The role of endoscopy in gastroduodenal obstruction and gastroparesis. Gastrointest Endosc 2011; 74(1):13–21. doi:10.1016/j.gie.2010.12.003
- Ros PR, Huprich JE. ACR appropriateness criteria on suspected small-bowel obstruction. J Am Coll Radiol 2006; 3(11):838–841. doi:10.1016/j.jacr.2006.09.018
- Pasricha PJ, Parkman HP. Gastroparesis: definitions and diagnosis. Gastroenterol Clin North Am 2015; 44(1):1–7. doi:10.1016/j.gtc.2014.11.001
- Stein B, Everhart KK, Lacy BE. Gastroparesis: a review of current diagnosis and treatment options. J Clin Gastroenterol 2015; 49(7):550–558. doi:10.1097/MCG.0000000000000320
- Camilleri M, Parkman HP, Shafi MA, Abell TL, Gerson L; American College of Gastroenterology. Clinical guideline: management of gastroparesis. Am J Gastroenterol 2013; 108(1):18–37.
- Gursoy O, Memis D, Sut N. Effect of proton pump inhibitors on gastric juice volume, gastric pH and gastric intramucosal pH in critically ill patients: a randomized, double-blind, placebo-controlled study. Clin Drug Investig 2008; 28(12):777–782. doi:10.2165/0044011-200828120-00005
- Kuwada SK, Alexander GL. Long-term outcome of endoscopic dilation of nonmalignant pyloric stenosis. Gastrointest Endosc 1995; 41(1):15–17. pmid:7698619
- Kochhar R, Sethy PK, Nagi B, Wig JD. Endoscopic balloon dilatation of benign gastric outlet obstruction. J Gastroenterol Hepatol 2004; 19(4):418–422. pmid:15012779
- Perng CL, Lin HJ, Lo WC, Lai CR, Guo WS, Lee SD. Characteristics of patients with benign gastric outlet obstruction requiring surgery after endoscopic balloon dilation. Am J Gastroenterol 1996; 91(5):987–990. pmid:8633593
- Taskin V, Gurer I, Ozyilkan E, Sare M, Hilmioglu F. Effect of Helicobacter pylori eradication on peptic ulcer disease complicated with outlet obstruction. Helicobacter 2000; 5(1):38–40. pmid:10672050
- de Boer WA, Driessen WM. Resolution of gastric outlet obstruction after eradication of Helicobacter pylori. J Clin Gastroenterol 1995; 21(4):329–330. pmid:8583113
- Tursi A, Cammarota G, Papa A, Montalto M, Fedeli G, Gasbarrini G. Helicobacter pylori eradication helps resolve pyloric and duodenal stenosis. J Clin Gastroenterol 1996; 23(2):157–158. pmid:8877648
- Schmassmann A. Mechanisms of ulcer healing and effects of nonsteroidal anti-inflammatory drugs. Am J Med 1998; 104(3A):43S–51S; discussion 79S–80S. pmid:9572320
- Kim HU. Diagnostic and treatment approaches for refractory peptic ulcers. Clin Endosc 2015; 48(4):285–290. doi:10.5946/ce.2015.48.4.285
- Ong TZ, Hawkey CJ, Ho KY. Nonsteroidal anti-inflammatory drug use is a significant cause of peptic ulcer disease in a tertiary hospital in Singapore: a prospective study. J Clin Gastroenterol 2006; 40(9):795–800. doi:10.1097/01.mcg.0000225610.41105.7f
- Lanas A, Sekar MC, Hirschowitz BI. Objective evidence of aspirin use in both ulcer and nonulcer upper and lower gastrointestinal bleeding. Gastroenterology 1992; 103(3):862–869. pmid:1499936
- Zhang LP, Tabrizian P, Nguyen S, Telem D, Divino C. Laparoscopic gastrojejunostomy for the treatment of gastric outlet obstruction. JSLS 2011; 15(2):169–173. doi:10.4293/108680811X13022985132074
- Lagoo J, Pappas TN, Perez A. A relic or still relevant: the narrowing role for vagotomy in the treatment of peptic ulcer disease. Am J Surg 2014; 207(1):120–126. doi:10.1016/j.amjsurg.2013.02.012
- Csendes A, Maluenda F, Braghetto I, Schutte H, Burdiles P, Diaz JC. Prospective randomized study comparing three surgical techniques for the treatment of gastric outlet obstruction secondary to duodenal ulcer. Am J Surg 1993; 166(1):45–49. pmid:8101050
- Ly J, O’Grady G, Mittal A, Plank L, Windsor JA. A systematic review of methods to palliate malignant gastric outlet obstruction. Surg Endosc 2010; 24(2):290–297. doi:10.1007/s00464-009-0577-1
- Goldberg EM. Palliative treatment of gastric outlet obstruction in terminal patients: SEMS. Stent every malignant stricture! Gastrointest Endosc 2014; 79(1):76–78. doi:10.1016/j.gie.2013.07.056
- Min SH, Son SY, Jung DH, et al. Laparoscopic gastrojejunostomy versus duodenal stenting in unresectable gastric cancer with gastric outlet obstruction. Ann Surg Treat Res 2017; 93(3):130–136. doi:10.4174/astr.2017.93.3.130
- Roy A, Kim M, Christein J, Varadarajulu S. Stenting versus gastrojejunostomy for management of malignant gastric outlet obstruction: comparison of clinical outcomes and costs. Surg Endosc 2012; 26(11):3114–119. doi:10.1007/s00464-012-2301-9
- Amin S, Sethi A. Endoscopic ultrasound-guided gastrojejunostomy. Gastrointest Endosc Clin N Am 2017; 27(4):707–713. doi:10.1016/j.giec.2017.06.009
A 72-year-old woman presents to the emergency department with progressive nausea and vomiting. One week earlier, she developed early satiety and nausea with vomiting after eating solid food. Three days later her symptoms progressed, and she became unable to take anything by mouth. The patient also experienced a 40-lb weight loss in the previous 3 months. She denies symptoms of abdominal pain, hematemesis, or melena. Her medical history includes cholecystectomy and type 2 diabetes mellitus, diagnosed 1 year ago. She has no family history of gastrointestinal malignancy. She says she smoked 1 pack a day in her 20s. She does not consume alcohol.
On physical examination, she is normotensive with a heart rate of 105 beats per minute. The oral mucosa is dry, and the abdomen is mildly distended and tender to palpation in the epigastrium. Laboratory evaluation reveals hypokalemia and metabolic alkalosis.
Computed tomography (CT) reveals a mass 3 cm by 4 cm in the pancreatic head. The mass has invaded the medial wall of the duodenum, with obstruction of the pancreatic and common bile ducts and extension into and occlusion of the superior mesenteric vein, with soft-tissue expansion around the superior mesenteric artery. CT also reveals retained stomach contents and an air-fluid level consistent with gastric outlet obstruction.
INTRINSIC OR EXTRINSIC BLOCKAGE
Gastric outlet obstruction, also called pyloric obstruction, is caused by intrinsic or extrinsic mechanical blockage of gastric emptying, generally in the distal stomach, pyloric channel, or duodenum, with associated symptoms of nausea, vomiting, abdominal pain, and early satiety. It is encountered in both the clinic and the hospital.
Here, we review the causes, diagnosis, and management of this disorder.
BENIGN AND MALIGNANT CAUSES
In a retrospective study of 76 patients hospitalized with gastric outlet obstruction between 2006 and 2015 at our institution,2 29 cases (38%) were due to malignancy and 47 (62%) were due to benign causes. Pancreatic adenocarcinoma accounted for 13 cases (17%), while gastric adenocarcinoma accounted for 5 cases (7%); less common malignant causes were cholangiocarcinoma, cancer of the ampulla of Vater, duodenal adenocarcinoma, hepatocellular carcinoma, and metastatic disease. Of the benign causes, the most common were peptic ulcer disease (13 cases, 17%) and postoperative strictures or adhesions (11 cases, 14%).
These numbers reflect general trends around the world.
Less gastric cancer, more pancreatic cancer
The last several decades have seen a trend toward more cases due to cancer and fewer due to benign causes.3–14
In earlier studies in both developed and developing countries, gastric adenocarcinoma was the most common malignant cause of gastric outlet obstruction. Since then, it has become less common in Western countries, although it remains more common in Asia and Africa.7–14 This trend likely reflects environmental factors, including decreased prevalence of Helicobacter pylori infection, a major risk factor for gastric cancer, in Western countries.15–17
At the same time, pancreatic cancer is on the rise,16 and up to 20% of patients with pancreatic cancer develop gastric outlet obstruction.18 In a prospective observational study of 108 patients with malignant gastric outlet obstruction undergoing endoscopic stenting, pancreatic cancer was by far the most common malignancy, occurring in 54% of patients, followed by gastric cancer in 13%.19
Less peptic ulcer disease, but still common
Peptic ulcer disease used to account for up to 90% of cases of gastric outlet obstruction, and it is still the most common benign cause.
In 1990, gastric outlet obstruction was estimated to occur in 5% to 10% of all hospital admissions for ulcer-related complications, accounting for 2,000 operations annually.20,21 Gastric outlet obstruction now occurs in fewer than 5% of patients with duodenal ulcer disease and fewer than 2% of patients with gastric ulcer disease.22
Peptic ulcer disease remains an important cause of obstruction in countries with poor access to acid-suppressing drugs.23
Gastric outlet obstruction occurs in both acute and chronic peptic ulcer disease. In acute peptic ulcer disease, tissue inflammation and edema result in mechanical obstruction. Chronic peptic ulcer disease results in tissue scarring and fibrosis with strictures.20
Environmental factors, including improved diet, hygiene, physical activity, and the decreased prevalence of H pylori infection, also contribute to the decreased prevalence of peptic ulcer disease and its complications, including gastric outlet obstruction.3 The continued occurrence of peptic ulcer disease is associated with widespread use of low-dose aspirin and nonsteroidal anti-inflammatory drugs (NSAIDs), the most common causes of peptic ulcer disease in Western countries.24,25
Other nonmalignant causes of gastric outlet obstruction are diverse and less common. They include caustic ingestion, postsurgical strictures, benign tumors of the gastrointestinal tract, Crohn disease, and pancreatic disorders including acute pancreatitis, pancreatic pseudocyst, chronic pancreatitis, and annular pancreas. Intramural duodenal hematoma may cause obstruction after blunt abdominal trauma, endoscopic biopsy, or gastrostomy tube migration, especially in the setting of a bleeding disorder or anticoagulation.26
Tuberculosis should be suspected in countries in which it is common.7 In a prospective study of 64 patients with benign gastric outlet obstruction in India,27 16 (25%) had corrosive injury, 16 (25%) had tuberculosis, and 15 (23%) had peptic ulcer disease. Compared with patients with corrosive injury and peptic ulcer disease, patients with gastroduodenal tuberculosis had the best outcomes with appropriate treatment.
Other reported causes include Bouveret syndrome (an impacted gallstone in the proximal duodenum), phytobezoar, diaphragmatic hernia, gastric volvulus, and Ladd bands (peritoneal bands associated with intestinal malrotation).7,28,29
PRESENTING SYMPTOMS
Symptoms of gastric outlet obstruction include nausea, nonbilious vomiting, epigastric pain, early satiety, abdominal distention, and weight loss.
In our patients, the most common presenting symptoms were nausea and vomiting (80%), followed by abdominal pain (72%); weight loss (15%), abdominal distention (15%), and early satiety (9%) were less common.2
Patients with gastric outlet obstruction secondary to malignancy generally present with a shorter duration of symptoms than those with peptic ulcer disease and are more likely to be older.8,13 Other conditions with an acute onset of symptoms include gastric polyp prolapse, percutaneous endoscopic gastrostomy tube migration, gastric volvulus, and gallstone impaction.
Patients with gastric outlet obstruction associated with peptic ulcer disease generally have a long-standing history of symptoms, including dyspepsia and weight loss over several years.4
SIGNS ON EXAMINATION
On examination, look for signs of chronic gastric obstruction and its consequences, such as malnutrition, cachexia, volume depletion, and dental erosions.
A succussion splash may suggest gastric outlet obstruction. This is elicited by rocking the patient back and forth by the hips or abdomen while listening over the stomach for a splash, which may be heard without a stethoscope. The test is considered positive if present 3 or more hours after drinking fluids and suggests retention of gastric materials.30,31
In thin individuals, chronic gastric outlet obstruction makes the stomach dilate and hypertrophy, which may be evident by a palpably thickened stomach with visible gastric peristalsis.4
Other notable findings on physical examination may include a palpable abdominal mass, epigastric pain, or an abnormality suggestive of metastatic gastric cancer, such as an enlarged left supraclavicular lymph node (Virchow node) or periumbilical lymph node (Sister Mary Joseph nodule). The Virchow node is at the junction of the thoracic duct and the left subclavian vein where the lymphatic circulation from the body drains into the systemic circulation, and it may be the first sign of gastric cancer.32 Sister Mary Joseph nodule (named after a surgical assistant to Dr. William James Mayo) refers to a palpable mass at the umbilicus, generally resulting from metastasis of an abdominal malignancy.33
SIGNS ON FURTHER STUDIES
Laboratory evaluation may show signs of poor oral intake and electrolyte abnormalities secondary to chronic nausea, vomiting, and dehydration, including hypochloremic metabolic alkalosis and hypokalemia.
The underlying cause of gastric outlet obstruction has major implications for treatment and prognosis and cannot be differentiated by clinical presentation alone.1,9 Diagnosis is based on clinical features and radiologic or endoscopic evaluation consistent with gastric outlet obstruction.
Plain radiography may reveal an enlarged gastric bubble, and contrast studies may be useful to determine whether the obstruction is partial or complete, depending on whether the contrast passes into the small bowel.
CT or magnetic resonance imaging may show gastric distention with retained stomach contents, suggesting a gastric, pyloric, duodenal, or pancreatic mass (Figure 1).
Upper endoscopy is often needed to establish the diagnosis and cause. Emptying the stomach with a nasogastric tube is recommended before endoscopy to minimize the risk of aspiration during the procedure, and endotracheal intubation should be considered for airway protection.34 Findings of gastric outlet obstruction on upper endoscopy include retained food and liquid. Endoscopic biopsy is important to differentiate between benign and malignant causes. For patients with malignancy, endoscopic ultrasonography is useful for diagnosis via tissue sampling with fine-needle aspiration and locoregional staging.35
A strategy. Most patients whose clinical presentation suggests gastric outlet obstruction require cross-sectional radiologic imaging, upper endoscopy, or both.36 CT is the preferred imaging study to evaluate for intestinal obstruction.36,37 Patients with suspected complete obstruction or perforation should undergo CT before upper endoscopy. Oral contrast may interfere with endoscopy and should be avoided if endoscopy is planned. Additionally, giving oral contrast may worsen patient discomfort and increase the risk of nausea, vomiting, and aspiration.36,37
Following radiographic evaluation, upper endoscopy can be performed after gastric decompression to identify the location and extent of the obstruction and to potentially provide a definitive diagnosis with biopsy.36
DIFFERENTIATE FROM GASTROPARESIS
Gastroparesis is a chronic neuromuscular disorder characterized by delayed gastric emptying without mechanical obstruction.38 The most common causes are diabetes, surgery, and idiopathy. Other causes include viral infection, connective tissue diseases, ischemia, infiltrative disorders, radiation, neurologic disorders, and paraneoplastic syndromes.39,40
Gastric outlet obstruction and gastroparesis share clinical symptoms including nausea, vomiting, abdominal pain, early satiety, and weight loss and are important to differentiate.36,38 Although abdominal pain may be present in both gastric outlet obstruction and gastroparesis, in gastroparesis it tends not to be the dominant symptom.40
Gastric scintigraphy is most commonly used to objectively quantify delayed gastric emptying.39 Upper endoscopy is imperative to exclude mechanical obstruction.39
MANAGEMENT
Initially, patients with signs and symptoms of gastric outlet obstruction should be given:
- Nothing by mouth (NPO)
- Intravenous fluids to correct volume depletion and electrolyte abnormalities
- A nasogastric tube for gastric decompression and symptom relief if symptoms persist despite being NPO
- A parenteral proton pump inhibitor, regardless of the cause of obstruction, to decrease gastric secretions41
- Medications for pain and nausea, if needed.
Definitive treatment of gastric outlet obstruction depends on the underlying cause, whether benign or malignant.
Management of benign gastric outlet obstruction
Symptoms of gastric outlet obstruction resolve spontaneously in about half of cases caused by acute peptic ulcer disease, as acute inflammation resolves.9,22
Endoscopic dilation is an important option in patients with benign gastric outlet obstruction, including peptic ulcer disease. Peptic ulcer disease-induced gastric outlet obstruction can be safely treated with endoscopic balloon dilation. This treatment almost always relieves symptoms immediately; however, the long-term response has varied from 16% to 100%, and patients may require more than 1 dilation procedure.25,42,43 The need for 2 or more dilation procedures may predict need for surgery.44 Gastric outlet obstruction after caustic ingestion or endoscopic submucosal dissection may also respond to endoscopic balloon dilation.36
Eradication of H pylori may be effective and lead to complete resolution of symptoms in patients with gastric outlet obstruction due to this infection.45–47
NSAIDs should be discontinued in patients with peptic ulcer disease and gastric outlet obstruction. These drugs damage the gastrointestinal mucosa by inhibiting cyclo-oxygenase (COX) enzymes and decreasing synthesis of prostaglandins, which are important for mucosal defense.48 Patients may be unaware of NSAIDs contained in over-the-counter medications and may have difficulty discontinuing NSAIDs taken for pain.49
These drugs are an important cause of refractory peptic ulcer disease and can be detected by platelet COX activity testing, although this test is not widely available. In a study of patients with peptic ulcer disease without definite NSAID use or H pylori infection, up to one-third had evidence of surreptitious NSAID use as detected by platelet COX activity testing.50 In another study,51 platelet COX activity testing discovered over 20% more aspirin users than clinical history alone.
Surgery for patients with benign gastric outlet obstruction is used only when medical management and endoscopic dilation fail. Ideally, surgery should relieve the obstruction and target the underlying cause, such as peptic ulcer disease. Laparoscopic surgery is generally preferred to open surgery because patients can resume oral intake sooner, have a shorter hospital stay, and have less intraoperative blood loss.52 The simplest surgical procedure to relieve obstruction is laparoscopic gastrojejunostomy.
Patients with gastric outlet obstruction and peptic ulcer disease warrant laparoscopic vagotomy and antrectomy or distal gastrectomy. This removes the obstruction and the stimulus for gastric secretion.53 An alternative is vagotomy with a drainage procedure (pyloroplasty or gastrojejunostomy), which has a similar postoperative course and reduction in gastric acid secretion compared with antrectomy or distal gastrectomy.53,54
Daily proton pump inhibitors can be used for patients with benign gastric outlet obstruction not associated with peptic ulcer disease or risk factors; for such cases, vagotomy is not required.
Management of malignant gastric outlet obstruction
Patients with malignant gastric outlet obstruction may have intractable nausea and abdominal pain secondary to retention of gastric contents. The major goal of therapy is to improve symptoms and restore tolerance of an oral diet. The short-term prognosis of malignant gastric outlet obstruction is poor, with a median survival of 3 to 4 months, as these patients often have unresectable disease.55
Surgical bypass used to be the standard of care for palliation of malignant gastric obstruction, but that was before endoscopic stenting was developed.
Endoscopic stenting allows patients to resume oral intake and get out of the hospital sooner with fewer complications than with open surgical bypass. It may be a more appropriate option for palliation of symptoms in patients with malignant obstruction who have a poor prognosis and prefer a less invasive intervention.55,56
Endoscopic duodenal stenting of malignant gastric outlet obstruction has a success rate of greater than 90%, and most patients can tolerate a mechanical soft diet afterward.34 The procedure is usually performed with a 9-cm or 12-cm self-expanding duodenal stent, 22 mm in diameter, placed over a guide wire under endoscopic and fluoroscopic guidance (Figure 2). The stent is placed by removing the outer catheter, with distal-to-proximal stent deployment.
Patients who also have biliary obstruction may require biliary stent placement, which is generally performed before duodenal stenting. For patients with an endoscopic stent who develop biliary obstruction, endoscopic retrograde cholangiopancreatography can be attempted with placement of a biliary stent; however, these patients may require biliary drain placement by percutaneous transhepatic cholangiography or by endoscopic ultrasonographically guided transduodenal or transgastric biliary drainage.
From 20% to 30% of patients require repeated endoscopic stent placement, although most patients die within several months after stenting.34 Surgical options for patients who do not respond to endoscopic stenting include open or laparoscopic gastrojejunostomy.55
Laparoscopic gastrojejunostomy may provide better long-term outcomes than duodenal stenting for patients with malignant gastric outlet obstruction and a life expectancy longer than a few months.
A 2017 retrospective study of 155 patients with gastric outlet obstruction secondary to unresectable gastric cancer suggested that those who underwent laparoscopic gastrojejunostomy had better oral intake, better tolerance of chemotherapy, and longer overall survival than those who underwent duodenal stenting. Postsurgical complications were more common in the laparoscopic gastrojejunostomy group (16%) than in the duodenal stenting group (0%).57
In most of the studies comparing endoscopic stenting with surgery, the surgery was open gastrojejunostomy; there are limited data directly comparing stenting with laparoscopic gastrojejunostomy.55 Endoscopic stenting is estimated to be significantly less costly than surgery, with a median cost of $12,000 less than gastrojejunostomy.58 As an alternative to enteral stenting and surgical gastrojejunostomy, ultrasonography-guided endoscopic gastrojejunostomy or gastroenterostomy with placement of a lumen-apposing metal stent is emerging as a third treatment option and is under active investigation.59
Patients with malignancy that is potentially curable by resection should undergo surgical evaluation before consideration of endoscopic stenting. For patients who are not candidates for surgery or endoscopic stenting, a percutaneous gastrostomy tube can be considered for gastric decompression and symptom relief.
CASE CONCLUDED
The patient underwent esophagogastroduodenoscopy with endoscopic ultrasonography for evaluation of her pancreatic mass. Before the procedure, she was intubated to minimize the risk of aspiration due to persistent nausea and retained gastric contents. A large submucosal mass was found in the duodenal bulb. Endoscopic ultrasonography showed a mass within the pancreatic head with pancreatic duct obstruction. Fine-needle aspiration biopsy was performed, and pathology study revealed pancreatic adenocarcinoma. The patient underwent stenting with a 22-mm by 12-cm WallFlex stent (Boston Scientific), which led to resolution of nausea and advancement to a mechanical soft diet on hospital discharge.
She was scheduled for follow-up in the outpatient clinic for treatment of pancreatic cancer.
A 72-year-old woman presents to the emergency department with progressive nausea and vomiting. One week earlier, she developed early satiety and nausea with vomiting after eating solid food. Three days later her symptoms progressed, and she became unable to take anything by mouth. The patient also experienced a 40-lb weight loss in the previous 3 months. She denies symptoms of abdominal pain, hematemesis, or melena. Her medical history includes cholecystectomy and type 2 diabetes mellitus, diagnosed 1 year ago. She has no family history of gastrointestinal malignancy. She says she smoked 1 pack a day in her 20s. She does not consume alcohol.
On physical examination, she is normotensive with a heart rate of 105 beats per minute. The oral mucosa is dry, and the abdomen is mildly distended and tender to palpation in the epigastrium. Laboratory evaluation reveals hypokalemia and metabolic alkalosis.
Computed tomography (CT) reveals a mass 3 cm by 4 cm in the pancreatic head. The mass has invaded the medial wall of the duodenum, with obstruction of the pancreatic and common bile ducts and extension into and occlusion of the superior mesenteric vein, with soft-tissue expansion around the superior mesenteric artery. CT also reveals retained stomach contents and an air-fluid level consistent with gastric outlet obstruction.
INTRINSIC OR EXTRINSIC BLOCKAGE
Gastric outlet obstruction, also called pyloric obstruction, is caused by intrinsic or extrinsic mechanical blockage of gastric emptying, generally in the distal stomach, pyloric channel, or duodenum, with associated symptoms of nausea, vomiting, abdominal pain, and early satiety. It is encountered in both the clinic and the hospital.
Here, we review the causes, diagnosis, and management of this disorder.
BENIGN AND MALIGNANT CAUSES
In a retrospective study of 76 patients hospitalized with gastric outlet obstruction between 2006 and 2015 at our institution,2 29 cases (38%) were due to malignancy and 47 (62%) were due to benign causes. Pancreatic adenocarcinoma accounted for 13 cases (17%), while gastric adenocarcinoma accounted for 5 cases (7%); less common malignant causes were cholangiocarcinoma, cancer of the ampulla of Vater, duodenal adenocarcinoma, hepatocellular carcinoma, and metastatic disease. Of the benign causes, the most common were peptic ulcer disease (13 cases, 17%) and postoperative strictures or adhesions (11 cases, 14%).
These numbers reflect general trends around the world.
Less gastric cancer, more pancreatic cancer
The last several decades have seen a trend toward more cases due to cancer and fewer due to benign causes.3–14
In earlier studies in both developed and developing countries, gastric adenocarcinoma was the most common malignant cause of gastric outlet obstruction. Since then, it has become less common in Western countries, although it remains more common in Asia and Africa.7–14 This trend likely reflects environmental factors, including decreased prevalence of Helicobacter pylori infection, a major risk factor for gastric cancer, in Western countries.15–17
At the same time, pancreatic cancer is on the rise,16 and up to 20% of patients with pancreatic cancer develop gastric outlet obstruction.18 In a prospective observational study of 108 patients with malignant gastric outlet obstruction undergoing endoscopic stenting, pancreatic cancer was by far the most common malignancy, occurring in 54% of patients, followed by gastric cancer in 13%.19
Less peptic ulcer disease, but still common
Peptic ulcer disease used to account for up to 90% of cases of gastric outlet obstruction, and it is still the most common benign cause.
In 1990, gastric outlet obstruction was estimated to occur in 5% to 10% of all hospital admissions for ulcer-related complications, accounting for 2,000 operations annually.20,21 Gastric outlet obstruction now occurs in fewer than 5% of patients with duodenal ulcer disease and fewer than 2% of patients with gastric ulcer disease.22
Peptic ulcer disease remains an important cause of obstruction in countries with poor access to acid-suppressing drugs.23
Gastric outlet obstruction occurs in both acute and chronic peptic ulcer disease. In acute peptic ulcer disease, tissue inflammation and edema result in mechanical obstruction. Chronic peptic ulcer disease results in tissue scarring and fibrosis with strictures.20
Environmental factors, including improved diet, hygiene, physical activity, and the decreased prevalence of H pylori infection, also contribute to the decreased prevalence of peptic ulcer disease and its complications, including gastric outlet obstruction.3 The continued occurrence of peptic ulcer disease is associated with widespread use of low-dose aspirin and nonsteroidal anti-inflammatory drugs (NSAIDs), the most common causes of peptic ulcer disease in Western countries.24,25
Other nonmalignant causes of gastric outlet obstruction are diverse and less common. They include caustic ingestion, postsurgical strictures, benign tumors of the gastrointestinal tract, Crohn disease, and pancreatic disorders including acute pancreatitis, pancreatic pseudocyst, chronic pancreatitis, and annular pancreas. Intramural duodenal hematoma may cause obstruction after blunt abdominal trauma, endoscopic biopsy, or gastrostomy tube migration, especially in the setting of a bleeding disorder or anticoagulation.26
Tuberculosis should be suspected in countries in which it is common.7 In a prospective study of 64 patients with benign gastric outlet obstruction in India,27 16 (25%) had corrosive injury, 16 (25%) had tuberculosis, and 15 (23%) had peptic ulcer disease. Compared with patients with corrosive injury and peptic ulcer disease, patients with gastroduodenal tuberculosis had the best outcomes with appropriate treatment.
Other reported causes include Bouveret syndrome (an impacted gallstone in the proximal duodenum), phytobezoar, diaphragmatic hernia, gastric volvulus, and Ladd bands (peritoneal bands associated with intestinal malrotation).7,28,29
PRESENTING SYMPTOMS
Symptoms of gastric outlet obstruction include nausea, nonbilious vomiting, epigastric pain, early satiety, abdominal distention, and weight loss.
In our patients, the most common presenting symptoms were nausea and vomiting (80%), followed by abdominal pain (72%); weight loss (15%), abdominal distention (15%), and early satiety (9%) were less common.2
Patients with gastric outlet obstruction secondary to malignancy generally present with a shorter duration of symptoms than those with peptic ulcer disease and are more likely to be older.8,13 Other conditions with an acute onset of symptoms include gastric polyp prolapse, percutaneous endoscopic gastrostomy tube migration, gastric volvulus, and gallstone impaction.
Patients with gastric outlet obstruction associated with peptic ulcer disease generally have a long-standing history of symptoms, including dyspepsia and weight loss over several years.4
SIGNS ON EXAMINATION
On examination, look for signs of chronic gastric obstruction and its consequences, such as malnutrition, cachexia, volume depletion, and dental erosions.
A succussion splash may suggest gastric outlet obstruction. This is elicited by rocking the patient back and forth by the hips or abdomen while listening over the stomach for a splash, which may be heard without a stethoscope. The test is considered positive if present 3 or more hours after drinking fluids and suggests retention of gastric materials.30,31
In thin individuals, chronic gastric outlet obstruction makes the stomach dilate and hypertrophy, which may be evident by a palpably thickened stomach with visible gastric peristalsis.4
Other notable findings on physical examination may include a palpable abdominal mass, epigastric pain, or an abnormality suggestive of metastatic gastric cancer, such as an enlarged left supraclavicular lymph node (Virchow node) or periumbilical lymph node (Sister Mary Joseph nodule). The Virchow node is at the junction of the thoracic duct and the left subclavian vein where the lymphatic circulation from the body drains into the systemic circulation, and it may be the first sign of gastric cancer.32 Sister Mary Joseph nodule (named after a surgical assistant to Dr. William James Mayo) refers to a palpable mass at the umbilicus, generally resulting from metastasis of an abdominal malignancy.33
SIGNS ON FURTHER STUDIES
Laboratory evaluation may show signs of poor oral intake and electrolyte abnormalities secondary to chronic nausea, vomiting, and dehydration, including hypochloremic metabolic alkalosis and hypokalemia.
The underlying cause of gastric outlet obstruction has major implications for treatment and prognosis and cannot be differentiated by clinical presentation alone.1,9 Diagnosis is based on clinical features and radiologic or endoscopic evaluation consistent with gastric outlet obstruction.
Plain radiography may reveal an enlarged gastric bubble, and contrast studies may be useful to determine whether the obstruction is partial or complete, depending on whether the contrast passes into the small bowel.
CT or magnetic resonance imaging may show gastric distention with retained stomach contents, suggesting a gastric, pyloric, duodenal, or pancreatic mass (Figure 1).
Upper endoscopy is often needed to establish the diagnosis and cause. Emptying the stomach with a nasogastric tube is recommended before endoscopy to minimize the risk of aspiration during the procedure, and endotracheal intubation should be considered for airway protection.34 Findings of gastric outlet obstruction on upper endoscopy include retained food and liquid. Endoscopic biopsy is important to differentiate between benign and malignant causes. For patients with malignancy, endoscopic ultrasonography is useful for diagnosis via tissue sampling with fine-needle aspiration and locoregional staging.35
A strategy. Most patients whose clinical presentation suggests gastric outlet obstruction require cross-sectional radiologic imaging, upper endoscopy, or both.36 CT is the preferred imaging study to evaluate for intestinal obstruction.36,37 Patients with suspected complete obstruction or perforation should undergo CT before upper endoscopy. Oral contrast may interfere with endoscopy and should be avoided if endoscopy is planned. Additionally, giving oral contrast may worsen patient discomfort and increase the risk of nausea, vomiting, and aspiration.36,37
Following radiographic evaluation, upper endoscopy can be performed after gastric decompression to identify the location and extent of the obstruction and to potentially provide a definitive diagnosis with biopsy.36
DIFFERENTIATE FROM GASTROPARESIS
Gastroparesis is a chronic neuromuscular disorder characterized by delayed gastric emptying without mechanical obstruction.38 The most common causes are diabetes, surgery, and idiopathy. Other causes include viral infection, connective tissue diseases, ischemia, infiltrative disorders, radiation, neurologic disorders, and paraneoplastic syndromes.39,40
Gastric outlet obstruction and gastroparesis share clinical symptoms including nausea, vomiting, abdominal pain, early satiety, and weight loss and are important to differentiate.36,38 Although abdominal pain may be present in both gastric outlet obstruction and gastroparesis, in gastroparesis it tends not to be the dominant symptom.40
Gastric scintigraphy is most commonly used to objectively quantify delayed gastric emptying.39 Upper endoscopy is imperative to exclude mechanical obstruction.39
MANAGEMENT
Initially, patients with signs and symptoms of gastric outlet obstruction should be given:
- Nothing by mouth (NPO)
- Intravenous fluids to correct volume depletion and electrolyte abnormalities
- A nasogastric tube for gastric decompression and symptom relief if symptoms persist despite being NPO
- A parenteral proton pump inhibitor, regardless of the cause of obstruction, to decrease gastric secretions41
- Medications for pain and nausea, if needed.
Definitive treatment of gastric outlet obstruction depends on the underlying cause, whether benign or malignant.
Management of benign gastric outlet obstruction
Symptoms of gastric outlet obstruction resolve spontaneously in about half of cases caused by acute peptic ulcer disease, as acute inflammation resolves.9,22
Endoscopic dilation is an important option in patients with benign gastric outlet obstruction, including peptic ulcer disease. Peptic ulcer disease-induced gastric outlet obstruction can be safely treated with endoscopic balloon dilation. This treatment almost always relieves symptoms immediately; however, the long-term response has varied from 16% to 100%, and patients may require more than 1 dilation procedure.25,42,43 The need for 2 or more dilation procedures may predict need for surgery.44 Gastric outlet obstruction after caustic ingestion or endoscopic submucosal dissection may also respond to endoscopic balloon dilation.36
Eradication of H pylori may be effective and lead to complete resolution of symptoms in patients with gastric outlet obstruction due to this infection.45–47
NSAIDs should be discontinued in patients with peptic ulcer disease and gastric outlet obstruction. These drugs damage the gastrointestinal mucosa by inhibiting cyclo-oxygenase (COX) enzymes and decreasing synthesis of prostaglandins, which are important for mucosal defense.48 Patients may be unaware of NSAIDs contained in over-the-counter medications and may have difficulty discontinuing NSAIDs taken for pain.49
These drugs are an important cause of refractory peptic ulcer disease and can be detected by platelet COX activity testing, although this test is not widely available. In a study of patients with peptic ulcer disease without definite NSAID use or H pylori infection, up to one-third had evidence of surreptitious NSAID use as detected by platelet COX activity testing.50 In another study,51 platelet COX activity testing discovered over 20% more aspirin users than clinical history alone.
Surgery for patients with benign gastric outlet obstruction is used only when medical management and endoscopic dilation fail. Ideally, surgery should relieve the obstruction and target the underlying cause, such as peptic ulcer disease. Laparoscopic surgery is generally preferred to open surgery because patients can resume oral intake sooner, have a shorter hospital stay, and have less intraoperative blood loss.52 The simplest surgical procedure to relieve obstruction is laparoscopic gastrojejunostomy.
Patients with gastric outlet obstruction and peptic ulcer disease warrant laparoscopic vagotomy and antrectomy or distal gastrectomy. This removes the obstruction and the stimulus for gastric secretion.53 An alternative is vagotomy with a drainage procedure (pyloroplasty or gastrojejunostomy), which has a similar postoperative course and reduction in gastric acid secretion compared with antrectomy or distal gastrectomy.53,54
Daily proton pump inhibitors can be used for patients with benign gastric outlet obstruction not associated with peptic ulcer disease or risk factors; for such cases, vagotomy is not required.
Management of malignant gastric outlet obstruction
Patients with malignant gastric outlet obstruction may have intractable nausea and abdominal pain secondary to retention of gastric contents. The major goal of therapy is to improve symptoms and restore tolerance of an oral diet. The short-term prognosis of malignant gastric outlet obstruction is poor, with a median survival of 3 to 4 months, as these patients often have unresectable disease.55
Surgical bypass used to be the standard of care for palliation of malignant gastric obstruction, but that was before endoscopic stenting was developed.
Endoscopic stenting allows patients to resume oral intake and get out of the hospital sooner with fewer complications than with open surgical bypass. It may be a more appropriate option for palliation of symptoms in patients with malignant obstruction who have a poor prognosis and prefer a less invasive intervention.55,56
Endoscopic duodenal stenting of malignant gastric outlet obstruction has a success rate of greater than 90%, and most patients can tolerate a mechanical soft diet afterward.34 The procedure is usually performed with a 9-cm or 12-cm self-expanding duodenal stent, 22 mm in diameter, placed over a guide wire under endoscopic and fluoroscopic guidance (Figure 2). The stent is placed by removing the outer catheter, with distal-to-proximal stent deployment.
Patients who also have biliary obstruction may require biliary stent placement, which is generally performed before duodenal stenting. For patients with an endoscopic stent who develop biliary obstruction, endoscopic retrograde cholangiopancreatography can be attempted with placement of a biliary stent; however, these patients may require biliary drain placement by percutaneous transhepatic cholangiography or by endoscopic ultrasonographically guided transduodenal or transgastric biliary drainage.
From 20% to 30% of patients require repeated endoscopic stent placement, although most patients die within several months after stenting.34 Surgical options for patients who do not respond to endoscopic stenting include open or laparoscopic gastrojejunostomy.55
Laparoscopic gastrojejunostomy may provide better long-term outcomes than duodenal stenting for patients with malignant gastric outlet obstruction and a life expectancy longer than a few months.
A 2017 retrospective study of 155 patients with gastric outlet obstruction secondary to unresectable gastric cancer suggested that those who underwent laparoscopic gastrojejunostomy had better oral intake, better tolerance of chemotherapy, and longer overall survival than those who underwent duodenal stenting. Postsurgical complications were more common in the laparoscopic gastrojejunostomy group (16%) than in the duodenal stenting group (0%).57
In most of the studies comparing endoscopic stenting with surgery, the surgery was open gastrojejunostomy; there are limited data directly comparing stenting with laparoscopic gastrojejunostomy.55 Endoscopic stenting is estimated to be significantly less costly than surgery, with a median cost of $12,000 less than gastrojejunostomy.58 As an alternative to enteral stenting and surgical gastrojejunostomy, ultrasonography-guided endoscopic gastrojejunostomy or gastroenterostomy with placement of a lumen-apposing metal stent is emerging as a third treatment option and is under active investigation.59
Patients with malignancy that is potentially curable by resection should undergo surgical evaluation before consideration of endoscopic stenting. For patients who are not candidates for surgery or endoscopic stenting, a percutaneous gastrostomy tube can be considered for gastric decompression and symptom relief.
CASE CONCLUDED
The patient underwent esophagogastroduodenoscopy with endoscopic ultrasonography for evaluation of her pancreatic mass. Before the procedure, she was intubated to minimize the risk of aspiration due to persistent nausea and retained gastric contents. A large submucosal mass was found in the duodenal bulb. Endoscopic ultrasonography showed a mass within the pancreatic head with pancreatic duct obstruction. Fine-needle aspiration biopsy was performed, and pathology study revealed pancreatic adenocarcinoma. The patient underwent stenting with a 22-mm by 12-cm WallFlex stent (Boston Scientific), which led to resolution of nausea and advancement to a mechanical soft diet on hospital discharge.
She was scheduled for follow-up in the outpatient clinic for treatment of pancreatic cancer.
- Johnson CD. Gastric outlet obstruction malignant until proved otherwise. Am J Gastroenterol 1995; 90(10):1740. pmid:7572886
- Koop AH, Palmer WC, Mareth K, Burton MC, Bowman A, Stancampiano F. Tu1335 - Pancreatic cancer most common cause of malignant gastric outlet obstruction at a tertiary referral center: a 10 year retrospective study [abstract]. Gastroenterology 2018; 154(6, suppl 1):S-1343.
- Hall R, Royston C, Bardhan KD. The scars of time: the disappearance of peptic ulcer-related pyloric stenosis through the 20th century. J R Coll Physicians Edinb 2014; 44(3):201–208. doi:10.4997/JRCPE.2014.303
- Kreel L, Ellis H. Pyloric stenosis in adults: a clinical and radiological study of 100 consecutive patients. Gut 1965; 6(3):253–261. pmid:18668780
- Shone DN, Nikoomanesh P, Smith-Meek MM, Bender JS. Malignancy is the most common cause of gastric outlet obstruction in the era of H2 blockers. Am J Gastroenterol 1995; 90(10):1769–1770. pmid:7572891
- Ellis H. The diagnosis of benign and malignant pyloric obstruction. Clin Oncol 1976; 2(1):11–15. pmid:1277618
- Samad A, Khanzada TW, Shoukat I. Gastric outlet obstruction: change in etiology. Pak J Surg 2007; 23(1):29–32.
- Chowdhury A, Dhali GK, Banerjee PK. Etiology of gastric outlet obstruction. Am J Gastroenterol 1996; 91(8):1679. pmid:8759707
- Johnson CD, Ellis H. Gastric outlet obstruction now predicts malignancy. Br J Surg 1990; 77(9):1023–1024. pmid:2207566
- Misra SP, Dwivedi M, Misra V. Malignancy is the most common cause of gastric outlet obstruction even in a developing country. Endoscopy 1998; 30(5):484–486. doi:10.1055/s-2007-1001313
- Essoun SD, Dakubo JCB. Update of aetiological patterns of adult gastric outlet obstruction in Accra, Ghana. Int J Clin Med 2014; 5(17):1059–1064. doi:10.4236/ijcm.2014.517136
- Jaka H, Mchembe MD, Rambau PF, Chalya PL. Gastric outlet obstruction at Bugando Medical Centre in Northwestern Tanzania: a prospective review of 184 cases. BMC Surg 2013; 13:41. doi:10.1186/1471-2482-13-41
- Sukumar V, Ravindran C, Prasad RV. Demographic and etiological patterns of gastric outlet obstruction in Kerala, South India. N Am J Med Sci 2015; 7(9):403–406. doi:10.4103/1947-2714.166220
- Yoursef M, Mirza MR, Khan S. Gastric outlet obstruction. Pak J Surg 2005; 10(4):48–50.
- Ferlay J, Soerjomataram I, Dikshit R, et al. Cancer incidence and mortality worldwide: sources, methods and major patterns in GLOBOCAN 2012. Int J Cancer 2015; 136(5):E359–E386. doi:10.1002/ijc.29210
- Parkin DM, Stjernsward J, Muir CS. Estimates of the worldwide frequency of twelve major cancers. Bull World Health Organ 1984; 62(2):163–182. pmid:6610488
- Karimi P, Islami F, Anandasabapathy S, Freedman ND, Kamangar F. Gastric cancer: descriptive epidemiology, risk factors, screening, and prevention. Cancer Epidemiol Biomarkers Prev 2014; 23(5):700–713. doi:10.1158/1055-9965.EPI-13-1057
- Jeurnink SM, Steyerberg EW, van Hooft JE, et al; Dutch SUSTENT Study Group. Surgical gastrojejunostomy or endoscopic stent placement for the palliation of malignant gastric outlet obstruction (SUSTENT) study): a multicenter randomized trial. Gastrointest Endosc 2010; 71(3):490–499. doi:10.1016/j.gie.2009.09.042
- Tringali A, Didden P, Repici A, et al. Endoscopic treatment of malignant gastric and duodenal strictures: a prospective, multicenter study. Gastrointest Endosc 2014; 79(1):66–75. doi:10.1016/j.gie.2013.06.032
- Malfertheiner P, Chan FK, McColl KE. Peptic ulcer disease. Lancet 2009; 374(9699):1449–1461. doi:10.1016/S0140-6736(09)60938-7
- Gibson JB, Behrman SW, Fabian TC, Britt LG. Gastric outlet obstruction resulting from peptic ulcer disease requiring surgical intervention is infrequently associated with Helicobacter pylori infection. J Am Coll Surg 2000; 191(1):32–37. pmid:10898181
- Kochhar R, Kochhar S. Endoscopic balloon dilation for benign gastric outlet obstruction in adults. World J Gastrointest Endosc 2010; 2(1):29–35. doi:10.4253/wjge.v2.i1.29
- Kotisso R. Gastric outlet obstruction in Northwestern Ethiopia. East Cent Afr J Surg 2000; 5(2):25-29.
- Hamzaoui L, Bouassida M, Ben Mansour I, et al. Balloon dilatation in patients with gastric outlet obstruction related to peptic ulcer disease. Arab J Gastroenterol 2015; 16(3–4):121–124. doi:10.1016/j.ajg.2015.07.004
- Najm WI. Peptic ulcer disease. Prim Care 2011; 38(3):383–394. doi:10.1016/j.pop.2011.05.001
- Veloso N, Amaro P, Ferreira M, Romaozinho JM, Sofia C. Acute pancreatitis associated with a nontraumatic, intramural duodenal hematoma. Endoscopy 2013; 45(suppl 2):E51–E52. doi:10.1055/s-0032-1325969
- Maharshi S, Puri AS, Sachdeva S, Kumar A, Dalal A, Gupta M. Aetiological spectrum of benign gastric outlet obstruction in India: new trends. Trop Doct 2016; 46(4):186–191. doi:10.1177/0049475515626032
- Sala MA, Ligabo AN, de Arruda MC, Indiani JM, Nacif MS. Intestinal malrotation associated with duodenal obstruction secondary to Ladd’s bands. Radiol Bras 2016; 49(4):271–272. doi:10.1590/0100-3984.2015.0106
- Alibegovic E, Kurtcehajic A, Hujdurovic A, Mujagic S, Alibegovic J, Kurtcehajic D. Bouveret syndrome or gallstone ileus. Am J Med 2018; 131(4):e175. doi:10.1016/j.amjmed.2017.10.044
- Lau JY, Chung SC, Sung JJ, et al. Through-the-scope balloon dilation for pyloric stenosis: long-term results. Gastrointest Endosc 1996; 43(2 Pt 1):98–101. pmid:8635729
- Ray K, Snowden C, Khatri K, McFall M. Gastric outlet obstruction from a caecal volvulus, herniated through epiploic foramen: a case report. BMJ Case Rep 2009; pii:bcr05.2009.1880. doi:10.1136/bcr.05.2009.1880
- Baumgart DC, Fischer A. Virchow’s node. Lancet 2007; 370(9598):1568. doi:10.1016/S0140-6736(07)61661-4
- Dar IH, Kamili MA, Dar SH, Kuchaai FA. Sister Mary Joseph nodule—a case report with review of literature. J Res Med Sci 2009; 14(6):385–387. pmid:21772912
- Tang SJ. Endoscopic stent placement for gastric outlet obstruction. Video Journal and Encyclopedia of GI Endoscopy 2013; 1(1):133–136.
- Valero M, Robles-Medranda C. Endoscopic ultrasound in oncology: an update of clinical applications in the gastrointestinal tract. World J Gastrointest Endosc 2017; 9(6):243–254.
- ASGE Standards of Practice Committee; Fukami N, Anderson MA, Khan K, et al. The role of endoscopy in gastroduodenal obstruction and gastroparesis. Gastrointest Endosc 2011; 74(1):13–21. doi:10.1016/j.gie.2010.12.003
- Ros PR, Huprich JE. ACR appropriateness criteria on suspected small-bowel obstruction. J Am Coll Radiol 2006; 3(11):838–841. doi:10.1016/j.jacr.2006.09.018
- Pasricha PJ, Parkman HP. Gastroparesis: definitions and diagnosis. Gastroenterol Clin North Am 2015; 44(1):1–7. doi:10.1016/j.gtc.2014.11.001
- Stein B, Everhart KK, Lacy BE. Gastroparesis: a review of current diagnosis and treatment options. J Clin Gastroenterol 2015; 49(7):550–558. doi:10.1097/MCG.0000000000000320
- Camilleri M, Parkman HP, Shafi MA, Abell TL, Gerson L; American College of Gastroenterology. Clinical guideline: management of gastroparesis. Am J Gastroenterol 2013; 108(1):18–37.
- Gursoy O, Memis D, Sut N. Effect of proton pump inhibitors on gastric juice volume, gastric pH and gastric intramucosal pH in critically ill patients: a randomized, double-blind, placebo-controlled study. Clin Drug Investig 2008; 28(12):777–782. doi:10.2165/0044011-200828120-00005
- Kuwada SK, Alexander GL. Long-term outcome of endoscopic dilation of nonmalignant pyloric stenosis. Gastrointest Endosc 1995; 41(1):15–17. pmid:7698619
- Kochhar R, Sethy PK, Nagi B, Wig JD. Endoscopic balloon dilatation of benign gastric outlet obstruction. J Gastroenterol Hepatol 2004; 19(4):418–422. pmid:15012779
- Perng CL, Lin HJ, Lo WC, Lai CR, Guo WS, Lee SD. Characteristics of patients with benign gastric outlet obstruction requiring surgery after endoscopic balloon dilation. Am J Gastroenterol 1996; 91(5):987–990. pmid:8633593
- Taskin V, Gurer I, Ozyilkan E, Sare M, Hilmioglu F. Effect of Helicobacter pylori eradication on peptic ulcer disease complicated with outlet obstruction. Helicobacter 2000; 5(1):38–40. pmid:10672050
- de Boer WA, Driessen WM. Resolution of gastric outlet obstruction after eradication of Helicobacter pylori. J Clin Gastroenterol 1995; 21(4):329–330. pmid:8583113
- Tursi A, Cammarota G, Papa A, Montalto M, Fedeli G, Gasbarrini G. Helicobacter pylori eradication helps resolve pyloric and duodenal stenosis. J Clin Gastroenterol 1996; 23(2):157–158. pmid:8877648
- Schmassmann A. Mechanisms of ulcer healing and effects of nonsteroidal anti-inflammatory drugs. Am J Med 1998; 104(3A):43S–51S; discussion 79S–80S. pmid:9572320
- Kim HU. Diagnostic and treatment approaches for refractory peptic ulcers. Clin Endosc 2015; 48(4):285–290. doi:10.5946/ce.2015.48.4.285
- Ong TZ, Hawkey CJ, Ho KY. Nonsteroidal anti-inflammatory drug use is a significant cause of peptic ulcer disease in a tertiary hospital in Singapore: a prospective study. J Clin Gastroenterol 2006; 40(9):795–800. doi:10.1097/01.mcg.0000225610.41105.7f
- Lanas A, Sekar MC, Hirschowitz BI. Objective evidence of aspirin use in both ulcer and nonulcer upper and lower gastrointestinal bleeding. Gastroenterology 1992; 103(3):862–869. pmid:1499936
- Zhang LP, Tabrizian P, Nguyen S, Telem D, Divino C. Laparoscopic gastrojejunostomy for the treatment of gastric outlet obstruction. JSLS 2011; 15(2):169–173. doi:10.4293/108680811X13022985132074
- Lagoo J, Pappas TN, Perez A. A relic or still relevant: the narrowing role for vagotomy in the treatment of peptic ulcer disease. Am J Surg 2014; 207(1):120–126. doi:10.1016/j.amjsurg.2013.02.012
- Csendes A, Maluenda F, Braghetto I, Schutte H, Burdiles P, Diaz JC. Prospective randomized study comparing three surgical techniques for the treatment of gastric outlet obstruction secondary to duodenal ulcer. Am J Surg 1993; 166(1):45–49. pmid:8101050
- Ly J, O’Grady G, Mittal A, Plank L, Windsor JA. A systematic review of methods to palliate malignant gastric outlet obstruction. Surg Endosc 2010; 24(2):290–297. doi:10.1007/s00464-009-0577-1
- Goldberg EM. Palliative treatment of gastric outlet obstruction in terminal patients: SEMS. Stent every malignant stricture! Gastrointest Endosc 2014; 79(1):76–78. doi:10.1016/j.gie.2013.07.056
- Min SH, Son SY, Jung DH, et al. Laparoscopic gastrojejunostomy versus duodenal stenting in unresectable gastric cancer with gastric outlet obstruction. Ann Surg Treat Res 2017; 93(3):130–136. doi:10.4174/astr.2017.93.3.130
- Roy A, Kim M, Christein J, Varadarajulu S. Stenting versus gastrojejunostomy for management of malignant gastric outlet obstruction: comparison of clinical outcomes and costs. Surg Endosc 2012; 26(11):3114–119. doi:10.1007/s00464-012-2301-9
- Amin S, Sethi A. Endoscopic ultrasound-guided gastrojejunostomy. Gastrointest Endosc Clin N Am 2017; 27(4):707–713. doi:10.1016/j.giec.2017.06.009
- Johnson CD. Gastric outlet obstruction malignant until proved otherwise. Am J Gastroenterol 1995; 90(10):1740. pmid:7572886
- Koop AH, Palmer WC, Mareth K, Burton MC, Bowman A, Stancampiano F. Tu1335 - Pancreatic cancer most common cause of malignant gastric outlet obstruction at a tertiary referral center: a 10 year retrospective study [abstract]. Gastroenterology 2018; 154(6, suppl 1):S-1343.
- Hall R, Royston C, Bardhan KD. The scars of time: the disappearance of peptic ulcer-related pyloric stenosis through the 20th century. J R Coll Physicians Edinb 2014; 44(3):201–208. doi:10.4997/JRCPE.2014.303
- Kreel L, Ellis H. Pyloric stenosis in adults: a clinical and radiological study of 100 consecutive patients. Gut 1965; 6(3):253–261. pmid:18668780
- Shone DN, Nikoomanesh P, Smith-Meek MM, Bender JS. Malignancy is the most common cause of gastric outlet obstruction in the era of H2 blockers. Am J Gastroenterol 1995; 90(10):1769–1770. pmid:7572891
- Ellis H. The diagnosis of benign and malignant pyloric obstruction. Clin Oncol 1976; 2(1):11–15. pmid:1277618
- Samad A, Khanzada TW, Shoukat I. Gastric outlet obstruction: change in etiology. Pak J Surg 2007; 23(1):29–32.
- Chowdhury A, Dhali GK, Banerjee PK. Etiology of gastric outlet obstruction. Am J Gastroenterol 1996; 91(8):1679. pmid:8759707
- Johnson CD, Ellis H. Gastric outlet obstruction now predicts malignancy. Br J Surg 1990; 77(9):1023–1024. pmid:2207566
- Misra SP, Dwivedi M, Misra V. Malignancy is the most common cause of gastric outlet obstruction even in a developing country. Endoscopy 1998; 30(5):484–486. doi:10.1055/s-2007-1001313
- Essoun SD, Dakubo JCB. Update of aetiological patterns of adult gastric outlet obstruction in Accra, Ghana. Int J Clin Med 2014; 5(17):1059–1064. doi:10.4236/ijcm.2014.517136
- Jaka H, Mchembe MD, Rambau PF, Chalya PL. Gastric outlet obstruction at Bugando Medical Centre in Northwestern Tanzania: a prospective review of 184 cases. BMC Surg 2013; 13:41. doi:10.1186/1471-2482-13-41
- Sukumar V, Ravindran C, Prasad RV. Demographic and etiological patterns of gastric outlet obstruction in Kerala, South India. N Am J Med Sci 2015; 7(9):403–406. doi:10.4103/1947-2714.166220
- Yoursef M, Mirza MR, Khan S. Gastric outlet obstruction. Pak J Surg 2005; 10(4):48–50.
- Ferlay J, Soerjomataram I, Dikshit R, et al. Cancer incidence and mortality worldwide: sources, methods and major patterns in GLOBOCAN 2012. Int J Cancer 2015; 136(5):E359–E386. doi:10.1002/ijc.29210
- Parkin DM, Stjernsward J, Muir CS. Estimates of the worldwide frequency of twelve major cancers. Bull World Health Organ 1984; 62(2):163–182. pmid:6610488
- Karimi P, Islami F, Anandasabapathy S, Freedman ND, Kamangar F. Gastric cancer: descriptive epidemiology, risk factors, screening, and prevention. Cancer Epidemiol Biomarkers Prev 2014; 23(5):700–713. doi:10.1158/1055-9965.EPI-13-1057
- Jeurnink SM, Steyerberg EW, van Hooft JE, et al; Dutch SUSTENT Study Group. Surgical gastrojejunostomy or endoscopic stent placement for the palliation of malignant gastric outlet obstruction (SUSTENT) study): a multicenter randomized trial. Gastrointest Endosc 2010; 71(3):490–499. doi:10.1016/j.gie.2009.09.042
- Tringali A, Didden P, Repici A, et al. Endoscopic treatment of malignant gastric and duodenal strictures: a prospective, multicenter study. Gastrointest Endosc 2014; 79(1):66–75. doi:10.1016/j.gie.2013.06.032
- Malfertheiner P, Chan FK, McColl KE. Peptic ulcer disease. Lancet 2009; 374(9699):1449–1461. doi:10.1016/S0140-6736(09)60938-7
- Gibson JB, Behrman SW, Fabian TC, Britt LG. Gastric outlet obstruction resulting from peptic ulcer disease requiring surgical intervention is infrequently associated with Helicobacter pylori infection. J Am Coll Surg 2000; 191(1):32–37. pmid:10898181
- Kochhar R, Kochhar S. Endoscopic balloon dilation for benign gastric outlet obstruction in adults. World J Gastrointest Endosc 2010; 2(1):29–35. doi:10.4253/wjge.v2.i1.29
- Kotisso R. Gastric outlet obstruction in Northwestern Ethiopia. East Cent Afr J Surg 2000; 5(2):25-29.
- Hamzaoui L, Bouassida M, Ben Mansour I, et al. Balloon dilatation in patients with gastric outlet obstruction related to peptic ulcer disease. Arab J Gastroenterol 2015; 16(3–4):121–124. doi:10.1016/j.ajg.2015.07.004
- Najm WI. Peptic ulcer disease. Prim Care 2011; 38(3):383–394. doi:10.1016/j.pop.2011.05.001
- Veloso N, Amaro P, Ferreira M, Romaozinho JM, Sofia C. Acute pancreatitis associated with a nontraumatic, intramural duodenal hematoma. Endoscopy 2013; 45(suppl 2):E51–E52. doi:10.1055/s-0032-1325969
- Maharshi S, Puri AS, Sachdeva S, Kumar A, Dalal A, Gupta M. Aetiological spectrum of benign gastric outlet obstruction in India: new trends. Trop Doct 2016; 46(4):186–191. doi:10.1177/0049475515626032
- Sala MA, Ligabo AN, de Arruda MC, Indiani JM, Nacif MS. Intestinal malrotation associated with duodenal obstruction secondary to Ladd’s bands. Radiol Bras 2016; 49(4):271–272. doi:10.1590/0100-3984.2015.0106
- Alibegovic E, Kurtcehajic A, Hujdurovic A, Mujagic S, Alibegovic J, Kurtcehajic D. Bouveret syndrome or gallstone ileus. Am J Med 2018; 131(4):e175. doi:10.1016/j.amjmed.2017.10.044
- Lau JY, Chung SC, Sung JJ, et al. Through-the-scope balloon dilation for pyloric stenosis: long-term results. Gastrointest Endosc 1996; 43(2 Pt 1):98–101. pmid:8635729
- Ray K, Snowden C, Khatri K, McFall M. Gastric outlet obstruction from a caecal volvulus, herniated through epiploic foramen: a case report. BMJ Case Rep 2009; pii:bcr05.2009.1880. doi:10.1136/bcr.05.2009.1880
- Baumgart DC, Fischer A. Virchow’s node. Lancet 2007; 370(9598):1568. doi:10.1016/S0140-6736(07)61661-4
- Dar IH, Kamili MA, Dar SH, Kuchaai FA. Sister Mary Joseph nodule—a case report with review of literature. J Res Med Sci 2009; 14(6):385–387. pmid:21772912
- Tang SJ. Endoscopic stent placement for gastric outlet obstruction. Video Journal and Encyclopedia of GI Endoscopy 2013; 1(1):133–136.
- Valero M, Robles-Medranda C. Endoscopic ultrasound in oncology: an update of clinical applications in the gastrointestinal tract. World J Gastrointest Endosc 2017; 9(6):243–254.
- ASGE Standards of Practice Committee; Fukami N, Anderson MA, Khan K, et al. The role of endoscopy in gastroduodenal obstruction and gastroparesis. Gastrointest Endosc 2011; 74(1):13–21. doi:10.1016/j.gie.2010.12.003
- Ros PR, Huprich JE. ACR appropriateness criteria on suspected small-bowel obstruction. J Am Coll Radiol 2006; 3(11):838–841. doi:10.1016/j.jacr.2006.09.018
- Pasricha PJ, Parkman HP. Gastroparesis: definitions and diagnosis. Gastroenterol Clin North Am 2015; 44(1):1–7. doi:10.1016/j.gtc.2014.11.001
- Stein B, Everhart KK, Lacy BE. Gastroparesis: a review of current diagnosis and treatment options. J Clin Gastroenterol 2015; 49(7):550–558. doi:10.1097/MCG.0000000000000320
- Camilleri M, Parkman HP, Shafi MA, Abell TL, Gerson L; American College of Gastroenterology. Clinical guideline: management of gastroparesis. Am J Gastroenterol 2013; 108(1):18–37.
- Gursoy O, Memis D, Sut N. Effect of proton pump inhibitors on gastric juice volume, gastric pH and gastric intramucosal pH in critically ill patients: a randomized, double-blind, placebo-controlled study. Clin Drug Investig 2008; 28(12):777–782. doi:10.2165/0044011-200828120-00005
- Kuwada SK, Alexander GL. Long-term outcome of endoscopic dilation of nonmalignant pyloric stenosis. Gastrointest Endosc 1995; 41(1):15–17. pmid:7698619
- Kochhar R, Sethy PK, Nagi B, Wig JD. Endoscopic balloon dilatation of benign gastric outlet obstruction. J Gastroenterol Hepatol 2004; 19(4):418–422. pmid:15012779
- Perng CL, Lin HJ, Lo WC, Lai CR, Guo WS, Lee SD. Characteristics of patients with benign gastric outlet obstruction requiring surgery after endoscopic balloon dilation. Am J Gastroenterol 1996; 91(5):987–990. pmid:8633593
- Taskin V, Gurer I, Ozyilkan E, Sare M, Hilmioglu F. Effect of Helicobacter pylori eradication on peptic ulcer disease complicated with outlet obstruction. Helicobacter 2000; 5(1):38–40. pmid:10672050
- de Boer WA, Driessen WM. Resolution of gastric outlet obstruction after eradication of Helicobacter pylori. J Clin Gastroenterol 1995; 21(4):329–330. pmid:8583113
- Tursi A, Cammarota G, Papa A, Montalto M, Fedeli G, Gasbarrini G. Helicobacter pylori eradication helps resolve pyloric and duodenal stenosis. J Clin Gastroenterol 1996; 23(2):157–158. pmid:8877648
- Schmassmann A. Mechanisms of ulcer healing and effects of nonsteroidal anti-inflammatory drugs. Am J Med 1998; 104(3A):43S–51S; discussion 79S–80S. pmid:9572320
- Kim HU. Diagnostic and treatment approaches for refractory peptic ulcers. Clin Endosc 2015; 48(4):285–290. doi:10.5946/ce.2015.48.4.285
- Ong TZ, Hawkey CJ, Ho KY. Nonsteroidal anti-inflammatory drug use is a significant cause of peptic ulcer disease in a tertiary hospital in Singapore: a prospective study. J Clin Gastroenterol 2006; 40(9):795–800. doi:10.1097/01.mcg.0000225610.41105.7f
- Lanas A, Sekar MC, Hirschowitz BI. Objective evidence of aspirin use in both ulcer and nonulcer upper and lower gastrointestinal bleeding. Gastroenterology 1992; 103(3):862–869. pmid:1499936
- Zhang LP, Tabrizian P, Nguyen S, Telem D, Divino C. Laparoscopic gastrojejunostomy for the treatment of gastric outlet obstruction. JSLS 2011; 15(2):169–173. doi:10.4293/108680811X13022985132074
- Lagoo J, Pappas TN, Perez A. A relic or still relevant: the narrowing role for vagotomy in the treatment of peptic ulcer disease. Am J Surg 2014; 207(1):120–126. doi:10.1016/j.amjsurg.2013.02.012
- Csendes A, Maluenda F, Braghetto I, Schutte H, Burdiles P, Diaz JC. Prospective randomized study comparing three surgical techniques for the treatment of gastric outlet obstruction secondary to duodenal ulcer. Am J Surg 1993; 166(1):45–49. pmid:8101050
- Ly J, O’Grady G, Mittal A, Plank L, Windsor JA. A systematic review of methods to palliate malignant gastric outlet obstruction. Surg Endosc 2010; 24(2):290–297. doi:10.1007/s00464-009-0577-1
- Goldberg EM. Palliative treatment of gastric outlet obstruction in terminal patients: SEMS. Stent every malignant stricture! Gastrointest Endosc 2014; 79(1):76–78. doi:10.1016/j.gie.2013.07.056
- Min SH, Son SY, Jung DH, et al. Laparoscopic gastrojejunostomy versus duodenal stenting in unresectable gastric cancer with gastric outlet obstruction. Ann Surg Treat Res 2017; 93(3):130–136. doi:10.4174/astr.2017.93.3.130
- Roy A, Kim M, Christein J, Varadarajulu S. Stenting versus gastrojejunostomy for management of malignant gastric outlet obstruction: comparison of clinical outcomes and costs. Surg Endosc 2012; 26(11):3114–119. doi:10.1007/s00464-012-2301-9
- Amin S, Sethi A. Endoscopic ultrasound-guided gastrojejunostomy. Gastrointest Endosc Clin N Am 2017; 27(4):707–713. doi:10.1016/j.giec.2017.06.009
KEY POINTS
- Causes of gastric outlet obstruction fall into 2 categories: benign and malignant. The cause should be presumed to be malignant until proven otherwise.
- Peptic ulcer disease, a benign cause, used to account for most cases of gastric outlet obstruction. It is still common but has declined in frequency with the development of acid-suppressing drugs.
- Gastric cancer used to be the most common malignant cause but has declined in frequency in Western countries with treatment for Helicobacter pylori infection. Now, pancreatic cancer predominates.
- Endoscopic stenting is an effective, minimally invasive treatment for patients with malignant gastric outlet obstruction and poor prognosis, allowing resumption of oral intake and improving quality of life.
Noninvasive FFRCT called ADVANCE in chest pain assessment
NEW ORLEANS – Fractional flow reserve derived noninvasively from coronary CT angiography showed clinical merit as a practical tool for evaluation of chest pain at 1 year of follow-up in the ADVANCE registry, Manesh R. Patel, MD, reported at the annual meeting of the American College of Cardiology.
In ADVANCE, a fractional flow reserve value greater than 0.80 derived from CT angiography, or FFRCT, was associated with a significantly lower rate of cardiovascular death or MI at 1 year than in patients with an FFRCT of 0.80 or lower, according to Dr. Patel, professor of medicine and chief of the division of cardiology at Duke University, Durham, N.C.
“The lower rates of revascularization and clinical events in patients with FFRCT who were managed conservatively provide reassurance regarding this clinical strategy if you were to put it into your practice,” he observed.
ADVANCE is in an international, real-world, prospective registry of more than 5,000 patients in Europe, Japan, and North America. All had clinically suspected ischemic coronary artery disease (CAD). They also had at least 30% atherosclerosis documented on coronary CT angiography as a trigger for noninvasive assessment of FFR calculated from computational fluid dynamics. The idea behind FFRCT is that by combining the anatomic information provided by CT angiography with the physiological, functional data from FFR, the result is a better guide to need for revascularization of true obstructive CAD than with conventional invasive coronary angiography alone. Indeed, FFRCT could eventually prove to be a cost-effective gatekeeper to the cardiac catheterization laboratory by cutting down on high rates of invasive coronary angiography for nonactionable CAD.
That point was suggested by the previously reported 90-day outcomes of the ADVANCE registry, the cardiologist explained. Participating physicians first classified patients and made a revascularization/no-revascularization management plan on the basis of the core laboratory CT angiography results alone. But when they received the FFRCT results, they reclassified patients and changed the management plan in 67% of cases. That’s because the prevalence of nonobstructive CAD was 44% in patients with an FFRCT greater than 0.80 in all coronary arteries, compared with just 14% in those with an FFRCT of 0.80 or less. As a result, 72% of patients with an FFRCT of 0.80 or less underwent revascularization, while the vast majority of patients with an FFRCT greater than 0.80 were initially managed conservatively (Eur Heart J. 2018 Nov 1;39[41]:3701-11).
The 1-year outcomes from ADVANCE as presented by Dr. Patel showed low rates of major adverse cardiovascular events overall. Of note, the composite endpoint of cardiovascular death or MI occurred significantly more often in patients with an FFRCT of 0.80 or less, by a margin of 0.8% versus 0.2%, for a 320% increased relative risk. The patients with a FFRCT greater than 0.80 continued to have a much lower revascularization rate from 90 days through 1 year: 5.8% versus 38.4% in the lower-FFRCT group. And 93% of patients placed on medical therapy alone after receiving their FFRCT results remained on medical therapy without revascularization or a major adverse cardiovascular event at 1 year.
Discussant Matthew J. Budoff, MD, commented that it’s time to move beyond observational studies and conduct randomized trials of an FFRCT-based screening strategy in patients with clinical suspicion of obstructive CAD.
“We want to understand the enormous advantages of having FFR-like data before we take patients to the cath lab. And I do think that adding physiology to the anatomy is going to be the approach that we’re going to be predominantly using in the future,” said Dr. Budoff, professor of medicine at the University of California, Los Angeles.
Dr. Patel noted that the ongoing, randomized, 2,100-patient PRECISE study is directed at determining in a more definitive way the clinical and cost-effectiveness of an FFRCT strategy.
The ADVANCE registry is funded by HeartFlow. Dr. Patel reported receiving research grants from that company and several others, as well as the National Institutes of Health. He serves on advisory boards for Bayer, Janssen, and Amgen.
Simultaneous with Dr. Patel’s presentation at ACC 2019, the 1-year ADVANCE registry results were published online (JACC Cardiovasc Imag. 2019 Mar 17. doi: 10.1016/j.jcmg.2019.03.003).
NEW ORLEANS – Fractional flow reserve derived noninvasively from coronary CT angiography showed clinical merit as a practical tool for evaluation of chest pain at 1 year of follow-up in the ADVANCE registry, Manesh R. Patel, MD, reported at the annual meeting of the American College of Cardiology.
In ADVANCE, a fractional flow reserve value greater than 0.80 derived from CT angiography, or FFRCT, was associated with a significantly lower rate of cardiovascular death or MI at 1 year than in patients with an FFRCT of 0.80 or lower, according to Dr. Patel, professor of medicine and chief of the division of cardiology at Duke University, Durham, N.C.
“The lower rates of revascularization and clinical events in patients with FFRCT who were managed conservatively provide reassurance regarding this clinical strategy if you were to put it into your practice,” he observed.
ADVANCE is in an international, real-world, prospective registry of more than 5,000 patients in Europe, Japan, and North America. All had clinically suspected ischemic coronary artery disease (CAD). They also had at least 30% atherosclerosis documented on coronary CT angiography as a trigger for noninvasive assessment of FFR calculated from computational fluid dynamics. The idea behind FFRCT is that by combining the anatomic information provided by CT angiography with the physiological, functional data from FFR, the result is a better guide to need for revascularization of true obstructive CAD than with conventional invasive coronary angiography alone. Indeed, FFRCT could eventually prove to be a cost-effective gatekeeper to the cardiac catheterization laboratory by cutting down on high rates of invasive coronary angiography for nonactionable CAD.
That point was suggested by the previously reported 90-day outcomes of the ADVANCE registry, the cardiologist explained. Participating physicians first classified patients and made a revascularization/no-revascularization management plan on the basis of the core laboratory CT angiography results alone. But when they received the FFRCT results, they reclassified patients and changed the management plan in 67% of cases. That’s because the prevalence of nonobstructive CAD was 44% in patients with an FFRCT greater than 0.80 in all coronary arteries, compared with just 14% in those with an FFRCT of 0.80 or less. As a result, 72% of patients with an FFRCT of 0.80 or less underwent revascularization, while the vast majority of patients with an FFRCT greater than 0.80 were initially managed conservatively (Eur Heart J. 2018 Nov 1;39[41]:3701-11).
The 1-year outcomes from ADVANCE as presented by Dr. Patel showed low rates of major adverse cardiovascular events overall. Of note, the composite endpoint of cardiovascular death or MI occurred significantly more often in patients with an FFRCT of 0.80 or less, by a margin of 0.8% versus 0.2%, for a 320% increased relative risk. The patients with a FFRCT greater than 0.80 continued to have a much lower revascularization rate from 90 days through 1 year: 5.8% versus 38.4% in the lower-FFRCT group. And 93% of patients placed on medical therapy alone after receiving their FFRCT results remained on medical therapy without revascularization or a major adverse cardiovascular event at 1 year.
Discussant Matthew J. Budoff, MD, commented that it’s time to move beyond observational studies and conduct randomized trials of an FFRCT-based screening strategy in patients with clinical suspicion of obstructive CAD.
“We want to understand the enormous advantages of having FFR-like data before we take patients to the cath lab. And I do think that adding physiology to the anatomy is going to be the approach that we’re going to be predominantly using in the future,” said Dr. Budoff, professor of medicine at the University of California, Los Angeles.
Dr. Patel noted that the ongoing, randomized, 2,100-patient PRECISE study is directed at determining in a more definitive way the clinical and cost-effectiveness of an FFRCT strategy.
The ADVANCE registry is funded by HeartFlow. Dr. Patel reported receiving research grants from that company and several others, as well as the National Institutes of Health. He serves on advisory boards for Bayer, Janssen, and Amgen.
Simultaneous with Dr. Patel’s presentation at ACC 2019, the 1-year ADVANCE registry results were published online (JACC Cardiovasc Imag. 2019 Mar 17. doi: 10.1016/j.jcmg.2019.03.003).
NEW ORLEANS – Fractional flow reserve derived noninvasively from coronary CT angiography showed clinical merit as a practical tool for evaluation of chest pain at 1 year of follow-up in the ADVANCE registry, Manesh R. Patel, MD, reported at the annual meeting of the American College of Cardiology.
In ADVANCE, a fractional flow reserve value greater than 0.80 derived from CT angiography, or FFRCT, was associated with a significantly lower rate of cardiovascular death or MI at 1 year than in patients with an FFRCT of 0.80 or lower, according to Dr. Patel, professor of medicine and chief of the division of cardiology at Duke University, Durham, N.C.
“The lower rates of revascularization and clinical events in patients with FFRCT who were managed conservatively provide reassurance regarding this clinical strategy if you were to put it into your practice,” he observed.
ADVANCE is in an international, real-world, prospective registry of more than 5,000 patients in Europe, Japan, and North America. All had clinically suspected ischemic coronary artery disease (CAD). They also had at least 30% atherosclerosis documented on coronary CT angiography as a trigger for noninvasive assessment of FFR calculated from computational fluid dynamics. The idea behind FFRCT is that by combining the anatomic information provided by CT angiography with the physiological, functional data from FFR, the result is a better guide to need for revascularization of true obstructive CAD than with conventional invasive coronary angiography alone. Indeed, FFRCT could eventually prove to be a cost-effective gatekeeper to the cardiac catheterization laboratory by cutting down on high rates of invasive coronary angiography for nonactionable CAD.
That point was suggested by the previously reported 90-day outcomes of the ADVANCE registry, the cardiologist explained. Participating physicians first classified patients and made a revascularization/no-revascularization management plan on the basis of the core laboratory CT angiography results alone. But when they received the FFRCT results, they reclassified patients and changed the management plan in 67% of cases. That’s because the prevalence of nonobstructive CAD was 44% in patients with an FFRCT greater than 0.80 in all coronary arteries, compared with just 14% in those with an FFRCT of 0.80 or less. As a result, 72% of patients with an FFRCT of 0.80 or less underwent revascularization, while the vast majority of patients with an FFRCT greater than 0.80 were initially managed conservatively (Eur Heart J. 2018 Nov 1;39[41]:3701-11).
The 1-year outcomes from ADVANCE as presented by Dr. Patel showed low rates of major adverse cardiovascular events overall. Of note, the composite endpoint of cardiovascular death or MI occurred significantly more often in patients with an FFRCT of 0.80 or less, by a margin of 0.8% versus 0.2%, for a 320% increased relative risk. The patients with a FFRCT greater than 0.80 continued to have a much lower revascularization rate from 90 days through 1 year: 5.8% versus 38.4% in the lower-FFRCT group. And 93% of patients placed on medical therapy alone after receiving their FFRCT results remained on medical therapy without revascularization or a major adverse cardiovascular event at 1 year.
Discussant Matthew J. Budoff, MD, commented that it’s time to move beyond observational studies and conduct randomized trials of an FFRCT-based screening strategy in patients with clinical suspicion of obstructive CAD.
“We want to understand the enormous advantages of having FFR-like data before we take patients to the cath lab. And I do think that adding physiology to the anatomy is going to be the approach that we’re going to be predominantly using in the future,” said Dr. Budoff, professor of medicine at the University of California, Los Angeles.
Dr. Patel noted that the ongoing, randomized, 2,100-patient PRECISE study is directed at determining in a more definitive way the clinical and cost-effectiveness of an FFRCT strategy.
The ADVANCE registry is funded by HeartFlow. Dr. Patel reported receiving research grants from that company and several others, as well as the National Institutes of Health. He serves on advisory boards for Bayer, Janssen, and Amgen.
Simultaneous with Dr. Patel’s presentation at ACC 2019, the 1-year ADVANCE registry results were published online (JACC Cardiovasc Imag. 2019 Mar 17. doi: 10.1016/j.jcmg.2019.03.003).
REPORTING FROM ACC 19
Prevalence and outcomes of incidental imaging findings
Background: As frequency of imaging studies increases, and those studies become more advanced, incidental findings on imaging are a growing concern. Incidentalomas can lead to anxiety for patients, increased testing, and possible interventions such as biopsies. Current literature does not provide adequate guidance for providers to discuss the risks of incidentalomas with patients, nor are there clear methods described to manage incidentalomas when discovered.
Study design: This study was an umbrella review of systematic reviews and meta-analyses. Authors conduced their own meta-analyses using data from pooled sources.
Setting: MEDLINE and EMBASE were searched, which resulted in 20 unique systematic reviews analyzed, 15 of which provided incidence data and 18 included outcome data.
Synopsis: To assess prevalence of incidentalomas, the authors conducted nine meta-analyses, with a median number of 14,409 patients. Each analysis was created based on the imaging modality used and the area of the body where the incidental finding occurred. They examined the outcomes specific to incidentalomas within those organs. Their analysis showed that CT of the chest had the highest prevalence of incidentalomas (45%; 95% confidence interval, 36%-55%). Incidental findings in the breast had the highest rates of malignancy (42%; 95% CI, 31%-54%). Noncancerous outcomes described included disc degeneration on MRIs of the spine, aneurysms in brain imaging, and subclinical Cushing’s syndrome. There was significant heterogeneity in all the meta-analyses conducted.
Limitations included variations in how primary study authors defined a positive result and in imaging protocols. Although the authors of this study used primary data extracted from the individual studies in the systematic reviews, they did not analyze the primary studies for inclusion based on methods.
Bottom line: This study provides guidance to clinicians regarding counseling patients on the risks of incidentalomas and how to manage those incidental findings.
Citation: O’Sullivan JW et al. Prevalence and outcomes of incidental imaging findings: umbrella review. BMJ. 2018 Jun 18. doi: 10.1136/bmj.k2387.
Dr. Witt is an assistant professor of medicine in the division of hospital medicine at Emory University, Atlanta.
Background: As frequency of imaging studies increases, and those studies become more advanced, incidental findings on imaging are a growing concern. Incidentalomas can lead to anxiety for patients, increased testing, and possible interventions such as biopsies. Current literature does not provide adequate guidance for providers to discuss the risks of incidentalomas with patients, nor are there clear methods described to manage incidentalomas when discovered.
Study design: This study was an umbrella review of systematic reviews and meta-analyses. Authors conduced their own meta-analyses using data from pooled sources.
Setting: MEDLINE and EMBASE were searched, which resulted in 20 unique systematic reviews analyzed, 15 of which provided incidence data and 18 included outcome data.
Synopsis: To assess prevalence of incidentalomas, the authors conducted nine meta-analyses, with a median number of 14,409 patients. Each analysis was created based on the imaging modality used and the area of the body where the incidental finding occurred. They examined the outcomes specific to incidentalomas within those organs. Their analysis showed that CT of the chest had the highest prevalence of incidentalomas (45%; 95% confidence interval, 36%-55%). Incidental findings in the breast had the highest rates of malignancy (42%; 95% CI, 31%-54%). Noncancerous outcomes described included disc degeneration on MRIs of the spine, aneurysms in brain imaging, and subclinical Cushing’s syndrome. There was significant heterogeneity in all the meta-analyses conducted.
Limitations included variations in how primary study authors defined a positive result and in imaging protocols. Although the authors of this study used primary data extracted from the individual studies in the systematic reviews, they did not analyze the primary studies for inclusion based on methods.
Bottom line: This study provides guidance to clinicians regarding counseling patients on the risks of incidentalomas and how to manage those incidental findings.
Citation: O’Sullivan JW et al. Prevalence and outcomes of incidental imaging findings: umbrella review. BMJ. 2018 Jun 18. doi: 10.1136/bmj.k2387.
Dr. Witt is an assistant professor of medicine in the division of hospital medicine at Emory University, Atlanta.
Background: As frequency of imaging studies increases, and those studies become more advanced, incidental findings on imaging are a growing concern. Incidentalomas can lead to anxiety for patients, increased testing, and possible interventions such as biopsies. Current literature does not provide adequate guidance for providers to discuss the risks of incidentalomas with patients, nor are there clear methods described to manage incidentalomas when discovered.
Study design: This study was an umbrella review of systematic reviews and meta-analyses. Authors conduced their own meta-analyses using data from pooled sources.
Setting: MEDLINE and EMBASE were searched, which resulted in 20 unique systematic reviews analyzed, 15 of which provided incidence data and 18 included outcome data.
Synopsis: To assess prevalence of incidentalomas, the authors conducted nine meta-analyses, with a median number of 14,409 patients. Each analysis was created based on the imaging modality used and the area of the body where the incidental finding occurred. They examined the outcomes specific to incidentalomas within those organs. Their analysis showed that CT of the chest had the highest prevalence of incidentalomas (45%; 95% confidence interval, 36%-55%). Incidental findings in the breast had the highest rates of malignancy (42%; 95% CI, 31%-54%). Noncancerous outcomes described included disc degeneration on MRIs of the spine, aneurysms in brain imaging, and subclinical Cushing’s syndrome. There was significant heterogeneity in all the meta-analyses conducted.
Limitations included variations in how primary study authors defined a positive result and in imaging protocols. Although the authors of this study used primary data extracted from the individual studies in the systematic reviews, they did not analyze the primary studies for inclusion based on methods.
Bottom line: This study provides guidance to clinicians regarding counseling patients on the risks of incidentalomas and how to manage those incidental findings.
Citation: O’Sullivan JW et al. Prevalence and outcomes of incidental imaging findings: umbrella review. BMJ. 2018 Jun 18. doi: 10.1136/bmj.k2387.
Dr. Witt is an assistant professor of medicine in the division of hospital medicine at Emory University, Atlanta.
Spontaneous coronary artery dissection: An often unrecognized cause of acute coronary syndrome
A 55-year-old woman with hypertension presented with sudden onset of severe pressure-like chest pain that started when she was taking a bath. The pain radiated to her back and was associated with nausea.
A 12-lead electrocardiogram (Figure 1) showed ST-segment elevation of more than 2 mm in leads V2, V3, V4, and V5, with no reciprocal changes.
In view of her ongoing severe chest pain, she was given aspirin and ticagrelor and was taken for emergency cardiac catheterization. Left ventriculography showed hypokinesis in the mid-anterior, distal anterior, apical, and distal inferior chamber walls. Coronary angiography (Figure 2) revealed a long segment of diffuse, smooth narrowing of the mid-left anterior descending coronary artery that did not reverse after administration of intracoronary nitroglycerin.
Based on the classic angiographic appearance and the absence of atherosclerotic disease in other coronary arteries, type 2 spontaneous coronary artery dissection (SCAD) was diagnosed.
CORONARY ARTERY WALL SEPARATION
SCAD is defined as a nontraumatic, noniatrogenic intramural hemorrhage leading to separation of the coronary arterial wall and the formation of a false lumen. The separation can occur between any of the coronary artery wall layers and may or may not involve an intimal tear. The bleeding may result in an intramural hematoma and possible narrowing of the arterial lumen. Depending on the severity of narrowing, blood supply to the myocardium could be compromised, resulting in symptoms of ischemia.1
SCAD usually involves a single coronary artery, although multiple coronary artery involvement has been reported.2
CASE CONTINUED: MANAGEMENT
Conservative management is generally recommended for SCAD. An initial decision was made to continue medical management alone. But because the patient continued to have severe chest pain that was unresponsive to intravenous nitroglycerin and intravenous morphine and was accompanied by frequent episodes of nonsustained ventricular tachycardia during cardiac catheterization, the management team decided to proceed with percutaneous coronary intervention (PCI). Implantation of a 2.25-by-38-mm drug-eluting stent in the left anterior descending artery was successful, resulting in return of normal flow (Thrombolysis in Myocardial Infarction [TIMI] score 3) and only a small distal residual non-flow-limiting dissection (Figure 3).
The patient recovered completely and was discharged home with plans to return for outpatient imaging for fibromuscular dysplasia.
SCAD: RARE OR JUST RARELY RECOGNIZED?
SCAD appears to be a rare cause of acute coronary syndrome, but it is likely underdiagnosed and is becoming increasingly recognized worldwide. Typically, it affects women younger than 50, with women in general outnumbering men 9 to 1.3 Overall, SCAD causes up to 4% of acute myocardial infarctions, but in women age 50 or younger, it is responsible for 24% to 35% of acute myocardial infarctions, and the proportion is even higher in pregnant women.4
Not just pregnancy-associated
SCAD was previously thought to be mainly idiopathic and mostly affecting women peripartum. Current understanding paints a different picture: pregnancy-associated SCAD does not account for the majority of cases. That said, SCAD is the most common cause of myocardial infarction peripartum, with the third trimester and early postpartum period being the times of highest risk.5 SCAD development at those times is believed to be related to hormonal changes causing weakening of coronary artery walls.6
Weakening of the coronary artery wall also may occur in the setting of fibromuscular dysplasia, connective tissue disease, recurrent pregnancies, systemic inflammatory disease, hormonal therapy, and other disease states that cause arteriopathy. Exposure to a stressor in a patient with underlying risk factors can lead to either an intimal tear or rupture of the vasa vasorum, with subsequent formation of intramural hemorrhage and eventually SCAD.7 Stressors can be emotional or physical and can include labor and delivery, intense physical exercise, the Valsalva maneuver, and drug abuse.8
Presentation is variable
SCAD presentation depends on the degree of flow limitation and extent of the dissection. Presentation can range from asymptomatic to sudden cardiac death and can include signs and symptoms of acute coronary syndrome caused by ST-segment elevation or non-ST-segment elevation myocardial infarction.
DIAGNOSIS BY ANGIOGRAPHY
SCAD can be diagnosed by coronary angiography. There are 3 angiographic types:
Type 1 (about 25% of SCAD cases) has typical contrast dye staining of the arterial wall and multiple radiolucent luminal abnormalities, with or without dye hang-up.
Type 2 (about 70%) has diffuse, smooth narrowing of the coronary artery, with the left anterior descending artery the most frequently affected.8
Type 3 (about 5%) mimics atherosclerosis, with focal or tubular stenosis.9
Types 1 and 2 are usually easy to recognize. To diagnose type 2, intravenous nitroglycerin should first be administered to rule out coronary spasm.
Type 3 SCAD is more challenging to diagnose because its appearance on angiography is similar to that of atherosclerosis. For equivocal findings in any type, but especially in type 3, intravascular ultrasonography or optical coherence tomography can help.10 Optical coherence tomography is preferred because of superior image resolution, although ultrasonography offers better tissue penetration.11
MANAGE MOST CASES CONSERVATIVELY
Management algorithms for SCAD are available.8,12
The initial and most critical step is to make the correct diagnosis. Although the presentation of acute coronary syndrome caused by SCAD is often identical to that of atherosclerosis, the conditions have different pathophysiologies and thus require different management. Theoretically, systemic anticoagulation may worsen an intramural hemorrhage.
First-line therapy for most patients with SCAD is conservative management and close inpatient monitoring for 3 to 5 days.13 More aggressive management is indicated for any of the following:
- Left main or severe proximal 2-vessel dissection
- Hemodynamic instability
- Ongoing ischemic symptoms.
In a prospective cohort of 168 patients, 134 (80%) were initially treated conservatively; of those, in-hospital myocardial infarction recurred in 4.5%, a major cardiac event occurred within 2 years in 17%, and SCAD recurred in 13%.8
Observational data on patients with SCAD who had repeat angiography weeks to months after the initial event has shown that lesions heal in 70% to 97% of patients.12
WHEN TO CONSIDER AGGRESSIVE MANAGEMENT
Under the circumstances listed above, revascularization with PCI or coronary artery bypass grafting (CABG) should be considered, with choice of procedure determined by feasibility, technical considerations, and local expertise.
The American Heart Association recommendations are as follows12:
- For left main or severe proximal 2-vessel dissection in clinically stable patients, consider CABG
- For active ischemia or hemodynamic instability, consider PCI if feasible or perform urgent CABG.
A few series have shown that the prognosis with conservative management or CABG is better than with PCI.8,13,14 The success rate for revascularization with PCI is only about 60% because of challenges including risk of inducing iatrogenic dissection, passing the wire into the false lumen and worsening a dissection, and propagating an intramural hematoma with stenting and further compromising coronary blood flow. In addition, dissection tends to extend into distal arteries that are difficult to stent. There is also the risk of stent malapposition after resorption of the intramural hematoma, causing late stent thrombosis.7
SCREEN FOR OTHER VASCULAR PROBLEMS
Imaging of the renal, iliac, and cerebral vasculature is recommended for all patients with SCAD.12 Screening for fibromuscular dysplasia can be done with angiography, computed tomographic angiography (CTA), or magnetic resonance angiography (MRA).12
Multifocal fibromuscular dysplasia in extracoronary arteries occurs with SCAD in 25% to 86% of cases. In a single-center series of 115 patients with confirmed SCAD who underwent CTA from 2010 to 2014, extracoronary vascular abnormalities were found in 66%, with fibromuscular dysplasia being the most common type (45%).15 In another single-center study, 327 patients with SCAD were prospectively followed from 2012 to 2016 with screening for cerebrovascular, renal, and iliac fibromuscular dysplasia using CTA or catheter angiography. Fibromuscular dysplasia was found in 63%, and intracranial aneurysm was found in 14% of patients with fibromuscular dysplasia.9
SCAD can also be associated with connective tissue disorders such as Ehlers-Danlos syndrome type IV and Marfan syndrome.16,17
LONG-TERM MANAGEMENT
Patients with SCAD should start long-term aspirin and 1 year of clopidogrel. Statins are indicated for patients with hyperlipidemia8,18 but otherwise offer no clear benefit for SCAD alone. If there are no contraindications, a beta-adrenergic blocker should be considered, especially if left ventricular dysfunction or arrhythmias are present. Angiotensin-converting enzyme inhibitors and angiotensin II receptor blockers should also be considered with concomitant left ventricular dysfunction. Antianginal therapy can be used for post-SCAD chest pain syndromes.12
Repeat angiography is recommended only to evaluate recurrent symptoms, to confirm an unclear initial diagnosis, to assess for atherosclerosis-related stenosis, or to evaluate high-risk anatomy, eg, involvement of the left main coronary artery.12
Genetic testing is reserved for patients with a high clinical suspicion of connective tissue disease or systemic arteriopathy.19
- Garcia NA, Khan AN, Boppana RC, Smith HL. Spontaneous coronary artery dissection: a case series and literature review. J Community Hosp Intern Med Perspect 2014; 4(4). doi:10.3402/jchimp.v4.25261
- Lempereur M, Gin K, Saw J. Multivessel spontaneous coronary artery dissection mimicking atherosclerosis. JACC Cardiovasc Interv 2014; 7(7):e87–e88. doi:10.1016/j.jcin.2013.12.207
- Mahmoud AN, Taduru SS, Mentias A, et al. Trends of incidence, clinical presentation, and in-hospital mortality among women with acute myocardial infarction with or without spontaneous coronary artery dissection: a population-based analysis. JACC Cardiovasc Interv 2018; 11(1):80–90. doi:10.1016/j.jcin.2017.08.016
- Saw J. Pregnancy-associated spontaneous coronary artery dissection represents an exceptionally high-risk spontaneous coronary artery dissection cohort. Circ Cardiovasc Interv 2017; 10(3)pii:e005119. doi:10.1161/CIRCINTERVENTIONS.117.005119
- Elkayam U, Jalnapurkar S, Barakkat MN, et al. Pregnancy-associated acute myocardial infarction: a review of contemporary experience in 150 cases between 2006 and 2011. Circulation 2014; 129(16):1695–1702. doi:10.1161/CIRCULATIONAHA.113.002054
- Vijayaraghavan R, Verma S, Gupta N, Saw J. Pregnancy-related spontaneous coronary artery dissection. Circulation 2014; 130(21):1915–1920. doi:10.1161/CIRCULATIONAHA.114.011422
- Saw J, Mancini GBJ, Humphries KH. Contemporary review on spontaneous coronary artery dissection. J Am Coll Cardiol 2016; 68(3):297–312. doi:10.1016/j.jacc.2016.05.034
- Saw J, Aymong E, Sedlak T, et al. Spontaneous coronary artery dissection: association with predisposing arteriopathies and precipitating stressors and cardiovascular outcomes. Circ Cardiovasc Interv 2014; 7(5):645–655. doi:10.1161/CIRCINTERVENTIONS.114.001760
- Saw J, Humphries K ,Aymong E, et al. Spontaneous coronary artery dissection: clinical outcomes and risk of recurrence. J Am Coll Cardiol 2017; 70(9):1148–1158. doi:10.1016/j.jacc.2017.06.053
- Alfonso F, Bastante T, Cuesta J, Rodríguez D, Benedicto A, Rivero F. Spontaneous coronary artery dissection: novel insights on diagnosis and management. Cardiovasc Diagn Ther 2015; 5(2):133–140. doi:10.3978/j.issn.2223-3652.2015.03.05
- Kern MJ, Meier B. Evaluation of the culprit plaque and the physiological significance of coronary atherosclerotic narrowings. Circulation 2001; 103(25):3142–3149. pmid:11425782
- Hayes SN, Kim ESH, Saw J, et al; American Heart Association Council on Peripheral Vascular Disease; Council on Clinical Cardiology; Council on Cardiovascular and Stroke Nursing; Council on Genomic and Precision Medicine; and Stroke Council. Spontaneous coronary artery dissection: current state of the science: a scientific statement from the American Heart Association. Circulation 2018; 137(19):e523–e557. doi:10.1161/CIR.0000000000000564
- Tweet MS, Eleid MF, Best PJ, et al. Spontaneous coronary artery dissection: revascularization versus conservative therapy. Circ Cardiovasc Interv 2014; 7(6):777–786. doi:10.1161/CIRCINTERVENTIONS.114.001659
- Tweet MS, Hayes SN, Pitta SR, et al. Clinical features, management, and prognosis of spontaneous coronary artery dissection. Circulation 2012; 126(5):579–588. doi:10.1161/CIRCULATIONAHA.112.105718
- Prasad M, Tweet MS, Hayes SN, et al. Prevalence of extracoronary vascular abnormalities and fibromuscular dysplasia in patients with spontaneous coronary artery dissection. Am J Cardiol 2015; 115(12):1672–1677. doi:10.1016/j.amjcard.2015.03.011
- Adès LC, Waltham RD, Chiodo AA, Bateman JF. Myocardial infarction resulting from coronary artery dissection in an adolescent with Ehlers-Danlos syndrome type IV due to a type III collagen mutation. Br Heart J 1995; 74(2):112–116. pmid:7546986
- Judge DP, Dietz HC. Marfan’s syndrome. Lancet 2005; 366(9501):1965–1976. doi:10.1016/S0140-6736(05)67789-6
- Saw J. Spontaneous coronary artery dissection. Can J Cardiol 2013; 29(9):1027–1033. doi:10.1016/j.cjca.2012.12.018
- Poloskey SL, Kim ES, Sanghani R, et al. Low yield of genetic testing for known vascular connective tissue disorders in patients with fibromuscular dysplasia. Vasc Med 2012; 17(6):371–378. doi:10.1177/1358863X12459650
A 55-year-old woman with hypertension presented with sudden onset of severe pressure-like chest pain that started when she was taking a bath. The pain radiated to her back and was associated with nausea.
A 12-lead electrocardiogram (Figure 1) showed ST-segment elevation of more than 2 mm in leads V2, V3, V4, and V5, with no reciprocal changes.
In view of her ongoing severe chest pain, she was given aspirin and ticagrelor and was taken for emergency cardiac catheterization. Left ventriculography showed hypokinesis in the mid-anterior, distal anterior, apical, and distal inferior chamber walls. Coronary angiography (Figure 2) revealed a long segment of diffuse, smooth narrowing of the mid-left anterior descending coronary artery that did not reverse after administration of intracoronary nitroglycerin.
Based on the classic angiographic appearance and the absence of atherosclerotic disease in other coronary arteries, type 2 spontaneous coronary artery dissection (SCAD) was diagnosed.
CORONARY ARTERY WALL SEPARATION
SCAD is defined as a nontraumatic, noniatrogenic intramural hemorrhage leading to separation of the coronary arterial wall and the formation of a false lumen. The separation can occur between any of the coronary artery wall layers and may or may not involve an intimal tear. The bleeding may result in an intramural hematoma and possible narrowing of the arterial lumen. Depending on the severity of narrowing, blood supply to the myocardium could be compromised, resulting in symptoms of ischemia.1
SCAD usually involves a single coronary artery, although multiple coronary artery involvement has been reported.2
CASE CONTINUED: MANAGEMENT
Conservative management is generally recommended for SCAD. An initial decision was made to continue medical management alone. But because the patient continued to have severe chest pain that was unresponsive to intravenous nitroglycerin and intravenous morphine and was accompanied by frequent episodes of nonsustained ventricular tachycardia during cardiac catheterization, the management team decided to proceed with percutaneous coronary intervention (PCI). Implantation of a 2.25-by-38-mm drug-eluting stent in the left anterior descending artery was successful, resulting in return of normal flow (Thrombolysis in Myocardial Infarction [TIMI] score 3) and only a small distal residual non-flow-limiting dissection (Figure 3).
The patient recovered completely and was discharged home with plans to return for outpatient imaging for fibromuscular dysplasia.
SCAD: RARE OR JUST RARELY RECOGNIZED?
SCAD appears to be a rare cause of acute coronary syndrome, but it is likely underdiagnosed and is becoming increasingly recognized worldwide. Typically, it affects women younger than 50, with women in general outnumbering men 9 to 1.3 Overall, SCAD causes up to 4% of acute myocardial infarctions, but in women age 50 or younger, it is responsible for 24% to 35% of acute myocardial infarctions, and the proportion is even higher in pregnant women.4
Not just pregnancy-associated
SCAD was previously thought to be mainly idiopathic and mostly affecting women peripartum. Current understanding paints a different picture: pregnancy-associated SCAD does not account for the majority of cases. That said, SCAD is the most common cause of myocardial infarction peripartum, with the third trimester and early postpartum period being the times of highest risk.5 SCAD development at those times is believed to be related to hormonal changes causing weakening of coronary artery walls.6
Weakening of the coronary artery wall also may occur in the setting of fibromuscular dysplasia, connective tissue disease, recurrent pregnancies, systemic inflammatory disease, hormonal therapy, and other disease states that cause arteriopathy. Exposure to a stressor in a patient with underlying risk factors can lead to either an intimal tear or rupture of the vasa vasorum, with subsequent formation of intramural hemorrhage and eventually SCAD.7 Stressors can be emotional or physical and can include labor and delivery, intense physical exercise, the Valsalva maneuver, and drug abuse.8
Presentation is variable
SCAD presentation depends on the degree of flow limitation and extent of the dissection. Presentation can range from asymptomatic to sudden cardiac death and can include signs and symptoms of acute coronary syndrome caused by ST-segment elevation or non-ST-segment elevation myocardial infarction.
DIAGNOSIS BY ANGIOGRAPHY
SCAD can be diagnosed by coronary angiography. There are 3 angiographic types:
Type 1 (about 25% of SCAD cases) has typical contrast dye staining of the arterial wall and multiple radiolucent luminal abnormalities, with or without dye hang-up.
Type 2 (about 70%) has diffuse, smooth narrowing of the coronary artery, with the left anterior descending artery the most frequently affected.8
Type 3 (about 5%) mimics atherosclerosis, with focal or tubular stenosis.9
Types 1 and 2 are usually easy to recognize. To diagnose type 2, intravenous nitroglycerin should first be administered to rule out coronary spasm.
Type 3 SCAD is more challenging to diagnose because its appearance on angiography is similar to that of atherosclerosis. For equivocal findings in any type, but especially in type 3, intravascular ultrasonography or optical coherence tomography can help.10 Optical coherence tomography is preferred because of superior image resolution, although ultrasonography offers better tissue penetration.11
MANAGE MOST CASES CONSERVATIVELY
Management algorithms for SCAD are available.8,12
The initial and most critical step is to make the correct diagnosis. Although the presentation of acute coronary syndrome caused by SCAD is often identical to that of atherosclerosis, the conditions have different pathophysiologies and thus require different management. Theoretically, systemic anticoagulation may worsen an intramural hemorrhage.
First-line therapy for most patients with SCAD is conservative management and close inpatient monitoring for 3 to 5 days.13 More aggressive management is indicated for any of the following:
- Left main or severe proximal 2-vessel dissection
- Hemodynamic instability
- Ongoing ischemic symptoms.
In a prospective cohort of 168 patients, 134 (80%) were initially treated conservatively; of those, in-hospital myocardial infarction recurred in 4.5%, a major cardiac event occurred within 2 years in 17%, and SCAD recurred in 13%.8
Observational data on patients with SCAD who had repeat angiography weeks to months after the initial event has shown that lesions heal in 70% to 97% of patients.12
WHEN TO CONSIDER AGGRESSIVE MANAGEMENT
Under the circumstances listed above, revascularization with PCI or coronary artery bypass grafting (CABG) should be considered, with choice of procedure determined by feasibility, technical considerations, and local expertise.
The American Heart Association recommendations are as follows12:
- For left main or severe proximal 2-vessel dissection in clinically stable patients, consider CABG
- For active ischemia or hemodynamic instability, consider PCI if feasible or perform urgent CABG.
A few series have shown that the prognosis with conservative management or CABG is better than with PCI.8,13,14 The success rate for revascularization with PCI is only about 60% because of challenges including risk of inducing iatrogenic dissection, passing the wire into the false lumen and worsening a dissection, and propagating an intramural hematoma with stenting and further compromising coronary blood flow. In addition, dissection tends to extend into distal arteries that are difficult to stent. There is also the risk of stent malapposition after resorption of the intramural hematoma, causing late stent thrombosis.7
SCREEN FOR OTHER VASCULAR PROBLEMS
Imaging of the renal, iliac, and cerebral vasculature is recommended for all patients with SCAD.12 Screening for fibromuscular dysplasia can be done with angiography, computed tomographic angiography (CTA), or magnetic resonance angiography (MRA).12
Multifocal fibromuscular dysplasia in extracoronary arteries occurs with SCAD in 25% to 86% of cases. In a single-center series of 115 patients with confirmed SCAD who underwent CTA from 2010 to 2014, extracoronary vascular abnormalities were found in 66%, with fibromuscular dysplasia being the most common type (45%).15 In another single-center study, 327 patients with SCAD were prospectively followed from 2012 to 2016 with screening for cerebrovascular, renal, and iliac fibromuscular dysplasia using CTA or catheter angiography. Fibromuscular dysplasia was found in 63%, and intracranial aneurysm was found in 14% of patients with fibromuscular dysplasia.9
SCAD can also be associated with connective tissue disorders such as Ehlers-Danlos syndrome type IV and Marfan syndrome.16,17
LONG-TERM MANAGEMENT
Patients with SCAD should start long-term aspirin and 1 year of clopidogrel. Statins are indicated for patients with hyperlipidemia8,18 but otherwise offer no clear benefit for SCAD alone. If there are no contraindications, a beta-adrenergic blocker should be considered, especially if left ventricular dysfunction or arrhythmias are present. Angiotensin-converting enzyme inhibitors and angiotensin II receptor blockers should also be considered with concomitant left ventricular dysfunction. Antianginal therapy can be used for post-SCAD chest pain syndromes.12
Repeat angiography is recommended only to evaluate recurrent symptoms, to confirm an unclear initial diagnosis, to assess for atherosclerosis-related stenosis, or to evaluate high-risk anatomy, eg, involvement of the left main coronary artery.12
Genetic testing is reserved for patients with a high clinical suspicion of connective tissue disease or systemic arteriopathy.19
A 55-year-old woman with hypertension presented with sudden onset of severe pressure-like chest pain that started when she was taking a bath. The pain radiated to her back and was associated with nausea.
A 12-lead electrocardiogram (Figure 1) showed ST-segment elevation of more than 2 mm in leads V2, V3, V4, and V5, with no reciprocal changes.
In view of her ongoing severe chest pain, she was given aspirin and ticagrelor and was taken for emergency cardiac catheterization. Left ventriculography showed hypokinesis in the mid-anterior, distal anterior, apical, and distal inferior chamber walls. Coronary angiography (Figure 2) revealed a long segment of diffuse, smooth narrowing of the mid-left anterior descending coronary artery that did not reverse after administration of intracoronary nitroglycerin.
Based on the classic angiographic appearance and the absence of atherosclerotic disease in other coronary arteries, type 2 spontaneous coronary artery dissection (SCAD) was diagnosed.
CORONARY ARTERY WALL SEPARATION
SCAD is defined as a nontraumatic, noniatrogenic intramural hemorrhage leading to separation of the coronary arterial wall and the formation of a false lumen. The separation can occur between any of the coronary artery wall layers and may or may not involve an intimal tear. The bleeding may result in an intramural hematoma and possible narrowing of the arterial lumen. Depending on the severity of narrowing, blood supply to the myocardium could be compromised, resulting in symptoms of ischemia.1
SCAD usually involves a single coronary artery, although multiple coronary artery involvement has been reported.2
CASE CONTINUED: MANAGEMENT
Conservative management is generally recommended for SCAD. An initial decision was made to continue medical management alone. But because the patient continued to have severe chest pain that was unresponsive to intravenous nitroglycerin and intravenous morphine and was accompanied by frequent episodes of nonsustained ventricular tachycardia during cardiac catheterization, the management team decided to proceed with percutaneous coronary intervention (PCI). Implantation of a 2.25-by-38-mm drug-eluting stent in the left anterior descending artery was successful, resulting in return of normal flow (Thrombolysis in Myocardial Infarction [TIMI] score 3) and only a small distal residual non-flow-limiting dissection (Figure 3).
The patient recovered completely and was discharged home with plans to return for outpatient imaging for fibromuscular dysplasia.
SCAD: RARE OR JUST RARELY RECOGNIZED?
SCAD appears to be a rare cause of acute coronary syndrome, but it is likely underdiagnosed and is becoming increasingly recognized worldwide. Typically, it affects women younger than 50, with women in general outnumbering men 9 to 1.3 Overall, SCAD causes up to 4% of acute myocardial infarctions, but in women age 50 or younger, it is responsible for 24% to 35% of acute myocardial infarctions, and the proportion is even higher in pregnant women.4
Not just pregnancy-associated
SCAD was previously thought to be mainly idiopathic and mostly affecting women peripartum. Current understanding paints a different picture: pregnancy-associated SCAD does not account for the majority of cases. That said, SCAD is the most common cause of myocardial infarction peripartum, with the third trimester and early postpartum period being the times of highest risk.5 SCAD development at those times is believed to be related to hormonal changes causing weakening of coronary artery walls.6
Weakening of the coronary artery wall also may occur in the setting of fibromuscular dysplasia, connective tissue disease, recurrent pregnancies, systemic inflammatory disease, hormonal therapy, and other disease states that cause arteriopathy. Exposure to a stressor in a patient with underlying risk factors can lead to either an intimal tear or rupture of the vasa vasorum, with subsequent formation of intramural hemorrhage and eventually SCAD.7 Stressors can be emotional or physical and can include labor and delivery, intense physical exercise, the Valsalva maneuver, and drug abuse.8
Presentation is variable
SCAD presentation depends on the degree of flow limitation and extent of the dissection. Presentation can range from asymptomatic to sudden cardiac death and can include signs and symptoms of acute coronary syndrome caused by ST-segment elevation or non-ST-segment elevation myocardial infarction.
DIAGNOSIS BY ANGIOGRAPHY
SCAD can be diagnosed by coronary angiography. There are 3 angiographic types:
Type 1 (about 25% of SCAD cases) has typical contrast dye staining of the arterial wall and multiple radiolucent luminal abnormalities, with or without dye hang-up.
Type 2 (about 70%) has diffuse, smooth narrowing of the coronary artery, with the left anterior descending artery the most frequently affected.8
Type 3 (about 5%) mimics atherosclerosis, with focal or tubular stenosis.9
Types 1 and 2 are usually easy to recognize. To diagnose type 2, intravenous nitroglycerin should first be administered to rule out coronary spasm.
Type 3 SCAD is more challenging to diagnose because its appearance on angiography is similar to that of atherosclerosis. For equivocal findings in any type, but especially in type 3, intravascular ultrasonography or optical coherence tomography can help.10 Optical coherence tomography is preferred because of superior image resolution, although ultrasonography offers better tissue penetration.11
MANAGE MOST CASES CONSERVATIVELY
Management algorithms for SCAD are available.8,12
The initial and most critical step is to make the correct diagnosis. Although the presentation of acute coronary syndrome caused by SCAD is often identical to that of atherosclerosis, the conditions have different pathophysiologies and thus require different management. Theoretically, systemic anticoagulation may worsen an intramural hemorrhage.
First-line therapy for most patients with SCAD is conservative management and close inpatient monitoring for 3 to 5 days.13 More aggressive management is indicated for any of the following:
- Left main or severe proximal 2-vessel dissection
- Hemodynamic instability
- Ongoing ischemic symptoms.
In a prospective cohort of 168 patients, 134 (80%) were initially treated conservatively; of those, in-hospital myocardial infarction recurred in 4.5%, a major cardiac event occurred within 2 years in 17%, and SCAD recurred in 13%.8
Observational data on patients with SCAD who had repeat angiography weeks to months after the initial event has shown that lesions heal in 70% to 97% of patients.12
WHEN TO CONSIDER AGGRESSIVE MANAGEMENT
Under the circumstances listed above, revascularization with PCI or coronary artery bypass grafting (CABG) should be considered, with choice of procedure determined by feasibility, technical considerations, and local expertise.
The American Heart Association recommendations are as follows12:
- For left main or severe proximal 2-vessel dissection in clinically stable patients, consider CABG
- For active ischemia or hemodynamic instability, consider PCI if feasible or perform urgent CABG.
A few series have shown that the prognosis with conservative management or CABG is better than with PCI.8,13,14 The success rate for revascularization with PCI is only about 60% because of challenges including risk of inducing iatrogenic dissection, passing the wire into the false lumen and worsening a dissection, and propagating an intramural hematoma with stenting and further compromising coronary blood flow. In addition, dissection tends to extend into distal arteries that are difficult to stent. There is also the risk of stent malapposition after resorption of the intramural hematoma, causing late stent thrombosis.7
SCREEN FOR OTHER VASCULAR PROBLEMS
Imaging of the renal, iliac, and cerebral vasculature is recommended for all patients with SCAD.12 Screening for fibromuscular dysplasia can be done with angiography, computed tomographic angiography (CTA), or magnetic resonance angiography (MRA).12
Multifocal fibromuscular dysplasia in extracoronary arteries occurs with SCAD in 25% to 86% of cases. In a single-center series of 115 patients with confirmed SCAD who underwent CTA from 2010 to 2014, extracoronary vascular abnormalities were found in 66%, with fibromuscular dysplasia being the most common type (45%).15 In another single-center study, 327 patients with SCAD were prospectively followed from 2012 to 2016 with screening for cerebrovascular, renal, and iliac fibromuscular dysplasia using CTA or catheter angiography. Fibromuscular dysplasia was found in 63%, and intracranial aneurysm was found in 14% of patients with fibromuscular dysplasia.9
SCAD can also be associated with connective tissue disorders such as Ehlers-Danlos syndrome type IV and Marfan syndrome.16,17
LONG-TERM MANAGEMENT
Patients with SCAD should start long-term aspirin and 1 year of clopidogrel. Statins are indicated for patients with hyperlipidemia8,18 but otherwise offer no clear benefit for SCAD alone. If there are no contraindications, a beta-adrenergic blocker should be considered, especially if left ventricular dysfunction or arrhythmias are present. Angiotensin-converting enzyme inhibitors and angiotensin II receptor blockers should also be considered with concomitant left ventricular dysfunction. Antianginal therapy can be used for post-SCAD chest pain syndromes.12
Repeat angiography is recommended only to evaluate recurrent symptoms, to confirm an unclear initial diagnosis, to assess for atherosclerosis-related stenosis, or to evaluate high-risk anatomy, eg, involvement of the left main coronary artery.12
Genetic testing is reserved for patients with a high clinical suspicion of connective tissue disease or systemic arteriopathy.19
- Garcia NA, Khan AN, Boppana RC, Smith HL. Spontaneous coronary artery dissection: a case series and literature review. J Community Hosp Intern Med Perspect 2014; 4(4). doi:10.3402/jchimp.v4.25261
- Lempereur M, Gin K, Saw J. Multivessel spontaneous coronary artery dissection mimicking atherosclerosis. JACC Cardiovasc Interv 2014; 7(7):e87–e88. doi:10.1016/j.jcin.2013.12.207
- Mahmoud AN, Taduru SS, Mentias A, et al. Trends of incidence, clinical presentation, and in-hospital mortality among women with acute myocardial infarction with or without spontaneous coronary artery dissection: a population-based analysis. JACC Cardiovasc Interv 2018; 11(1):80–90. doi:10.1016/j.jcin.2017.08.016
- Saw J. Pregnancy-associated spontaneous coronary artery dissection represents an exceptionally high-risk spontaneous coronary artery dissection cohort. Circ Cardiovasc Interv 2017; 10(3)pii:e005119. doi:10.1161/CIRCINTERVENTIONS.117.005119
- Elkayam U, Jalnapurkar S, Barakkat MN, et al. Pregnancy-associated acute myocardial infarction: a review of contemporary experience in 150 cases between 2006 and 2011. Circulation 2014; 129(16):1695–1702. doi:10.1161/CIRCULATIONAHA.113.002054
- Vijayaraghavan R, Verma S, Gupta N, Saw J. Pregnancy-related spontaneous coronary artery dissection. Circulation 2014; 130(21):1915–1920. doi:10.1161/CIRCULATIONAHA.114.011422
- Saw J, Mancini GBJ, Humphries KH. Contemporary review on spontaneous coronary artery dissection. J Am Coll Cardiol 2016; 68(3):297–312. doi:10.1016/j.jacc.2016.05.034
- Saw J, Aymong E, Sedlak T, et al. Spontaneous coronary artery dissection: association with predisposing arteriopathies and precipitating stressors and cardiovascular outcomes. Circ Cardiovasc Interv 2014; 7(5):645–655. doi:10.1161/CIRCINTERVENTIONS.114.001760
- Saw J, Humphries K ,Aymong E, et al. Spontaneous coronary artery dissection: clinical outcomes and risk of recurrence. J Am Coll Cardiol 2017; 70(9):1148–1158. doi:10.1016/j.jacc.2017.06.053
- Alfonso F, Bastante T, Cuesta J, Rodríguez D, Benedicto A, Rivero F. Spontaneous coronary artery dissection: novel insights on diagnosis and management. Cardiovasc Diagn Ther 2015; 5(2):133–140. doi:10.3978/j.issn.2223-3652.2015.03.05
- Kern MJ, Meier B. Evaluation of the culprit plaque and the physiological significance of coronary atherosclerotic narrowings. Circulation 2001; 103(25):3142–3149. pmid:11425782
- Hayes SN, Kim ESH, Saw J, et al; American Heart Association Council on Peripheral Vascular Disease; Council on Clinical Cardiology; Council on Cardiovascular and Stroke Nursing; Council on Genomic and Precision Medicine; and Stroke Council. Spontaneous coronary artery dissection: current state of the science: a scientific statement from the American Heart Association. Circulation 2018; 137(19):e523–e557. doi:10.1161/CIR.0000000000000564
- Tweet MS, Eleid MF, Best PJ, et al. Spontaneous coronary artery dissection: revascularization versus conservative therapy. Circ Cardiovasc Interv 2014; 7(6):777–786. doi:10.1161/CIRCINTERVENTIONS.114.001659
- Tweet MS, Hayes SN, Pitta SR, et al. Clinical features, management, and prognosis of spontaneous coronary artery dissection. Circulation 2012; 126(5):579–588. doi:10.1161/CIRCULATIONAHA.112.105718
- Prasad M, Tweet MS, Hayes SN, et al. Prevalence of extracoronary vascular abnormalities and fibromuscular dysplasia in patients with spontaneous coronary artery dissection. Am J Cardiol 2015; 115(12):1672–1677. doi:10.1016/j.amjcard.2015.03.011
- Adès LC, Waltham RD, Chiodo AA, Bateman JF. Myocardial infarction resulting from coronary artery dissection in an adolescent with Ehlers-Danlos syndrome type IV due to a type III collagen mutation. Br Heart J 1995; 74(2):112–116. pmid:7546986
- Judge DP, Dietz HC. Marfan’s syndrome. Lancet 2005; 366(9501):1965–1976. doi:10.1016/S0140-6736(05)67789-6
- Saw J. Spontaneous coronary artery dissection. Can J Cardiol 2013; 29(9):1027–1033. doi:10.1016/j.cjca.2012.12.018
- Poloskey SL, Kim ES, Sanghani R, et al. Low yield of genetic testing for known vascular connective tissue disorders in patients with fibromuscular dysplasia. Vasc Med 2012; 17(6):371–378. doi:10.1177/1358863X12459650
- Garcia NA, Khan AN, Boppana RC, Smith HL. Spontaneous coronary artery dissection: a case series and literature review. J Community Hosp Intern Med Perspect 2014; 4(4). doi:10.3402/jchimp.v4.25261
- Lempereur M, Gin K, Saw J. Multivessel spontaneous coronary artery dissection mimicking atherosclerosis. JACC Cardiovasc Interv 2014; 7(7):e87–e88. doi:10.1016/j.jcin.2013.12.207
- Mahmoud AN, Taduru SS, Mentias A, et al. Trends of incidence, clinical presentation, and in-hospital mortality among women with acute myocardial infarction with or without spontaneous coronary artery dissection: a population-based analysis. JACC Cardiovasc Interv 2018; 11(1):80–90. doi:10.1016/j.jcin.2017.08.016
- Saw J. Pregnancy-associated spontaneous coronary artery dissection represents an exceptionally high-risk spontaneous coronary artery dissection cohort. Circ Cardiovasc Interv 2017; 10(3)pii:e005119. doi:10.1161/CIRCINTERVENTIONS.117.005119
- Elkayam U, Jalnapurkar S, Barakkat MN, et al. Pregnancy-associated acute myocardial infarction: a review of contemporary experience in 150 cases between 2006 and 2011. Circulation 2014; 129(16):1695–1702. doi:10.1161/CIRCULATIONAHA.113.002054
- Vijayaraghavan R, Verma S, Gupta N, Saw J. Pregnancy-related spontaneous coronary artery dissection. Circulation 2014; 130(21):1915–1920. doi:10.1161/CIRCULATIONAHA.114.011422
- Saw J, Mancini GBJ, Humphries KH. Contemporary review on spontaneous coronary artery dissection. J Am Coll Cardiol 2016; 68(3):297–312. doi:10.1016/j.jacc.2016.05.034
- Saw J, Aymong E, Sedlak T, et al. Spontaneous coronary artery dissection: association with predisposing arteriopathies and precipitating stressors and cardiovascular outcomes. Circ Cardiovasc Interv 2014; 7(5):645–655. doi:10.1161/CIRCINTERVENTIONS.114.001760
- Saw J, Humphries K ,Aymong E, et al. Spontaneous coronary artery dissection: clinical outcomes and risk of recurrence. J Am Coll Cardiol 2017; 70(9):1148–1158. doi:10.1016/j.jacc.2017.06.053
- Alfonso F, Bastante T, Cuesta J, Rodríguez D, Benedicto A, Rivero F. Spontaneous coronary artery dissection: novel insights on diagnosis and management. Cardiovasc Diagn Ther 2015; 5(2):133–140. doi:10.3978/j.issn.2223-3652.2015.03.05
- Kern MJ, Meier B. Evaluation of the culprit plaque and the physiological significance of coronary atherosclerotic narrowings. Circulation 2001; 103(25):3142–3149. pmid:11425782
- Hayes SN, Kim ESH, Saw J, et al; American Heart Association Council on Peripheral Vascular Disease; Council on Clinical Cardiology; Council on Cardiovascular and Stroke Nursing; Council on Genomic and Precision Medicine; and Stroke Council. Spontaneous coronary artery dissection: current state of the science: a scientific statement from the American Heart Association. Circulation 2018; 137(19):e523–e557. doi:10.1161/CIR.0000000000000564
- Tweet MS, Eleid MF, Best PJ, et al. Spontaneous coronary artery dissection: revascularization versus conservative therapy. Circ Cardiovasc Interv 2014; 7(6):777–786. doi:10.1161/CIRCINTERVENTIONS.114.001659
- Tweet MS, Hayes SN, Pitta SR, et al. Clinical features, management, and prognosis of spontaneous coronary artery dissection. Circulation 2012; 126(5):579–588. doi:10.1161/CIRCULATIONAHA.112.105718
- Prasad M, Tweet MS, Hayes SN, et al. Prevalence of extracoronary vascular abnormalities and fibromuscular dysplasia in patients with spontaneous coronary artery dissection. Am J Cardiol 2015; 115(12):1672–1677. doi:10.1016/j.amjcard.2015.03.011
- Adès LC, Waltham RD, Chiodo AA, Bateman JF. Myocardial infarction resulting from coronary artery dissection in an adolescent with Ehlers-Danlos syndrome type IV due to a type III collagen mutation. Br Heart J 1995; 74(2):112–116. pmid:7546986
- Judge DP, Dietz HC. Marfan’s syndrome. Lancet 2005; 366(9501):1965–1976. doi:10.1016/S0140-6736(05)67789-6
- Saw J. Spontaneous coronary artery dissection. Can J Cardiol 2013; 29(9):1027–1033. doi:10.1016/j.cjca.2012.12.018
- Poloskey SL, Kim ES, Sanghani R, et al. Low yield of genetic testing for known vascular connective tissue disorders in patients with fibromuscular dysplasia. Vasc Med 2012; 17(6):371–378. doi:10.1177/1358863X12459650
KEY POINTS
- SCAD often presents with symptoms of acute coronary syndrome but can be asymptomatic or cause sudden death.
- Management is generally conservative, but a left main or severe proximal 2-vessel dissection, hemodynamic instability, or ongoing ischemic symptoms may warrant revascularization.
- All patients with SCAD should be screened for other vascular problems, especially fibromuscular dysplasia.
- Long-term aspirin therapy and 1 year of clopidogrel are recommended after an episode of SCAD.
Gastroparesis in a patient with diabetic ketoacidosis
A 40-year-old man with type 1 diabetes mellitus and recurrent renal calculi presented to the emergency department with nausea, vomiting, and abdominal pain for the past day. He had been checking his blood glucose level regularly, and it had usually been within the normal range until 2 or 3 days previously, when he stopped taking his insulin because he ran out and could not afford to buy more.
He said he initially vomited clear mucus but then had 2 episodes of black vomit. His abdominal pain was diffuse but more intense in his flanks. He said he had never had nausea or vomiting before this episode.
In the emergency department, his heart rate was 136 beats per minute and respiratory rate 24 breaths per minute. He appeared to be in mild distress, and physical examination revealed a distended abdomen, decreased bowel sounds on auscultation, tympanic sound elicited by percussion, and diffuse abdominal tenderness to palpation without rebound tenderness or rigidity. His blood glucose level was 993 mg/dL, and his anion gap was 36 mmol/L.
Computed tomography (CT) showed new severe gastric distention; a scan 11 months previously to look for renal stones had been normal (Figure 1). The patient’s presentation, physical examination, and laboratory and radiographic investigations narrowed the working diagnosis to gastric outlet obstruction or acute gastroparesis, but since CT showed no obstructing mass, the diagnosis of acute gastroparesis that coexisted with diabetic ketoacidosis was more likely.
The patient was treated with hydration, insulin, and a nasogastric tube to relieve the pressure. The following day, his symptoms had significantly improved, his abdomen was less distended, his bowel sounds had returned, and his plasma glucose levels were in the normal range. The nasogastric tube was removed after he started to have bowel movements; he was given liquids by mouth and eventually solid food. Since his condition had significantly improved and he had started to have bowel movements, no follow-up imaging was done. The next day, he was symptom-free, his laboratory values were normal, and he was discharged home.
GASTROPARESIS
Gastroparesis is defined by delayed gastric emptying in the absence of a mechanical obstruction, with symptoms of nausea, vomiting, bloating, and abdominal pain. Most commonly it is idiopathic or caused by long-standing uncontrolled diabetes.
Diabetic gastroparesis is thought to result from impaired neural control of gastric function. Damage to the pacemaker interstitial cells of Cajal and underlying smooth muscle may be contributing factors.1 It is usually chronic, with a mean duration of symptoms of 26.5 months.2 However, acute gastroparesis can occur after an acute elevation in the plasma glucose concentration, which can affect gastric sensory and motor function3 via relaxation of the proximal stomach, decrease in antral pressure waves, and increase in pyloric pressure waves.4
Patients with diabetic ketoacidosis often present with symptoms similar to those of gastroparesis, including nausea, vomiting, and abdominal pain.5 But acute gastroparesis can coexist with diabetic ketoacidosis, as in our patient, and the gastroparesis can go undiagnosed, since imaging studies are not routinely done for diabetic ketoacidosis unless there is another reason—as in our patient.
More study is needed to answer questions on long-term outcomes for patients presenting with acute gastroparesis: Do they develop chronic gastroparesis? And is there is a correlation with progression of neuropathy?
The diagnosis usually requires a high level of suspicion in patients with nausea, vomiting, fullness, abdominal pain, and bloating; exclusion of gastric outlet obstruction by a mass or antral stenosis; and evidence of delayed gastric emptying. Gastric outlet obstruction can be ruled out by endoscopy, abdominal CT, or magnetic resonance enterography. Delayed gastric emptying can be quantified with scintigraphy and endoscopy. In our patient, gastroparesis was diagnosed on the basis of the clinical symptoms and CT findings.
Treatment is usually directed at symptoms, with better glycemic control and dietary modification for moderate cases, and prokinetics and a gastrostomy tube for severe cases.
TAKE-HOME POINTS
- Gastroparesis is usually chronic but can present acutely with acute severe hyperglycemia.
- Gastrointestinal tract motor function is affected by plasma glucose levels and can change over brief intervals.
- Diabetic ketoacidosis symptoms can mask acute gastroparesis, as imaging studies are not routinely done.
- Acute gastroparesis can be diagnosed clinically along with abdominal CT or endoscopy to rule out gastric outlet obstruction.
- Acute gastroparesis caused by diabetic ketoacidosis can resolve promptly with tight control of plasma glucose levels, anion gap closing, and nasogastric tube placement.
- Parkman HP, Hasler WL, Fisher RS; American Gastroenterological Association. American Gastroenterological Association technical review on the diagnosis and treatment of gastroparesis. Gastroenterology 2004; 127(5):1592–1622. pmid:15521026
- Dudekula A, O’Connell M, Bielefeldt K. Hospitalizations and testing in gastroparesis. J Gastroenterol Hepatol 2011; 26(8):1275–1282. doi:10.1111/j.1440-1746.2011.06735.x
- Fraser RJ, Horowitz M, Maddox AF, Harding PE, Chatterton BE, Dent J. Hyperglycaemia slows gastric emptying in type 1 (insulin-dependent) diabetes mellitus. Diabetologia 1990; 33(11):675–680. pmid:2076799
- Mearin F, Malagelada JR. Gastroparesis and dyspepsia in patients with diabetes mellitus. Eur J Gastroenterol Hepatol 1995; 7(8):717–723. pmid:7496857
- Malone ML, Gennis V, Goodwin JS. Characteristics of diabetic ketoacidosis in older versus younger adults. J Am Geriatr Soc 1992; 40(11):1100–1104. pmid:1401693
A 40-year-old man with type 1 diabetes mellitus and recurrent renal calculi presented to the emergency department with nausea, vomiting, and abdominal pain for the past day. He had been checking his blood glucose level regularly, and it had usually been within the normal range until 2 or 3 days previously, when he stopped taking his insulin because he ran out and could not afford to buy more.
He said he initially vomited clear mucus but then had 2 episodes of black vomit. His abdominal pain was diffuse but more intense in his flanks. He said he had never had nausea or vomiting before this episode.
In the emergency department, his heart rate was 136 beats per minute and respiratory rate 24 breaths per minute. He appeared to be in mild distress, and physical examination revealed a distended abdomen, decreased bowel sounds on auscultation, tympanic sound elicited by percussion, and diffuse abdominal tenderness to palpation without rebound tenderness or rigidity. His blood glucose level was 993 mg/dL, and his anion gap was 36 mmol/L.
Computed tomography (CT) showed new severe gastric distention; a scan 11 months previously to look for renal stones had been normal (Figure 1). The patient’s presentation, physical examination, and laboratory and radiographic investigations narrowed the working diagnosis to gastric outlet obstruction or acute gastroparesis, but since CT showed no obstructing mass, the diagnosis of acute gastroparesis that coexisted with diabetic ketoacidosis was more likely.
The patient was treated with hydration, insulin, and a nasogastric tube to relieve the pressure. The following day, his symptoms had significantly improved, his abdomen was less distended, his bowel sounds had returned, and his plasma glucose levels were in the normal range. The nasogastric tube was removed after he started to have bowel movements; he was given liquids by mouth and eventually solid food. Since his condition had significantly improved and he had started to have bowel movements, no follow-up imaging was done. The next day, he was symptom-free, his laboratory values were normal, and he was discharged home.
GASTROPARESIS
Gastroparesis is defined by delayed gastric emptying in the absence of a mechanical obstruction, with symptoms of nausea, vomiting, bloating, and abdominal pain. Most commonly it is idiopathic or caused by long-standing uncontrolled diabetes.
Diabetic gastroparesis is thought to result from impaired neural control of gastric function. Damage to the pacemaker interstitial cells of Cajal and underlying smooth muscle may be contributing factors.1 It is usually chronic, with a mean duration of symptoms of 26.5 months.2 However, acute gastroparesis can occur after an acute elevation in the plasma glucose concentration, which can affect gastric sensory and motor function3 via relaxation of the proximal stomach, decrease in antral pressure waves, and increase in pyloric pressure waves.4
Patients with diabetic ketoacidosis often present with symptoms similar to those of gastroparesis, including nausea, vomiting, and abdominal pain.5 But acute gastroparesis can coexist with diabetic ketoacidosis, as in our patient, and the gastroparesis can go undiagnosed, since imaging studies are not routinely done for diabetic ketoacidosis unless there is another reason—as in our patient.
More study is needed to answer questions on long-term outcomes for patients presenting with acute gastroparesis: Do they develop chronic gastroparesis? And is there is a correlation with progression of neuropathy?
The diagnosis usually requires a high level of suspicion in patients with nausea, vomiting, fullness, abdominal pain, and bloating; exclusion of gastric outlet obstruction by a mass or antral stenosis; and evidence of delayed gastric emptying. Gastric outlet obstruction can be ruled out by endoscopy, abdominal CT, or magnetic resonance enterography. Delayed gastric emptying can be quantified with scintigraphy and endoscopy. In our patient, gastroparesis was diagnosed on the basis of the clinical symptoms and CT findings.
Treatment is usually directed at symptoms, with better glycemic control and dietary modification for moderate cases, and prokinetics and a gastrostomy tube for severe cases.
TAKE-HOME POINTS
- Gastroparesis is usually chronic but can present acutely with acute severe hyperglycemia.
- Gastrointestinal tract motor function is affected by plasma glucose levels and can change over brief intervals.
- Diabetic ketoacidosis symptoms can mask acute gastroparesis, as imaging studies are not routinely done.
- Acute gastroparesis can be diagnosed clinically along with abdominal CT or endoscopy to rule out gastric outlet obstruction.
- Acute gastroparesis caused by diabetic ketoacidosis can resolve promptly with tight control of plasma glucose levels, anion gap closing, and nasogastric tube placement.
A 40-year-old man with type 1 diabetes mellitus and recurrent renal calculi presented to the emergency department with nausea, vomiting, and abdominal pain for the past day. He had been checking his blood glucose level regularly, and it had usually been within the normal range until 2 or 3 days previously, when he stopped taking his insulin because he ran out and could not afford to buy more.
He said he initially vomited clear mucus but then had 2 episodes of black vomit. His abdominal pain was diffuse but more intense in his flanks. He said he had never had nausea or vomiting before this episode.
In the emergency department, his heart rate was 136 beats per minute and respiratory rate 24 breaths per minute. He appeared to be in mild distress, and physical examination revealed a distended abdomen, decreased bowel sounds on auscultation, tympanic sound elicited by percussion, and diffuse abdominal tenderness to palpation without rebound tenderness or rigidity. His blood glucose level was 993 mg/dL, and his anion gap was 36 mmol/L.
Computed tomography (CT) showed new severe gastric distention; a scan 11 months previously to look for renal stones had been normal (Figure 1). The patient’s presentation, physical examination, and laboratory and radiographic investigations narrowed the working diagnosis to gastric outlet obstruction or acute gastroparesis, but since CT showed no obstructing mass, the diagnosis of acute gastroparesis that coexisted with diabetic ketoacidosis was more likely.
The patient was treated with hydration, insulin, and a nasogastric tube to relieve the pressure. The following day, his symptoms had significantly improved, his abdomen was less distended, his bowel sounds had returned, and his plasma glucose levels were in the normal range. The nasogastric tube was removed after he started to have bowel movements; he was given liquids by mouth and eventually solid food. Since his condition had significantly improved and he had started to have bowel movements, no follow-up imaging was done. The next day, he was symptom-free, his laboratory values were normal, and he was discharged home.
GASTROPARESIS
Gastroparesis is defined by delayed gastric emptying in the absence of a mechanical obstruction, with symptoms of nausea, vomiting, bloating, and abdominal pain. Most commonly it is idiopathic or caused by long-standing uncontrolled diabetes.
Diabetic gastroparesis is thought to result from impaired neural control of gastric function. Damage to the pacemaker interstitial cells of Cajal and underlying smooth muscle may be contributing factors.1 It is usually chronic, with a mean duration of symptoms of 26.5 months.2 However, acute gastroparesis can occur after an acute elevation in the plasma glucose concentration, which can affect gastric sensory and motor function3 via relaxation of the proximal stomach, decrease in antral pressure waves, and increase in pyloric pressure waves.4
Patients with diabetic ketoacidosis often present with symptoms similar to those of gastroparesis, including nausea, vomiting, and abdominal pain.5 But acute gastroparesis can coexist with diabetic ketoacidosis, as in our patient, and the gastroparesis can go undiagnosed, since imaging studies are not routinely done for diabetic ketoacidosis unless there is another reason—as in our patient.
More study is needed to answer questions on long-term outcomes for patients presenting with acute gastroparesis: Do they develop chronic gastroparesis? And is there is a correlation with progression of neuropathy?
The diagnosis usually requires a high level of suspicion in patients with nausea, vomiting, fullness, abdominal pain, and bloating; exclusion of gastric outlet obstruction by a mass or antral stenosis; and evidence of delayed gastric emptying. Gastric outlet obstruction can be ruled out by endoscopy, abdominal CT, or magnetic resonance enterography. Delayed gastric emptying can be quantified with scintigraphy and endoscopy. In our patient, gastroparesis was diagnosed on the basis of the clinical symptoms and CT findings.
Treatment is usually directed at symptoms, with better glycemic control and dietary modification for moderate cases, and prokinetics and a gastrostomy tube for severe cases.
TAKE-HOME POINTS
- Gastroparesis is usually chronic but can present acutely with acute severe hyperglycemia.
- Gastrointestinal tract motor function is affected by plasma glucose levels and can change over brief intervals.
- Diabetic ketoacidosis symptoms can mask acute gastroparesis, as imaging studies are not routinely done.
- Acute gastroparesis can be diagnosed clinically along with abdominal CT or endoscopy to rule out gastric outlet obstruction.
- Acute gastroparesis caused by diabetic ketoacidosis can resolve promptly with tight control of plasma glucose levels, anion gap closing, and nasogastric tube placement.
- Parkman HP, Hasler WL, Fisher RS; American Gastroenterological Association. American Gastroenterological Association technical review on the diagnosis and treatment of gastroparesis. Gastroenterology 2004; 127(5):1592–1622. pmid:15521026
- Dudekula A, O’Connell M, Bielefeldt K. Hospitalizations and testing in gastroparesis. J Gastroenterol Hepatol 2011; 26(8):1275–1282. doi:10.1111/j.1440-1746.2011.06735.x
- Fraser RJ, Horowitz M, Maddox AF, Harding PE, Chatterton BE, Dent J. Hyperglycaemia slows gastric emptying in type 1 (insulin-dependent) diabetes mellitus. Diabetologia 1990; 33(11):675–680. pmid:2076799
- Mearin F, Malagelada JR. Gastroparesis and dyspepsia in patients with diabetes mellitus. Eur J Gastroenterol Hepatol 1995; 7(8):717–723. pmid:7496857
- Malone ML, Gennis V, Goodwin JS. Characteristics of diabetic ketoacidosis in older versus younger adults. J Am Geriatr Soc 1992; 40(11):1100–1104. pmid:1401693
- Parkman HP, Hasler WL, Fisher RS; American Gastroenterological Association. American Gastroenterological Association technical review on the diagnosis and treatment of gastroparesis. Gastroenterology 2004; 127(5):1592–1622. pmid:15521026
- Dudekula A, O’Connell M, Bielefeldt K. Hospitalizations and testing in gastroparesis. J Gastroenterol Hepatol 2011; 26(8):1275–1282. doi:10.1111/j.1440-1746.2011.06735.x
- Fraser RJ, Horowitz M, Maddox AF, Harding PE, Chatterton BE, Dent J. Hyperglycaemia slows gastric emptying in type 1 (insulin-dependent) diabetes mellitus. Diabetologia 1990; 33(11):675–680. pmid:2076799
- Mearin F, Malagelada JR. Gastroparesis and dyspepsia in patients with diabetes mellitus. Eur J Gastroenterol Hepatol 1995; 7(8):717–723. pmid:7496857
- Malone ML, Gennis V, Goodwin JS. Characteristics of diabetic ketoacidosis in older versus younger adults. J Am Geriatr Soc 1992; 40(11):1100–1104. pmid:1401693
Is neuroimaging necessary to evaluate syncope?
A 40-year-old woman with a history of hypertension, who was recently started on a diuretic, presents to the emergency department after a witnessed syncopal event. She reports a prodrome of lightheadedness, nausea, and darkening of her vision that occurred a few seconds after standing, followed by loss of consciousness. She had a complete, spontaneous recovery after 10 seconds, but upon arousal she noticed she had lost bladder control.
Her blood pressure is 120/80 mm Hg supine, 110/70 mm Hg sitting, and 90/60 mm Hg standing. She has no focal neurologic deficits. The cardiac examination is normal, without murmurs, and electrocardiography shows sinus tachycardia (heart rate 110 bpm) without other abnormalities. Results of laboratory testing are unremarkable.
Should you order neuroimaging to evaluate for syncope?
DEFINITIONS, CLASSIFICATIONS
Syncope is an abrupt loss of consciousness due to transient global cerebral hypoperfusion, with a concomitant loss of postural tone and rapid, spontaneous recovery.1 Recovery from syncope is characterized by immediate restoration of orientation and normal behavior, although the period after recovery may be accompanied by fatigue.2
The European Society of Cardiology2 has classified syncope into 3 main categories: reflex (neurally mediated) syncope, syncope due to orthostatic hypotension, and cardiac syncope. Determining the cause is critical, as this determines the prognosis.
KEYS TO THE EVALUATION
According to the 2017 American College of Cardiology/American Heart Association (ACC/AHA) and the 2009 European Society of Cardiology guidelines, the evaluation of syncope should include a thorough history, taken from the patient and witnesses, and a complete physical examination. This can identify the cause of syncope in up to 50% of cases and differentiate between cardiac and noncardiac causes. Features that point to cardiac syncope include age older than 60, male sex, known heart disease, brief prodrome, syncope during exertion or when supine, first syncopal event, family history of sudden cardiac death, and abnormal physical examination.1
Features that suggest noncardiac syncope are young age; syncope only when standing; recurrent syncope; a prodrome of nausea, vomiting, and a warm sensation; and triggers such as dehydration, pain, distressful stimulus, cough, laugh micturition, defecation, and swallowing.1
Electrocardiography should follow the history and physical examination. When done at presentation, electrocardiography is diagnostic in only about 5% of cases. However, given the importance of the diagnosis, it remains an essential part of the initial evaluation of syncope.3
If a clear cause of syncope is identified at this point, no further workup is needed, and the cause of syncope should be addressed.1 If the cause is still unclear, the ACC/AHA guidelines recommend further evaluation based on the clinical presentation and risk stratification.
WHEN TO PURSUE ADDITIONAL TESTING
Routine use of additional testing is costly; tests should be ordered on the basis of their potential diagnostic and prognostic value. Additional evaluation should follow a stepwise approach and can include targeted blood work, autonomic nerve evaluation, tilt-table testing, transthoracic echocardiography, stress testing, electrocardiographic monitoring, and electrophysiologic testing.1
Syncope is rarely a manifestation of neurologic disease, yet 11% to 58% of patients with a first episode of uncomplicated syncope undergo extensive neuroimaging with magnetic resonance imaging, computed tomography, electroencephalography (EEG), and carotid ultrasonography.4 Evidence suggests that routine neurologic testing is of limited value given its low diagnostic yield and high cost.
Epilepsy is the most common neurologic cause of loss of consciousness but is estimated to account for less than 5% of patients with syncope.5 A thorough and thoughtful neurologic history and examination is often enough to distinguish between syncope, convulsive syncope, epileptic convulsions, and pseudosyncope.
In syncope, the loss of consciousness usually occurs 30 seconds to several minutes after standing. It presents with or without a prodrome (warmth, palpitations, and diaphoresis) and can be relieved with supine positioning. True loss of consciousness usually lasts less than a minute and is accompanied by loss of postural tone, with little or no fatigue in the recovery period.6
Conversely, in convulsive syncope, the prodrome can include pallor and diaphoresis. Loss of consciousness lasts about 30 seconds but is accompanied by fixed gaze, upward eye deviation, nuchal rigidity, tonic spasms, myoclonic jerks, tonic-clonic convulsions, and oral automatisms.6
Pseudosyncope is characterized by a prodrome of lightheadedness, shortness of breath, chest pain, and tingling sensations, followed by episodes of apparent loss of consciousness that last longer than several minutes and occur multiple times a day. During these episodes, patients purposefully try to avoid trauma when they lose consciousness, and almost always keep their eyes closed, in contrast to syncopal episodes, when the eyes are open and glassy.7
ROLE OF ELECTROENCEPHALOGRAPHY
If the diagnosis remains unclear after the history and neurologic examination, EEG is recommended (class IIa, ie, reasonable, can be useful) during tilt-table testing, as it can help differentiate syncope, pseudosyncope, and epilepsy.1
In an epileptic convulsion, EEG shows epileptiform discharges, whereas in syncope, it shows diffuse brainwave slowing with delta waves and a flatline pattern. In pseudosyncope and psychogenic nonepileptic seizures, EEG shows normal activity.8
Routine EEG is not recommended if there are no specific neurologic signs of epilepsy or if the history and neurologic examination indicate syncope or pseudosyncope.1
Structural brain disease does not typically present with transient global cerebral hypoperfusion resulting in syncope, so magnetic resonance imaging and computed tomography have a low diagnostic yield. Studies have revealed that for the 11% to 58% of patients who undergo neuroimaging, it establishes a diagnosis in only 0.2% to 1%.9 For this reason and in view of their high cost, these imaging tests should not be routinely ordered in the evaluation of syncope.4,10 Similarly, carotid artery imaging should not be routinely ordered if there is no focal neurologic finding suggesting unilateral ischemia.10
CASE CONTINUED
In our 40-year-old patient, the history suggests dehydration, as she recently started taking a diuretic. Thus, laboratory testing is reasonable.
Loss of bladder control is often interpreted as a red flag for neurologic disease, but syncope can often present with urinary incontinence. Urinary incontinence may also occur in epileptic seizure and in nonepileptic events such as syncope. A pooled analysis by Brigo et al11 determined that urinary incontinence had no value in distinguishing between epilepsy and syncope. Therefore, this physical finding should not incline the clinician to one diagnosis or the other.
Given our patient’s presentation, findings on physical examination, and absence of focal neurologic deficits, she should not undergo neuroimaging for syncope evaluation. The more likely cause of her syncope is orthostatic intolerance (orthostatic hypotension or vasovagal syncope) in the setting of intravascular volume depletion, likely secondary to diuretic use. Obtaining orthostatic vital signs is mandatory, and this confirms the diagnosis.
- Shen WK, Sheldon RS, Benditt DG, et al. 2017 ACC/AHA/HRS guideline for the evaluation and management of patients with syncope: a report of the American College of Cardiology/American Heart Association Task Force on Clinical Practice Guidelines and the Heart Rhythm Society. J Am Coll Cardiol 2017; 70(5):e39–e110. doi:10.1016/j.jacc.2017.03.003
- Task Force for the Diagnosis and Management of Syncope; European Society of Cardiology (ESC); European Heart Rhythm Association (EHRA); Heart Failure Association (HFA); Heart Rhythm Society (HRS), Moya A, Sutton R, Ammirati F, et al. Guidelines for the diagnosis and management of syncope (version 2009). Eur Heart J 2009; 30(21):2631–2671. doi:10.1093/eurheartj/ehp298
- Mehlsen J, Kaijer MN, Mehlsen AB. Autonomic and electrocardiographic changes in cardioinhibitory syncope. Europace 2008; 10(1):91–95. doi:10.1093/europace/eum237
- Goyal N, Donnino MW, Vachhani R, Bajwa R, Ahmad T, Otero R. The utility of head computed tomography in the emergency department evaluation of syncope. Intern Emerg Med 2006; 1(2):148–150. pmid:17111790
- Kapoor WN, Karpf M, Wieand S, Peterson JR, Levey GS. A prospective evaluation and follow-up of patients with syncope. N Engl J Med 1983; 309(4):197–204. doi:10.1056/NEJM198307283090401
- Sheldon R. How to differentiate syncope from seizure. Cardiol Clin 2015; 33(3):377–385. doi:10.1016/j.ccl.2015.04.006
- Raj V, Rowe AA, Fleisch SB, Paranjape SY, Arain AM, Nicolson SE. Psychogenic pseudosyncope: diagnosis and management. Auton Neurosci 2014; 184:66–72. doi:10.1016/j.autneu.2014.05.003
- Mecarelli O, Pulitano P, Vicenzini E, Vanacore N, Accornero N, De Marinis M. Observations on EEG patterns in neurally-mediated syncope: an inspective and quantitative study. Neurophysiol Clin 2004; 34(5):203–207. doi:10.1016/j.neucli.2004.09.004
- Johnson PC, Ammar H, Zohdy W, Fouda R, Govindu R. Yield of diagnostic tests and its impact on cost in adult patients with syncope presenting to a community hospital. South Med J 2014; 107(11):707–714. doi:10.14423/SMJ.0000000000000184
- Sclafani JJ, My J, Zacher LL, Eckart RE. Intensive education on evidence-based evaluation of syncope increases sudden death risk stratification but fails to reduce use of neuroimaging. Arch Intern Med 2010; 170(13):1150–1154. doi:10.1001/archinternmed.2010.205
- Brigo F, Nardone R Ausserer H, et al. The diagnostic value of urinary incontinence in the differential diagnosis of seizures. Seizure 2013; 22(2):85–90. doi:10.1016/j.seizure.2012.10.011
A 40-year-old woman with a history of hypertension, who was recently started on a diuretic, presents to the emergency department after a witnessed syncopal event. She reports a prodrome of lightheadedness, nausea, and darkening of her vision that occurred a few seconds after standing, followed by loss of consciousness. She had a complete, spontaneous recovery after 10 seconds, but upon arousal she noticed she had lost bladder control.
Her blood pressure is 120/80 mm Hg supine, 110/70 mm Hg sitting, and 90/60 mm Hg standing. She has no focal neurologic deficits. The cardiac examination is normal, without murmurs, and electrocardiography shows sinus tachycardia (heart rate 110 bpm) without other abnormalities. Results of laboratory testing are unremarkable.
Should you order neuroimaging to evaluate for syncope?
DEFINITIONS, CLASSIFICATIONS
Syncope is an abrupt loss of consciousness due to transient global cerebral hypoperfusion, with a concomitant loss of postural tone and rapid, spontaneous recovery.1 Recovery from syncope is characterized by immediate restoration of orientation and normal behavior, although the period after recovery may be accompanied by fatigue.2
The European Society of Cardiology2 has classified syncope into 3 main categories: reflex (neurally mediated) syncope, syncope due to orthostatic hypotension, and cardiac syncope. Determining the cause is critical, as this determines the prognosis.
KEYS TO THE EVALUATION
According to the 2017 American College of Cardiology/American Heart Association (ACC/AHA) and the 2009 European Society of Cardiology guidelines, the evaluation of syncope should include a thorough history, taken from the patient and witnesses, and a complete physical examination. This can identify the cause of syncope in up to 50% of cases and differentiate between cardiac and noncardiac causes. Features that point to cardiac syncope include age older than 60, male sex, known heart disease, brief prodrome, syncope during exertion or when supine, first syncopal event, family history of sudden cardiac death, and abnormal physical examination.1
Features that suggest noncardiac syncope are young age; syncope only when standing; recurrent syncope; a prodrome of nausea, vomiting, and a warm sensation; and triggers such as dehydration, pain, distressful stimulus, cough, laugh micturition, defecation, and swallowing.1
Electrocardiography should follow the history and physical examination. When done at presentation, electrocardiography is diagnostic in only about 5% of cases. However, given the importance of the diagnosis, it remains an essential part of the initial evaluation of syncope.3
If a clear cause of syncope is identified at this point, no further workup is needed, and the cause of syncope should be addressed.1 If the cause is still unclear, the ACC/AHA guidelines recommend further evaluation based on the clinical presentation and risk stratification.
WHEN TO PURSUE ADDITIONAL TESTING
Routine use of additional testing is costly; tests should be ordered on the basis of their potential diagnostic and prognostic value. Additional evaluation should follow a stepwise approach and can include targeted blood work, autonomic nerve evaluation, tilt-table testing, transthoracic echocardiography, stress testing, electrocardiographic monitoring, and electrophysiologic testing.1
Syncope is rarely a manifestation of neurologic disease, yet 11% to 58% of patients with a first episode of uncomplicated syncope undergo extensive neuroimaging with magnetic resonance imaging, computed tomography, electroencephalography (EEG), and carotid ultrasonography.4 Evidence suggests that routine neurologic testing is of limited value given its low diagnostic yield and high cost.
Epilepsy is the most common neurologic cause of loss of consciousness but is estimated to account for less than 5% of patients with syncope.5 A thorough and thoughtful neurologic history and examination is often enough to distinguish between syncope, convulsive syncope, epileptic convulsions, and pseudosyncope.
In syncope, the loss of consciousness usually occurs 30 seconds to several minutes after standing. It presents with or without a prodrome (warmth, palpitations, and diaphoresis) and can be relieved with supine positioning. True loss of consciousness usually lasts less than a minute and is accompanied by loss of postural tone, with little or no fatigue in the recovery period.6
Conversely, in convulsive syncope, the prodrome can include pallor and diaphoresis. Loss of consciousness lasts about 30 seconds but is accompanied by fixed gaze, upward eye deviation, nuchal rigidity, tonic spasms, myoclonic jerks, tonic-clonic convulsions, and oral automatisms.6
Pseudosyncope is characterized by a prodrome of lightheadedness, shortness of breath, chest pain, and tingling sensations, followed by episodes of apparent loss of consciousness that last longer than several minutes and occur multiple times a day. During these episodes, patients purposefully try to avoid trauma when they lose consciousness, and almost always keep their eyes closed, in contrast to syncopal episodes, when the eyes are open and glassy.7
ROLE OF ELECTROENCEPHALOGRAPHY
If the diagnosis remains unclear after the history and neurologic examination, EEG is recommended (class IIa, ie, reasonable, can be useful) during tilt-table testing, as it can help differentiate syncope, pseudosyncope, and epilepsy.1
In an epileptic convulsion, EEG shows epileptiform discharges, whereas in syncope, it shows diffuse brainwave slowing with delta waves and a flatline pattern. In pseudosyncope and psychogenic nonepileptic seizures, EEG shows normal activity.8
Routine EEG is not recommended if there are no specific neurologic signs of epilepsy or if the history and neurologic examination indicate syncope or pseudosyncope.1
Structural brain disease does not typically present with transient global cerebral hypoperfusion resulting in syncope, so magnetic resonance imaging and computed tomography have a low diagnostic yield. Studies have revealed that for the 11% to 58% of patients who undergo neuroimaging, it establishes a diagnosis in only 0.2% to 1%.9 For this reason and in view of their high cost, these imaging tests should not be routinely ordered in the evaluation of syncope.4,10 Similarly, carotid artery imaging should not be routinely ordered if there is no focal neurologic finding suggesting unilateral ischemia.10
CASE CONTINUED
In our 40-year-old patient, the history suggests dehydration, as she recently started taking a diuretic. Thus, laboratory testing is reasonable.
Loss of bladder control is often interpreted as a red flag for neurologic disease, but syncope can often present with urinary incontinence. Urinary incontinence may also occur in epileptic seizure and in nonepileptic events such as syncope. A pooled analysis by Brigo et al11 determined that urinary incontinence had no value in distinguishing between epilepsy and syncope. Therefore, this physical finding should not incline the clinician to one diagnosis or the other.
Given our patient’s presentation, findings on physical examination, and absence of focal neurologic deficits, she should not undergo neuroimaging for syncope evaluation. The more likely cause of her syncope is orthostatic intolerance (orthostatic hypotension or vasovagal syncope) in the setting of intravascular volume depletion, likely secondary to diuretic use. Obtaining orthostatic vital signs is mandatory, and this confirms the diagnosis.
A 40-year-old woman with a history of hypertension, who was recently started on a diuretic, presents to the emergency department after a witnessed syncopal event. She reports a prodrome of lightheadedness, nausea, and darkening of her vision that occurred a few seconds after standing, followed by loss of consciousness. She had a complete, spontaneous recovery after 10 seconds, but upon arousal she noticed she had lost bladder control.
Her blood pressure is 120/80 mm Hg supine, 110/70 mm Hg sitting, and 90/60 mm Hg standing. She has no focal neurologic deficits. The cardiac examination is normal, without murmurs, and electrocardiography shows sinus tachycardia (heart rate 110 bpm) without other abnormalities. Results of laboratory testing are unremarkable.
Should you order neuroimaging to evaluate for syncope?
DEFINITIONS, CLASSIFICATIONS
Syncope is an abrupt loss of consciousness due to transient global cerebral hypoperfusion, with a concomitant loss of postural tone and rapid, spontaneous recovery.1 Recovery from syncope is characterized by immediate restoration of orientation and normal behavior, although the period after recovery may be accompanied by fatigue.2
The European Society of Cardiology2 has classified syncope into 3 main categories: reflex (neurally mediated) syncope, syncope due to orthostatic hypotension, and cardiac syncope. Determining the cause is critical, as this determines the prognosis.
KEYS TO THE EVALUATION
According to the 2017 American College of Cardiology/American Heart Association (ACC/AHA) and the 2009 European Society of Cardiology guidelines, the evaluation of syncope should include a thorough history, taken from the patient and witnesses, and a complete physical examination. This can identify the cause of syncope in up to 50% of cases and differentiate between cardiac and noncardiac causes. Features that point to cardiac syncope include age older than 60, male sex, known heart disease, brief prodrome, syncope during exertion or when supine, first syncopal event, family history of sudden cardiac death, and abnormal physical examination.1
Features that suggest noncardiac syncope are young age; syncope only when standing; recurrent syncope; a prodrome of nausea, vomiting, and a warm sensation; and triggers such as dehydration, pain, distressful stimulus, cough, laugh micturition, defecation, and swallowing.1
Electrocardiography should follow the history and physical examination. When done at presentation, electrocardiography is diagnostic in only about 5% of cases. However, given the importance of the diagnosis, it remains an essential part of the initial evaluation of syncope.3
If a clear cause of syncope is identified at this point, no further workup is needed, and the cause of syncope should be addressed.1 If the cause is still unclear, the ACC/AHA guidelines recommend further evaluation based on the clinical presentation and risk stratification.
WHEN TO PURSUE ADDITIONAL TESTING
Routine use of additional testing is costly; tests should be ordered on the basis of their potential diagnostic and prognostic value. Additional evaluation should follow a stepwise approach and can include targeted blood work, autonomic nerve evaluation, tilt-table testing, transthoracic echocardiography, stress testing, electrocardiographic monitoring, and electrophysiologic testing.1
Syncope is rarely a manifestation of neurologic disease, yet 11% to 58% of patients with a first episode of uncomplicated syncope undergo extensive neuroimaging with magnetic resonance imaging, computed tomography, electroencephalography (EEG), and carotid ultrasonography.4 Evidence suggests that routine neurologic testing is of limited value given its low diagnostic yield and high cost.
Epilepsy is the most common neurologic cause of loss of consciousness but is estimated to account for less than 5% of patients with syncope.5 A thorough and thoughtful neurologic history and examination is often enough to distinguish between syncope, convulsive syncope, epileptic convulsions, and pseudosyncope.
In syncope, the loss of consciousness usually occurs 30 seconds to several minutes after standing. It presents with or without a prodrome (warmth, palpitations, and diaphoresis) and can be relieved with supine positioning. True loss of consciousness usually lasts less than a minute and is accompanied by loss of postural tone, with little or no fatigue in the recovery period.6
Conversely, in convulsive syncope, the prodrome can include pallor and diaphoresis. Loss of consciousness lasts about 30 seconds but is accompanied by fixed gaze, upward eye deviation, nuchal rigidity, tonic spasms, myoclonic jerks, tonic-clonic convulsions, and oral automatisms.6
Pseudosyncope is characterized by a prodrome of lightheadedness, shortness of breath, chest pain, and tingling sensations, followed by episodes of apparent loss of consciousness that last longer than several minutes and occur multiple times a day. During these episodes, patients purposefully try to avoid trauma when they lose consciousness, and almost always keep their eyes closed, in contrast to syncopal episodes, when the eyes are open and glassy.7
ROLE OF ELECTROENCEPHALOGRAPHY
If the diagnosis remains unclear after the history and neurologic examination, EEG is recommended (class IIa, ie, reasonable, can be useful) during tilt-table testing, as it can help differentiate syncope, pseudosyncope, and epilepsy.1
In an epileptic convulsion, EEG shows epileptiform discharges, whereas in syncope, it shows diffuse brainwave slowing with delta waves and a flatline pattern. In pseudosyncope and psychogenic nonepileptic seizures, EEG shows normal activity.8
Routine EEG is not recommended if there are no specific neurologic signs of epilepsy or if the history and neurologic examination indicate syncope or pseudosyncope.1
Structural brain disease does not typically present with transient global cerebral hypoperfusion resulting in syncope, so magnetic resonance imaging and computed tomography have a low diagnostic yield. Studies have revealed that for the 11% to 58% of patients who undergo neuroimaging, it establishes a diagnosis in only 0.2% to 1%.9 For this reason and in view of their high cost, these imaging tests should not be routinely ordered in the evaluation of syncope.4,10 Similarly, carotid artery imaging should not be routinely ordered if there is no focal neurologic finding suggesting unilateral ischemia.10
CASE CONTINUED
In our 40-year-old patient, the history suggests dehydration, as she recently started taking a diuretic. Thus, laboratory testing is reasonable.
Loss of bladder control is often interpreted as a red flag for neurologic disease, but syncope can often present with urinary incontinence. Urinary incontinence may also occur in epileptic seizure and in nonepileptic events such as syncope. A pooled analysis by Brigo et al11 determined that urinary incontinence had no value in distinguishing between epilepsy and syncope. Therefore, this physical finding should not incline the clinician to one diagnosis or the other.
Given our patient’s presentation, findings on physical examination, and absence of focal neurologic deficits, she should not undergo neuroimaging for syncope evaluation. The more likely cause of her syncope is orthostatic intolerance (orthostatic hypotension or vasovagal syncope) in the setting of intravascular volume depletion, likely secondary to diuretic use. Obtaining orthostatic vital signs is mandatory, and this confirms the diagnosis.
- Shen WK, Sheldon RS, Benditt DG, et al. 2017 ACC/AHA/HRS guideline for the evaluation and management of patients with syncope: a report of the American College of Cardiology/American Heart Association Task Force on Clinical Practice Guidelines and the Heart Rhythm Society. J Am Coll Cardiol 2017; 70(5):e39–e110. doi:10.1016/j.jacc.2017.03.003
- Task Force for the Diagnosis and Management of Syncope; European Society of Cardiology (ESC); European Heart Rhythm Association (EHRA); Heart Failure Association (HFA); Heart Rhythm Society (HRS), Moya A, Sutton R, Ammirati F, et al. Guidelines for the diagnosis and management of syncope (version 2009). Eur Heart J 2009; 30(21):2631–2671. doi:10.1093/eurheartj/ehp298
- Mehlsen J, Kaijer MN, Mehlsen AB. Autonomic and electrocardiographic changes in cardioinhibitory syncope. Europace 2008; 10(1):91–95. doi:10.1093/europace/eum237
- Goyal N, Donnino MW, Vachhani R, Bajwa R, Ahmad T, Otero R. The utility of head computed tomography in the emergency department evaluation of syncope. Intern Emerg Med 2006; 1(2):148–150. pmid:17111790
- Kapoor WN, Karpf M, Wieand S, Peterson JR, Levey GS. A prospective evaluation and follow-up of patients with syncope. N Engl J Med 1983; 309(4):197–204. doi:10.1056/NEJM198307283090401
- Sheldon R. How to differentiate syncope from seizure. Cardiol Clin 2015; 33(3):377–385. doi:10.1016/j.ccl.2015.04.006
- Raj V, Rowe AA, Fleisch SB, Paranjape SY, Arain AM, Nicolson SE. Psychogenic pseudosyncope: diagnosis and management. Auton Neurosci 2014; 184:66–72. doi:10.1016/j.autneu.2014.05.003
- Mecarelli O, Pulitano P, Vicenzini E, Vanacore N, Accornero N, De Marinis M. Observations on EEG patterns in neurally-mediated syncope: an inspective and quantitative study. Neurophysiol Clin 2004; 34(5):203–207. doi:10.1016/j.neucli.2004.09.004
- Johnson PC, Ammar H, Zohdy W, Fouda R, Govindu R. Yield of diagnostic tests and its impact on cost in adult patients with syncope presenting to a community hospital. South Med J 2014; 107(11):707–714. doi:10.14423/SMJ.0000000000000184
- Sclafani JJ, My J, Zacher LL, Eckart RE. Intensive education on evidence-based evaluation of syncope increases sudden death risk stratification but fails to reduce use of neuroimaging. Arch Intern Med 2010; 170(13):1150–1154. doi:10.1001/archinternmed.2010.205
- Brigo F, Nardone R Ausserer H, et al. The diagnostic value of urinary incontinence in the differential diagnosis of seizures. Seizure 2013; 22(2):85–90. doi:10.1016/j.seizure.2012.10.011
- Shen WK, Sheldon RS, Benditt DG, et al. 2017 ACC/AHA/HRS guideline for the evaluation and management of patients with syncope: a report of the American College of Cardiology/American Heart Association Task Force on Clinical Practice Guidelines and the Heart Rhythm Society. J Am Coll Cardiol 2017; 70(5):e39–e110. doi:10.1016/j.jacc.2017.03.003
- Task Force for the Diagnosis and Management of Syncope; European Society of Cardiology (ESC); European Heart Rhythm Association (EHRA); Heart Failure Association (HFA); Heart Rhythm Society (HRS), Moya A, Sutton R, Ammirati F, et al. Guidelines for the diagnosis and management of syncope (version 2009). Eur Heart J 2009; 30(21):2631–2671. doi:10.1093/eurheartj/ehp298
- Mehlsen J, Kaijer MN, Mehlsen AB. Autonomic and electrocardiographic changes in cardioinhibitory syncope. Europace 2008; 10(1):91–95. doi:10.1093/europace/eum237
- Goyal N, Donnino MW, Vachhani R, Bajwa R, Ahmad T, Otero R. The utility of head computed tomography in the emergency department evaluation of syncope. Intern Emerg Med 2006; 1(2):148–150. pmid:17111790
- Kapoor WN, Karpf M, Wieand S, Peterson JR, Levey GS. A prospective evaluation and follow-up of patients with syncope. N Engl J Med 1983; 309(4):197–204. doi:10.1056/NEJM198307283090401
- Sheldon R. How to differentiate syncope from seizure. Cardiol Clin 2015; 33(3):377–385. doi:10.1016/j.ccl.2015.04.006
- Raj V, Rowe AA, Fleisch SB, Paranjape SY, Arain AM, Nicolson SE. Psychogenic pseudosyncope: diagnosis and management. Auton Neurosci 2014; 184:66–72. doi:10.1016/j.autneu.2014.05.003
- Mecarelli O, Pulitano P, Vicenzini E, Vanacore N, Accornero N, De Marinis M. Observations on EEG patterns in neurally-mediated syncope: an inspective and quantitative study. Neurophysiol Clin 2004; 34(5):203–207. doi:10.1016/j.neucli.2004.09.004
- Johnson PC, Ammar H, Zohdy W, Fouda R, Govindu R. Yield of diagnostic tests and its impact on cost in adult patients with syncope presenting to a community hospital. South Med J 2014; 107(11):707–714. doi:10.14423/SMJ.0000000000000184
- Sclafani JJ, My J, Zacher LL, Eckart RE. Intensive education on evidence-based evaluation of syncope increases sudden death risk stratification but fails to reduce use of neuroimaging. Arch Intern Med 2010; 170(13):1150–1154. doi:10.1001/archinternmed.2010.205
- Brigo F, Nardone R Ausserer H, et al. The diagnostic value of urinary incontinence in the differential diagnosis of seizures. Seizure 2013; 22(2):85–90. doi:10.1016/j.seizure.2012.10.011
Patient selection for acute stroke thrombectomy stirs controversy
HONOLULU – A little more than a year ago, results from the DAWN and DEFUSE 3 trials substantially broadened the time window for endovascular thrombectomy of acute ischemic stroke by selecting patients using brain imaging. Stroke clinicians are now trying to reconcile widespread, routine use of this life-changing treatment against an uncertain need to replicate the higher-end perfusion CT and analytical software imaging that these landmark trials used for patient selection. This has produced a schism in what experts advise for using endovascular thrombectomy on acute ischemic stroke patients.
“Go open the artery, people!” Michael D. Hill, MD, exhorted during a talk at the International Stroke Conference, sponsored by the American Heart Association. “Don’t get over-selective; more people will benefit than you think,” said Dr. Hill, a professor of clinical neurosciences at the University of Calgary (Alta.).
“We are over-selecting, and depriving patients,” commented Raul C. Nogueira, MD, professor of neurology at Emory University in Atlanta and a lead investigator of the DAWN trial, speaking from the audience during a discussion at the session where Dr. Hill spoke.
“The prevalence of treatable [acute ischemic stroke] patients is far higher than the prevalence of patients who are not good candidates, so you just want to exclude the ‘wipe-outs’; that’s what we do,” Dr. Hill explained. “Fortunately, endovascular therapy is very safe, and you’re not going to harm many patients. With other treatments [used routinely in medicine] some patients don’t benefit, but when you have a large effect size we use the treatment on almost everyone. The effect size from thrombectomy is so large it’s an argument to treat almost everyone, although the patients in the trials were selected by imaging.”
Dr. Hill repeatedly stressed that for most patients a non-contrast CT image is usually adequate to identify patients with salvageable brain tissue and a low risk for hemorrhage from intervention, and he endorsed also doing CT angiography to further inform the diagnosis. But he dismissed CT perfusion imaging as unnecessary. “Noncontrast CT and CT angiography are more than adequate to make treatment decisions,” he said. “The prevalence of poor collaterals is quite low, about 10%,” which means that about 90% of acute ischemic stroke patients will have more slowly progressing infarction,” making them amenable to treatment in an expanded time window and boosting the volume of salvageable tissue.
But these appeals for more liberal use of thrombectomy without the perfusion CT imaging used in DAWN (N Engl J Med. 2018 Jan 4;378[1]:11-21)and DEFUSE-3 (N Engl J Med. 2018 Feb 22;378[8]:708-18) received push back. Maarten G. Lansberg, MD, a co-investigator on the DEFUSE 3 trial, highlighted the speed and simplicity of CT perfusion imaging, and its utility in helping to better target thrombectomy to the right patients. It’s “speedy, simple, and safe,” it “excludes patients who will not benefit” from thrombectomy, and it helps when the patient’s history and noncontrast CT images are inconclusive, said Dr. Lansberg, a neurologist at Stanford (Calif.) University.
“Clinical presentation will only tell you so much.” With imaging that includes CT perfusion, “you can find out, in 5, 10 minutes, whether there is an occlusion, its location, the extent of dead tissue – that’s all really helpful,” said Marc Fisher, MD, professor of neurology at Harvard Medical School in Boston. “There is a tension now between doing treatment really fast and the concept of slow and fast evolvers. For slow evolvers, the concern about speed is irrelevant because it can take days” for their brains to have substantial damage. “For the fast evolvers, time matters, but they could also possibly be harmed; that’s why we need more data.”
A pitch for more data also came from Pooja Khatri, MD, who also spoke at the session. “There is a real tension now between personalizing the imaging and figuring out exactly the right patients against the time trade off for doing that. Some argue to keep it simple and move fast, and by doing that you’ll wash out any difference from doing more fancy stuff. Plus some places, even in developed countries, can’t afford the image-processing software” used in the DAWN and DEFUSE 3 trials. The correct approach remains unclear and has created “an area ripe for a trial,” declared Dr. Khatri, professor of neurology at director of acute stroke at the University of Cincinnati.
Dr. Hill has received honoraria from Merck and received research funding from Boehringer Ingelheim, Covidien, Medtronic, and Stryker. He has an ownership interest in Calgary Scientific and holds a patent on acute stroke triage methods. Dr. Nogueira has financial relationships with many companies. Dr. Lansberg and Dr. Fisher had no disclosures. Dr. Khatri has been a consultant to Lumosa and has received research funding from Cerenovus/Johnson & Johnson, Genentech, and Nervive.
HONOLULU – A little more than a year ago, results from the DAWN and DEFUSE 3 trials substantially broadened the time window for endovascular thrombectomy of acute ischemic stroke by selecting patients using brain imaging. Stroke clinicians are now trying to reconcile widespread, routine use of this life-changing treatment against an uncertain need to replicate the higher-end perfusion CT and analytical software imaging that these landmark trials used for patient selection. This has produced a schism in what experts advise for using endovascular thrombectomy on acute ischemic stroke patients.
“Go open the artery, people!” Michael D. Hill, MD, exhorted during a talk at the International Stroke Conference, sponsored by the American Heart Association. “Don’t get over-selective; more people will benefit than you think,” said Dr. Hill, a professor of clinical neurosciences at the University of Calgary (Alta.).
“We are over-selecting, and depriving patients,” commented Raul C. Nogueira, MD, professor of neurology at Emory University in Atlanta and a lead investigator of the DAWN trial, speaking from the audience during a discussion at the session where Dr. Hill spoke.
“The prevalence of treatable [acute ischemic stroke] patients is far higher than the prevalence of patients who are not good candidates, so you just want to exclude the ‘wipe-outs’; that’s what we do,” Dr. Hill explained. “Fortunately, endovascular therapy is very safe, and you’re not going to harm many patients. With other treatments [used routinely in medicine] some patients don’t benefit, but when you have a large effect size we use the treatment on almost everyone. The effect size from thrombectomy is so large it’s an argument to treat almost everyone, although the patients in the trials were selected by imaging.”
Dr. Hill repeatedly stressed that for most patients a non-contrast CT image is usually adequate to identify patients with salvageable brain tissue and a low risk for hemorrhage from intervention, and he endorsed also doing CT angiography to further inform the diagnosis. But he dismissed CT perfusion imaging as unnecessary. “Noncontrast CT and CT angiography are more than adequate to make treatment decisions,” he said. “The prevalence of poor collaterals is quite low, about 10%,” which means that about 90% of acute ischemic stroke patients will have more slowly progressing infarction,” making them amenable to treatment in an expanded time window and boosting the volume of salvageable tissue.
But these appeals for more liberal use of thrombectomy without the perfusion CT imaging used in DAWN (N Engl J Med. 2018 Jan 4;378[1]:11-21)and DEFUSE-3 (N Engl J Med. 2018 Feb 22;378[8]:708-18) received push back. Maarten G. Lansberg, MD, a co-investigator on the DEFUSE 3 trial, highlighted the speed and simplicity of CT perfusion imaging, and its utility in helping to better target thrombectomy to the right patients. It’s “speedy, simple, and safe,” it “excludes patients who will not benefit” from thrombectomy, and it helps when the patient’s history and noncontrast CT images are inconclusive, said Dr. Lansberg, a neurologist at Stanford (Calif.) University.
“Clinical presentation will only tell you so much.” With imaging that includes CT perfusion, “you can find out, in 5, 10 minutes, whether there is an occlusion, its location, the extent of dead tissue – that’s all really helpful,” said Marc Fisher, MD, professor of neurology at Harvard Medical School in Boston. “There is a tension now between doing treatment really fast and the concept of slow and fast evolvers. For slow evolvers, the concern about speed is irrelevant because it can take days” for their brains to have substantial damage. “For the fast evolvers, time matters, but they could also possibly be harmed; that’s why we need more data.”
A pitch for more data also came from Pooja Khatri, MD, who also spoke at the session. “There is a real tension now between personalizing the imaging and figuring out exactly the right patients against the time trade off for doing that. Some argue to keep it simple and move fast, and by doing that you’ll wash out any difference from doing more fancy stuff. Plus some places, even in developed countries, can’t afford the image-processing software” used in the DAWN and DEFUSE 3 trials. The correct approach remains unclear and has created “an area ripe for a trial,” declared Dr. Khatri, professor of neurology at director of acute stroke at the University of Cincinnati.
Dr. Hill has received honoraria from Merck and received research funding from Boehringer Ingelheim, Covidien, Medtronic, and Stryker. He has an ownership interest in Calgary Scientific and holds a patent on acute stroke triage methods. Dr. Nogueira has financial relationships with many companies. Dr. Lansberg and Dr. Fisher had no disclosures. Dr. Khatri has been a consultant to Lumosa and has received research funding from Cerenovus/Johnson & Johnson, Genentech, and Nervive.
HONOLULU – A little more than a year ago, results from the DAWN and DEFUSE 3 trials substantially broadened the time window for endovascular thrombectomy of acute ischemic stroke by selecting patients using brain imaging. Stroke clinicians are now trying to reconcile widespread, routine use of this life-changing treatment against an uncertain need to replicate the higher-end perfusion CT and analytical software imaging that these landmark trials used for patient selection. This has produced a schism in what experts advise for using endovascular thrombectomy on acute ischemic stroke patients.
“Go open the artery, people!” Michael D. Hill, MD, exhorted during a talk at the International Stroke Conference, sponsored by the American Heart Association. “Don’t get over-selective; more people will benefit than you think,” said Dr. Hill, a professor of clinical neurosciences at the University of Calgary (Alta.).
“We are over-selecting, and depriving patients,” commented Raul C. Nogueira, MD, professor of neurology at Emory University in Atlanta and a lead investigator of the DAWN trial, speaking from the audience during a discussion at the session where Dr. Hill spoke.
“The prevalence of treatable [acute ischemic stroke] patients is far higher than the prevalence of patients who are not good candidates, so you just want to exclude the ‘wipe-outs’; that’s what we do,” Dr. Hill explained. “Fortunately, endovascular therapy is very safe, and you’re not going to harm many patients. With other treatments [used routinely in medicine] some patients don’t benefit, but when you have a large effect size we use the treatment on almost everyone. The effect size from thrombectomy is so large it’s an argument to treat almost everyone, although the patients in the trials were selected by imaging.”
Dr. Hill repeatedly stressed that for most patients a non-contrast CT image is usually adequate to identify patients with salvageable brain tissue and a low risk for hemorrhage from intervention, and he endorsed also doing CT angiography to further inform the diagnosis. But he dismissed CT perfusion imaging as unnecessary. “Noncontrast CT and CT angiography are more than adequate to make treatment decisions,” he said. “The prevalence of poor collaterals is quite low, about 10%,” which means that about 90% of acute ischemic stroke patients will have more slowly progressing infarction,” making them amenable to treatment in an expanded time window and boosting the volume of salvageable tissue.
But these appeals for more liberal use of thrombectomy without the perfusion CT imaging used in DAWN (N Engl J Med. 2018 Jan 4;378[1]:11-21)and DEFUSE-3 (N Engl J Med. 2018 Feb 22;378[8]:708-18) received push back. Maarten G. Lansberg, MD, a co-investigator on the DEFUSE 3 trial, highlighted the speed and simplicity of CT perfusion imaging, and its utility in helping to better target thrombectomy to the right patients. It’s “speedy, simple, and safe,” it “excludes patients who will not benefit” from thrombectomy, and it helps when the patient’s history and noncontrast CT images are inconclusive, said Dr. Lansberg, a neurologist at Stanford (Calif.) University.
“Clinical presentation will only tell you so much.” With imaging that includes CT perfusion, “you can find out, in 5, 10 minutes, whether there is an occlusion, its location, the extent of dead tissue – that’s all really helpful,” said Marc Fisher, MD, professor of neurology at Harvard Medical School in Boston. “There is a tension now between doing treatment really fast and the concept of slow and fast evolvers. For slow evolvers, the concern about speed is irrelevant because it can take days” for their brains to have substantial damage. “For the fast evolvers, time matters, but they could also possibly be harmed; that’s why we need more data.”
A pitch for more data also came from Pooja Khatri, MD, who also spoke at the session. “There is a real tension now between personalizing the imaging and figuring out exactly the right patients against the time trade off for doing that. Some argue to keep it simple and move fast, and by doing that you’ll wash out any difference from doing more fancy stuff. Plus some places, even in developed countries, can’t afford the image-processing software” used in the DAWN and DEFUSE 3 trials. The correct approach remains unclear and has created “an area ripe for a trial,” declared Dr. Khatri, professor of neurology at director of acute stroke at the University of Cincinnati.
Dr. Hill has received honoraria from Merck and received research funding from Boehringer Ingelheim, Covidien, Medtronic, and Stryker. He has an ownership interest in Calgary Scientific and holds a patent on acute stroke triage methods. Dr. Nogueira has financial relationships with many companies. Dr. Lansberg and Dr. Fisher had no disclosures. Dr. Khatri has been a consultant to Lumosa and has received research funding from Cerenovus/Johnson & Johnson, Genentech, and Nervive.
REPORTING FROM ISC 2019
Developing clinical mastery at HM19
Boosting your bedside diagnostic skills
A new three-session minitrack devoted to the clinical mastery of diagnostic and treatment skills at the hospitalized patient’s bedside should be a highlight of the Society of Hospital Medicine’s 2019 annual conference.
The “Clinical Mastery” track is designed to help hospitalists enhance their skills in making expert diagnoses at the bedside, said Dustin T. Smith, MD, SFHM, course director for HM19, and associate professor of medicine at Emory University, Atlanta. “We feel that all of the didactic sessions offered at HM19 are highly useful for hospitalists, but there is growing interest in having sessions devoted to learning clinical pearls that can aid in practicing medicine and acquiring the skill set of a master clinician.”
The three clinical mastery sessions at HM19 will address neurologic symptoms, ECG interpretation, and the role of point-of-care ultrasound (POCUS), currently a hot topic in hospital medicine. Recent advances in ultrasound technology have resulted in probes that can cost as little as $2,000, fit inside a lab coat pocket, and be read from a smartphone – making ultrasound far easier to bring to the bedside of hospitalized patients, said Ria Dancel, MD, FHM, associate professor of internal medicine and pediatrics at the University of North Carolina at Chapel Hill.
Dr. Dancel will copresent the POCUS clinical mastery track at HM19. “Our focus will be on how POCUS and the physical exam relate to each other. These are not competing technologies but complementary, reflecting the evolution in bedside medicine. Because these new devices will soon be in the pockets of your colleagues, residents, physician assistants, and others, you should at least have the knowledge and vocabulary to communicate with them,” she said.
POCUS is a new technology that is not yet in wide use at the hospital bedside, but clearly a wave is building, said Dr. Dancel’s copresenter, Michael Janjigian, MD, associate professor in the department of medicine at NYU Langone Health in New York City.
“We’re at the inflection point where the cost of the machine and the availability of training means that hospitals need to decide if it’s time to embrace it,” he said. Hospitalists may also consider petitioning their hospital’s leadership to offer the machines and training.
“Hospitalists’ competencies and strengths lie primarily in making diagnoses,” Dr. Janjigian said. “We like to think of ourselves as master diagnosticians. Our session at HM19 will explore the strengths and weaknesses of both the physical exam and POCUS, presenting clinical scenarios common to hospital medicine. This course is designed for those who have never picked up an ultrasound probe and want to better understand why they should, and for those who want a better sense of how they might integrate it into their practice.”
While radiology and cardiology have been using ultrasound for decades, internists are finding uses at the bedside to speed diagnosis or focus their next diagnostic steps, Dr. Dancel noted. For certain diagnoses, the physical exam is still the tool of choice. But when looking for fluid around the heart or ascites buildup in the abdomen or when looking at the heart itself, she said, there is no better tool at the bedside than ultrasound.
In January 2019, the SHM issued a position statement on POCUS1, which is intended to inform hospitalists about the technology and its uses, encourage them to be more integrally involved in decision making processes surrounding POCUS program management for their hospitals, and promote development of standards for hospitalists in POCUS training and assessment. The SHM has also developed a pathway to teach the use of ultrasound, the Point-of-Care Ultrasound Certificate of Completion.
In order to qualify, clinicians complete online training modules, attend two live learning courses, compile a portfolio of ultrasound video clips on the job that are reviewed by a panel of experts, and then pass a final exam. The exam will be offered at HM19 for clinicians who have completed preliminary work for this new certificate – as well as precourses devoted to ultrasound and other procedures – and another workshop on POCUS.
Earning the POCUS certificate of completion requires a lot of effort, Dr. Dancel acknowledged. “It is a big commitment, and we don’t want hospitalists thinking that just because they have completed the certificate that they have fully mastered ultrasound. We encourage hospitalists to find a proctor in their own hospitals and to work with them to continue to refine their skills.”
Dr. Dancel and Dr. Janjigian reported no relevant disclosures.
References
1. Soni NJ et al. Point-of-care ultrasound for hospitalists: A position statement of the Society of Hospital Medicine. J Hosp Med. 2019 Jan 2. doi: 10.12788/jhm.3079.
Boosting your bedside diagnostic skills
Boosting your bedside diagnostic skills
A new three-session minitrack devoted to the clinical mastery of diagnostic and treatment skills at the hospitalized patient’s bedside should be a highlight of the Society of Hospital Medicine’s 2019 annual conference.
The “Clinical Mastery” track is designed to help hospitalists enhance their skills in making expert diagnoses at the bedside, said Dustin T. Smith, MD, SFHM, course director for HM19, and associate professor of medicine at Emory University, Atlanta. “We feel that all of the didactic sessions offered at HM19 are highly useful for hospitalists, but there is growing interest in having sessions devoted to learning clinical pearls that can aid in practicing medicine and acquiring the skill set of a master clinician.”
The three clinical mastery sessions at HM19 will address neurologic symptoms, ECG interpretation, and the role of point-of-care ultrasound (POCUS), currently a hot topic in hospital medicine. Recent advances in ultrasound technology have resulted in probes that can cost as little as $2,000, fit inside a lab coat pocket, and be read from a smartphone – making ultrasound far easier to bring to the bedside of hospitalized patients, said Ria Dancel, MD, FHM, associate professor of internal medicine and pediatrics at the University of North Carolina at Chapel Hill.
Dr. Dancel will copresent the POCUS clinical mastery track at HM19. “Our focus will be on how POCUS and the physical exam relate to each other. These are not competing technologies but complementary, reflecting the evolution in bedside medicine. Because these new devices will soon be in the pockets of your colleagues, residents, physician assistants, and others, you should at least have the knowledge and vocabulary to communicate with them,” she said.
POCUS is a new technology that is not yet in wide use at the hospital bedside, but clearly a wave is building, said Dr. Dancel’s copresenter, Michael Janjigian, MD, associate professor in the department of medicine at NYU Langone Health in New York City.
“We’re at the inflection point where the cost of the machine and the availability of training means that hospitals need to decide if it’s time to embrace it,” he said. Hospitalists may also consider petitioning their hospital’s leadership to offer the machines and training.
“Hospitalists’ competencies and strengths lie primarily in making diagnoses,” Dr. Janjigian said. “We like to think of ourselves as master diagnosticians. Our session at HM19 will explore the strengths and weaknesses of both the physical exam and POCUS, presenting clinical scenarios common to hospital medicine. This course is designed for those who have never picked up an ultrasound probe and want to better understand why they should, and for those who want a better sense of how they might integrate it into their practice.”
While radiology and cardiology have been using ultrasound for decades, internists are finding uses at the bedside to speed diagnosis or focus their next diagnostic steps, Dr. Dancel noted. For certain diagnoses, the physical exam is still the tool of choice. But when looking for fluid around the heart or ascites buildup in the abdomen or when looking at the heart itself, she said, there is no better tool at the bedside than ultrasound.
In January 2019, the SHM issued a position statement on POCUS1, which is intended to inform hospitalists about the technology and its uses, encourage them to be more integrally involved in decision making processes surrounding POCUS program management for their hospitals, and promote development of standards for hospitalists in POCUS training and assessment. The SHM has also developed a pathway to teach the use of ultrasound, the Point-of-Care Ultrasound Certificate of Completion.
In order to qualify, clinicians complete online training modules, attend two live learning courses, compile a portfolio of ultrasound video clips on the job that are reviewed by a panel of experts, and then pass a final exam. The exam will be offered at HM19 for clinicians who have completed preliminary work for this new certificate – as well as precourses devoted to ultrasound and other procedures – and another workshop on POCUS.
Earning the POCUS certificate of completion requires a lot of effort, Dr. Dancel acknowledged. “It is a big commitment, and we don’t want hospitalists thinking that just because they have completed the certificate that they have fully mastered ultrasound. We encourage hospitalists to find a proctor in their own hospitals and to work with them to continue to refine their skills.”
Dr. Dancel and Dr. Janjigian reported no relevant disclosures.
References
1. Soni NJ et al. Point-of-care ultrasound for hospitalists: A position statement of the Society of Hospital Medicine. J Hosp Med. 2019 Jan 2. doi: 10.12788/jhm.3079.
A new three-session minitrack devoted to the clinical mastery of diagnostic and treatment skills at the hospitalized patient’s bedside should be a highlight of the Society of Hospital Medicine’s 2019 annual conference.
The “Clinical Mastery” track is designed to help hospitalists enhance their skills in making expert diagnoses at the bedside, said Dustin T. Smith, MD, SFHM, course director for HM19, and associate professor of medicine at Emory University, Atlanta. “We feel that all of the didactic sessions offered at HM19 are highly useful for hospitalists, but there is growing interest in having sessions devoted to learning clinical pearls that can aid in practicing medicine and acquiring the skill set of a master clinician.”
The three clinical mastery sessions at HM19 will address neurologic symptoms, ECG interpretation, and the role of point-of-care ultrasound (POCUS), currently a hot topic in hospital medicine. Recent advances in ultrasound technology have resulted in probes that can cost as little as $2,000, fit inside a lab coat pocket, and be read from a smartphone – making ultrasound far easier to bring to the bedside of hospitalized patients, said Ria Dancel, MD, FHM, associate professor of internal medicine and pediatrics at the University of North Carolina at Chapel Hill.
Dr. Dancel will copresent the POCUS clinical mastery track at HM19. “Our focus will be on how POCUS and the physical exam relate to each other. These are not competing technologies but complementary, reflecting the evolution in bedside medicine. Because these new devices will soon be in the pockets of your colleagues, residents, physician assistants, and others, you should at least have the knowledge and vocabulary to communicate with them,” she said.
POCUS is a new technology that is not yet in wide use at the hospital bedside, but clearly a wave is building, said Dr. Dancel’s copresenter, Michael Janjigian, MD, associate professor in the department of medicine at NYU Langone Health in New York City.
“We’re at the inflection point where the cost of the machine and the availability of training means that hospitals need to decide if it’s time to embrace it,” he said. Hospitalists may also consider petitioning their hospital’s leadership to offer the machines and training.
“Hospitalists’ competencies and strengths lie primarily in making diagnoses,” Dr. Janjigian said. “We like to think of ourselves as master diagnosticians. Our session at HM19 will explore the strengths and weaknesses of both the physical exam and POCUS, presenting clinical scenarios common to hospital medicine. This course is designed for those who have never picked up an ultrasound probe and want to better understand why they should, and for those who want a better sense of how they might integrate it into their practice.”
While radiology and cardiology have been using ultrasound for decades, internists are finding uses at the bedside to speed diagnosis or focus their next diagnostic steps, Dr. Dancel noted. For certain diagnoses, the physical exam is still the tool of choice. But when looking for fluid around the heart or ascites buildup in the abdomen or when looking at the heart itself, she said, there is no better tool at the bedside than ultrasound.
In January 2019, the SHM issued a position statement on POCUS1, which is intended to inform hospitalists about the technology and its uses, encourage them to be more integrally involved in decision making processes surrounding POCUS program management for their hospitals, and promote development of standards for hospitalists in POCUS training and assessment. The SHM has also developed a pathway to teach the use of ultrasound, the Point-of-Care Ultrasound Certificate of Completion.
In order to qualify, clinicians complete online training modules, attend two live learning courses, compile a portfolio of ultrasound video clips on the job that are reviewed by a panel of experts, and then pass a final exam. The exam will be offered at HM19 for clinicians who have completed preliminary work for this new certificate – as well as precourses devoted to ultrasound and other procedures – and another workshop on POCUS.
Earning the POCUS certificate of completion requires a lot of effort, Dr. Dancel acknowledged. “It is a big commitment, and we don’t want hospitalists thinking that just because they have completed the certificate that they have fully mastered ultrasound. We encourage hospitalists to find a proctor in their own hospitals and to work with them to continue to refine their skills.”
Dr. Dancel and Dr. Janjigian reported no relevant disclosures.
References
1. Soni NJ et al. Point-of-care ultrasound for hospitalists: A position statement of the Society of Hospital Medicine. J Hosp Med. 2019 Jan 2. doi: 10.12788/jhm.3079.